Você está na página 1de 92

Conjuntos A ssunto

1
Matemtica I

1. Conceitos
1.1 Elemento e conjunto Teorema 1 (nmero de subconjuntos):
Um conjunto A de n elementos possui 2n subconjuntos.
So conceitos primitivos, isto , no so definidos.
Se um elemento x pertence ao conjunto A, diz-se que x A. Caso
Dem.: Seja A = {a1, a2,...,an}. Para formar um subconjunto X de A,
contrrio, diz-se que x A.
devemos decidir, para cada elemento de A, se ele pertencer ou no a X.
Usam-se geralmente letras maisculas para representar conjuntos e Como temos n elementos e para cada elemento, temos duas possibilidades
letras minsculas para representar elementos. (ou ele est no subconjunto ou no est), o nmero de subconjuntos de
A 2 2 ... 2 = 2n

1.2 Conjunto vazio n vezes

O conjunto vazio aquele que no possui elementos. Representa-se 1.7 Conjunto universo
por ou { }. um conjunto que contm todos os elementos do contexto
envolvido e tambm todos os conjuntos desse contexto. Por exemplo, se
Ex.: {x N|1 < x < 2} estivermos em um problema envolvendo conjuntos de nmeros inteiros,
um possvel conjunto universo Z. Poderamos escolher tambm para
conjunto universo Q. Em geral, usa-se a letra U para representar o
1.3 Conjunto unitrio conjunto universo.
O conjunto unitrio aquele que possui apenas um elemento.
1.8 Diagramas de Venn
Ex.: A = {2012}, B = {x N|1 < x < 3}
Os diagramas de Venn so diagramas que mostram todas as possveis
relaes lgicas entre uma coleo finita de conjuntos. mais frequente o
1.4 Subconjunto uso de diagramas de Venn para representar dois ou trs conjuntos. Nesse
Um conjunto A um subconjunto de um conjunto B quando todos os caso, usamos crculos para represent-los.
elementos de A pertencem a B. Representamos a incluso de conjuntos
por A B (l-se A est contido em B) ou B A (l-se B contm A). Se Obs.: os diagramas de Venn so muito teis para resolver problemas
no ocorre a incluso, usamos (no est contido) ou / (no contm). de conjuntos, pois ajudam a organizar os dados do problema de forma
bastante clara, como veremos nos exerccios resolvidos.
Ex.: {1, 2} {1, 2, 3, 4} ; {1, 3} {2, 3, 4}
2. Operaes envolvendo conjuntos
Obs.: A (o conjunto vazio subconjunto de qualquer conjunto)
2.1 Unio
De fato, dizemos que A B se existe a A tal que a B. Dizer, A unio de dois conjuntos A e B o conjunto formado por todos os
portanto, que A significa dizer que existe elemento x tal que elementos que pertenam a A ou B (para que um elemento esteja na unio,
x A. Isso no possvel, j que o conjunto vazio no possui elemento. basta que ele pertena a pelo menos um dos conjuntos).

1.5 Igualdade de conjuntos A B = {x | x A ou x B}


Ex.:
Dois conjuntos A e B so ditos iguais quando todos os elementos de
A pertencem a B e vice-versa, isto , A B e B A. {1, 2} {3, 4} = {1, 2, 3, 4} ; {1, 2} {2, 3} = {1, 2, 3} ;
{1} = {1}
2.2 Interseo
1.6 Conjunto das partes (conjunto potncia)
A interseo de dois conjuntos A e B o conjunto formado pelos
o conjunto formado por todos os subconjuntos de um certo conjunto.
elementos que pertenam a A e a B (para que um elemento esteja na
O conjunto das partes representado por P(A) ou 2A (esta ltima no interseo, ele deve pertencer aos dois conjuntos).
to usual.)
A B = {x|x A e x B}
Ex.: Se S o conjunto de trs elementos {1, 2, 3}, a lista de Ex.:
subconjuntos de S : {1, 2, 3} {3, 4, 5} = {3} ; {1, 2, 3, 4} {1, 4} = {1, 4} ;
{1, 2, 3} {4, 5, 6} =
, {1}, {2}, {3}, {1, 2}, {1, 3}, {2, 3}, {1, 2, 3}
Obs.: Dois conjuntos A e B so ditos disjuntos quando A B =
Assim, o conjunto das partes de S P(S) = {, {1}, {2}, {3},
{1, 2}, {1, 3}, {2, 3}, {1, 2, 3}}. Repare que P(S) tem 8 = 23 elementos
e que S tem 3 elementos.

AFA-EFOMM 159
Matemtica I Assunto 1

2.3 Diferena de conjuntos Caso 2: x (B C)


Dados dois conjuntos A e B, definimos A B = {x tal que x A e x B} Aqui, x B e x C e ento x A B e x A C, o que nos d
(elementos que pertencem a A, mas no pertencem a B). x (A B) (A C).

Parte 2: A (B C) (A B) (A C)
Ex.: A = {1, 2, 3, 4, 5, 6} e B = {1, 2, 3, 6}. Temos que A B = {4 ,5}.

Anlogo parte 1 (mas em uma prova, voc deve escrever).


Obs.: Tambm representamos a diferena entre os conjuntos A e B
por A \ B. 3.2 Distributiva(da interseo em relao unio)

A (B C) = (A B) (A C)
Complementar
Dados dois conjuntos A e B com A B, o complementar de A em
relao a B o conjunto cujos elementos esto em B e no esto em 3.3 1a Lei de De Morgan
A. O complementar de A em relao a B denotado por CBA = B A.
O complementar de um conjunto A em relao ao conjunto universo (A B)C = AC BC
U, representado por AC (ou A), o conjunto formado pelos elementos
do universo U que no pertencem ao conjunto A.
Dem.:
Parte 1: (A B)C AC BC
Ex.: A = {1, 2, 3, 6} e B = {1, 2, 3, 4, 5, 6} ; CBA
= B A = {4, 5}
U = * e A = {x *|x > 4}. Temos que AC = {1, 2, 3}. Seja x (A B)C. Queremos provar que x AC BC. Como x (A
B), ento x A e x B. Com isso, segue que x AC e x BC, o que nos
d x AC BC.
2.4 Diferena simtrica
A diferena simtrica de dois conjuntos A e B o conjunto dos Parte 2: (A B)C AC BC:
elementos que pertencem unio dos dois conjuntos, mas no pertencem Anlogo parte 1.
interseo dos dois conjuntos.
3.4 2a Lei de De Morgan
A B = (A B) (B A) = (A B) (A B)
(A B)C = AC BC
Ex.: A = {1, 2, 3} e B = {2, 3, 4, 5}. Temos que A B = {1, 4, 5}.
3.5 A B = A BC
3. Propriedades 4. Princpio da incluso-excluso:
3.1 Distributiva (da unio em relao interseo) Para calcular o nmero de elementos de uma unio, usamos as
seguintes frmulas (vlidas apenas para conjuntos finitos):
A (B C) = (A B) (A C) Obs.: n(X) representar a quantidade de elementos de X.
4.1 Para dois conjuntos
Dem.:
A demonstrao consiste em duas partes (este passo padro em n(A B) = n(A) + n(B) n(A B)
demonstraes de igualdades de conjuntos):
Essa frmula expressa que para calcular o nmero de elementos
Parte 1: A (B C) (A B) (A C) de uma unio, no basta somar as quantidades de elementos dos dois
Seja x A (B C). Queremos provar que x (A B) (A C). conjuntos, pois alguns elementos podem ser contados duas vezes. Esses
Temos que, x A ou x (B C). Dividiremos ento em dois casos: elementos que so contados duas vezes so justamente os elementos da
interseo e, por isso, devemos retirar n(A B). Essa ideia se estende
Caso 1: x A de maneira anloga para mais conjuntos.
Aqui, como x A, segue que x A B e x A C e ento 4.2 Para trs conjuntos
x (A B) (A C) .
n(A B C) = n(A) + n(B) + n(C) n(A B) n(B C)
n (C A) + n(A B C)

160 Vol. 1
Conjuntos

EXERCCIOS RESOLVIDOS
01 (EN) Sendo A = {{1}, {2}, {1, 2}} pode-se afirmar que: Usando a informao (I) e o total de pacientes, temos que

a. {1} A x + y + 16 = 41
b. {1} A
3 x + 2 y + 24 = 75
c. {1} {2} A
d. 2 A
e. {1} {2} A Resolvendo o sistema, temos que x = 1 (e y = 24).

Soluo 03 (UFC) Sejam M e N conjuntos que possuem um nico elemento


a. F, pois {1} elemento de A ( verdade que {1} A) em comum. Se o nmero de subconjuntos de M igual ao dobro do
b. F, pois 1 no elemento de A nmero de subconjuntos de N, o nmero de elementos do conjunto M
c. F, pois {1} {2} = e sabemos que A para qualquer N :
conjunto A
d. F, pois 2 no elemento de A (A) o triplo do nmero de elementos de M.
e. V, pois {1} {2} = {1, 2} e {1, 2} elemento de A. (B) o triplo do nmero de elementos de N.
(C) o qudruplo do nmero de elementos de M.
Obs.: Usa-se que a A se existe uma cpia de a dentro do (D) o dobro do nmero de elementos de M.
conjunto A. Alm disso, usa-se {a} A se a A . Lembre, tambm, (E) o dobro do nmero de elementos de N.
que no h nenhum problema em um conjunto ser elemento de outro
conjunto (e neste caso, usamos o smbolo de pertence). Soluo: Letra E.
Sejam m e n as quantidades de elementos dos conjuntos M e N. O
02 (UNESP) Considere os pacientes da AIDS classificados em trs nmero de subconjuntos de M 2m e o nmero de subconjuntos de N
grupos de risco: hemoflicos, homossexuais e toxicmanos. Em um 2n. Do enunciado, temos que 2m = 2 2n m = n + 1. O nmero
certo pas, de 75 pacientes, verificou-se que: de elementos da unio M N dado por n(M) + n(N) n(M N) =
n + 1 + n 1 = 2n, que o dobro do nmero de elementos de N.
I. 41 so homossexuais;
II. 9 so homossexuais e hemoflicos, e no so toxicmanos; 04 (Princpio da incluso-excluso para trs conjuntos) Sendo A, B
III. 7 so homossexuais e toxicmanos, e no so hemoflicos; e C conjuntos, prove que:
IV. 2 so hemoflicos e toxicmanos, e no so homossexuais;
V. 6 pertencem apenas ao grupo de risco dos toxicmanos; n(A B C) = n(A) + n(B) +n(C) n(A B) n(B C)
VI. o nmero de pacientes que so apenas hemoflicos igual ao n(C A) + n(A B C)
nmero de pacientes que so apenas homossexuais;
VII. o nmero de pacientes que pertencem simultaneamente aos trs Soluo
grupos de risco a metade do nmero de pacientes que no
pertencem a nenhum dos grupos de risco. Vamos usar o mesmo princpio para dois conjuntos:

Quantos pacientes pertencem simultaneamente aos trs grupos de n(X Y) = n(X) +n(Y) n(X Y)
risco?
n ( A B C) = n ( A B) + n ( C) n (( A B) C)
Temos que: (*)
Soluo n ( A B ) = n ( A) + n ( B ) n ( A B )
Esse um problema clssico de conjuntos e a principal ideia montar
o diagrama de Venn com os trs conjuntos: A o conjunto dos
homossexuais, B o dos hemoflicos e C o dos toxicmanos. Sejam x o Usando a distributiva da interseo em relao unio, temos
nmero de pacientes que esto nos 3 grupos e y o nmero de pacientes que (A B) C = (A C) (B C). Usando mais uma vez
que so apenas homossexuais. Utilizando as informaes de (II) a (VII), o princpio da incluso-excluso com dois conjuntos, temos que
temos o seguinte diagrama: n (( A C) ( B C)) = n ( A C) + ( B C) n ( A B C) ( * * )
(aqui, usamos que (A C) (B C) = A B C.
A
Somando as equaes de (*) e substituindo (**), temos o resultado.
9 B
Obs. 1: O princpio bastante intuitivo e pode ser entendido de maneira
y y
informal atravs de um diagrama de Venn.
x
2 Obs. 2: Para demonstrar o caso geral (com n conjuntos), podemos
7
usar o princpio da induo finita ou um argumento de combinatria.
6
2x
C

AFA-EFOMM 161
Matemtica I Assunto 1

EXERCCIOS NVEL 1 V. O nmero de pessoas que assistem somente aos programas B e C


a metade dos que assistem somente a A e B;
VI. 25 s assistem a dois programas; e
01 Dado um conjunto E = {1, 2, 3, 4, 5} e trs subconjuntos de E, a
VII. 72 s assistem a um dos programas.
saber A, B e C, tais que:
Pode-se concluir que o nmero de pessoas que assistem
A B = {2, 4}; A B = {2, 3, 4, 5}; A C = {2, 3}; A C =
{1, 2, 3, 4}, determine C (B A) e A (B C).
(A) ao programa A 30.
(B) ao programa C 39.
02 (CMRJ) So dados os conjuntos A, B e C, tais que n(B C) = 18,
(C) aos trs programas 6.
n(A B) = 6, n(A C) = 5, n(A B C) = 2 e n(A B C) = 21.
(D) aos programas A e C 13.
O valor de n[A (B C)] :
(E) aos programas A ou B 63.
(A) 6. (D) 9.
05 (EPCAR) Para uma turma de 80 alunos do CPCAR, foi aplicada uma
(B) 7. (E) 10.
prova de matemtica valendo 9,0 pontos distribudos igualmente em 3
(C) 8.
questes sobre:
03 (CN) Sejam U o conjunto das brasileiras, A o conjunto das cariocas,
I. Funo;
B o conjunto das morenas e C o conjunto das mulheres de olhos azuis.
II. Geometria;
O diagrama que representa o conjunto de mulheres morenas ou de olhos
III. Polinmios.
azuis, e no cariocas; ou mulheres cariocas e no morenas e nem de olhos
azuis :
Sabe-se que:
(A) (D)
A A B apesar de 70% dos alunos terem acertado a questo sobre FUNO,
B
apenas 1/10 da turma conseguiu nota 9,0;
20 alunos acertaram as questes sobre FUNO e GEOMETRIA;
22 acertaram as questes sobre GEOMETRIA e POLINMIOS;
18 acertaram as questes sobre FUNO e POLINMIOS.

A turma estava completa nessa avaliao, ningum tirou nota zero, no


critrio de correo no houve questes com acertos parciais e o nmero
de acertos apenas em GEOMETRIA o mesmo que o nmero de acertos
C C apenas em POLINMIOS.
Nessas condies, correto afirmar que:
(B) (E)
A B A B
(A) o nmero de alunos que s acertaram a segunda questo o dobro
do nmero de alunos que acertaram todas as questes.
(B) metade da turma s acertou uma questo.
(C) mais de 50% da turma errou a terceira questo.
(D) apenas 3/4 da turma atingiu a mdia maior ou igual a 5,0.

06 (PUC) Numa comunidade constituda por 1800 pessoas h trs tipos


favoritos de programas de TV: Esporte (E), Novela (N) e Humorismo (H).
C C A tabela seguinte indica quantas pessoas assistem a estes programas.
(C) A B
Programas Nmero de espectadores
E 400
N 1220
H 1080
EeN 220
NeH 800
EeH 180
E, N e H 100
C

04 (CN) Em um grupo de 142 pessoas foi feita uma pesquisa sobre trs Atravs desses dados, calcule o nmero de pessoas da comunidade
programas de televiso A, B e C e constatou-se que: que no assiste a qualquer dos trs tipos de programa.

I. 40 no assistem a nenhum dos trs programas;


II. 103 no assistem ao programa C;
III. 25 s assistem ao programa B;
IV. 13 assistem aos programas A e B;

162 Vol. 1
Conjuntos

07 (UDESC) O Festival de Dana de Joinville considerado o maior do 12 (ITA) Sejam A um conjunto com 8 elementos e B um conjunto tal
mundo pelo Guinness Book of Records de 2005. Desde 1998, este festival que A B contenha 12 elementos. Ento, o nmero de elementos de
realizado no Centreventos Cau Hansen, que tem capacidade para 4.200 P (B \ A) P () igual a:
pessoas por noite.
Suponha que no 28o Festival de Dana, realizado em julho de 2010, houve (A) 8.
uma noite exclusiva para cada uma das seguintes modalidades: ballet, dana (B) 16.
de rua e jazz. A noite da dana de rua teve seus ingressos esgotados; na (C) 20.
noite do jazz restaram 5% dos ingressos; e a noite do ballet teve 90% dos (D) 17.
ingressos disponveis vendidos. Sabe-se que algumas pessoas costumam (E) 9.
prestigiar mais de uma noite do Festival. Neste ano, 700 pessoas assistiram
dana de rua e ao jazz; 1.610 assistiram ao ballet e dana de rua; 380 13 (UDESC) Considere em um conjunto universo, com 7 elementos, os
assistiram ao ballet e ao jazz e 105 prestigiaram as trs modalidades de dana. subconjuntos A, B e C, com 3, 5 e 7 elementos, respectivamente. correto
Se todas as pessoas que adquiriram os ingressos do Festival assistiram (s) afirmar que:
apresentao(es), ento o nmero total de pessoas distintas que assistiu
a pelo menos uma das trs modalidades anteriormente mencionadas foi: (A) (A B) C tem no mximo 2 elementos.
(B) (A B) C tem no mnimo 1 elemento.
(A) 9385. (D) 6275. (C) C B tem 3 elementos.
(B) 9070. (E) 6905. (D) A C tem 2 elementos.
(C) 9595. (E) A B pode ser vazio.

08 (UFF) Considere T o conjunto de todas as pessoas do mundo; M o 14 (AFA) Em um grupo de n cadetes da Aeronutica, 17 nadam, 19 jogam
conjunto de todas aquelas que so muulmanas e A o conjunto de todas basquetebol, 21 jogam voleibol, 5 nadam e jogam basquetebol, 2 nadam e
aquelas que so rabes. Sabendo que nem toda pessoa que muulmana jogam voleibol, 5 jogam basquetebol e voleibol e 2 fazem os trs esportes.
rabe, pode-se representar o conjunto de pessoas do mundo que no Qual o valor de n, sabendo-se que todos os cadetes desse grupo praticam
so muulmanas nem rabes por: pelo menos um desses esportes?

(A) T (A M). (D) (A M) (M A). (A) 31.


(B) T A. (E) M A. (B) 37.
(C) T (A M). (C) 47.
(D) 51.
09 (UFRJ) Um clube oferece a seus associados aulas de trs modalidades
de esporte: natao, tnis e futebol. Nenhum associado pde se inscrever 15 (AFA) Entrevistando 100 oficiais da AFA, descobriu-se que 20 deles pilotam
simultaneamente em tnis e futebol, pois, por problemas administrativos, a aeronave TUCANO, 40 pilotam o helicptero ESQUILO e 50 no so pilotos.
as aulas destes dois esportes sero dadas no mesmo horrio. Encerradas Dos oficiais entrevistados, quantos pilotam o TUCANO e o ESQUILO?
as inscries, verificou-se que: dos 85 inscritos em natao, 50 s faro
natao; o total de inscritos para as aulas de tnis foi de 17 e, para futebol, (A) 5.
de 38; o nmero de inscritos s para as aulas de futebol excede em 10 o (B) 10.
nmero de inscritos s para as de tnis. Quantos associados se inscreveram (C) 15.
simultaneamente para aulas de futebol e natao? (D) 20.

10 (UFRJ) Uma amostra de 100 caixas de plulas anticoncepcionais 16 (EFOMM) Representando graficamente o conjunto CBA C, temos:
fabricadas pela Nascebem S.A. foi enviada para a fiscalizao sanitria.
No teste de qualidade, 60 foram aprovadas e 40 reprovadas, por conterem (A)
plulas de farinha. No teste de quantidade, 74 foram aprovadas e 26 A C
reprovadas, por conterem um nmero menor de plulas que o especificado.
O resultado dos dois testes mostrou que 14 caixas foram reprovadas em
ambos os testes. Quantas caixas foram aprovadas em ambos os testes?

11 (UFRJ) Os 87 alunos do 3o ano do ensino mdio de uma certa escola


prestaram vestibular para trs universidades: A, B e C. Todos os alunos
dessa escola foram aprovados em pelo menos uma das universidades,
mas somente um tero do total obteve aprovao em todas elas. As provas B
da universidade A foram mais difceis e todos os alunos aprovados nesta
foram tambm aprovados em pelo menos uma das outras duas. (B)
A C
Os totais de alunos aprovados nas universidades A e B foram,
respectivamente, 51 e 65. Sabe-se que, dos alunos aprovados em B,
50 foram tambm aprovados em C. Sabe-se tambm que o nmero de
aprovados em A e em B igual ao de aprovados em A e em C.
Quantos alunos foram aprovados em apenas um dos trs vestibulares
prestados? Justifique.

AFA-EFOMM 163
Matemtica I Assunto 1

(C) 19 (AFA Adaptada) Considere um subconjunto A contido em *


A C
e constitudo por y elementos dos quais 13 so mltiplos de 4; 7 so
mltiplos de 10; 5 so mltiplos de 20 e 9 so nmeros mpares. correto
dizer que y um nmero.

(A) par menor que 19.


(B) De 24 a 29.
(C) mpar entre 10 e 20.
(D) primo maior que 21.
B
20 (CN) Considere os conjuntos A = {1, {1}, 2} e B = {1, 2,{2}} e as cinco
(D)
A C afirmaes:

I. A B = {1}
II. {2} B AC
III. {1} A
IV. A B = {1, 2, {1, 2}}
V. B A = {{2}}

Logo,
B
(E) (A) todas as afirmaes esto erradas.
A C (B) se existe apenas uma afirmao correta.
(C) as afirmaes mpares esto corretas.
(D) as afirmaes III e V esto corretas.
(E) as afirmaes I e IV so as nicas incorretas.

EXERCCIOS NVEL 2

01 (CMRJ) Considere o conjunto C = {1, 2, 3}. Para n C, sejam:


B An = {x R |2n 2 < x < 2n} e Bn = {x R|2n 1 < x < 2n + 1}.
Podemos afirmar que:

17 (EFOMM) Foi feita uma pesquisa entre 50 alunos de uma turma sobre suas (A) a interseo da unio dos conjuntos An com a unio dos conjuntos Bn
preferncias em relao a dois professores A e B. O resultado foi o seguinte: o intervalo ]0, 7].
(B) a unio de todos os conjuntos da forma An Bn o intervalo ]1, 6[.
I. Vinte alunos preferiram o professor A. (C) a interseo de todos os conjuntos da forma An Bn vazia.
II. Trinta e cinco alunos preferiram o professor B. (D) a unio da interseo dos conjuntos An com a interseo dos conjuntos
III. Cinco alunos no tiveram preferncia. Bn o intervalo ]2, 4[.
Baseado nesse resultado, pode-se afirmar que o nmero de alunos que (E) a interseo da interseo dos conjuntos An com a interseo dos
preferiu os dois professores foi: conjuntos Bn o intervalo ]1, 7[.

(A) 5. 02 Seja P o conjunto das pessoas em uma festa. Para cada pessoa x
(B) 10. P, vamos definir o subconjunto Ax P como o conjunto dos amigos de x,
(C) 15. isto , Ax = {y P; y amigo de x}. Estamos considerando aqui que, se
(D) 20. x amigo de y, ento y tambm amigo de x e tambm que x Ax (x
(E) 25. amigo de si prprio). Assinale a alternativa incorreta.

18 (EFOMM) Dados os conjuntos: (A) Se x e y tem um amigo em comum, ento Ax Ay .

A = {x R | 2 < x 4}
(B) Se a interseo de todos os subconjuntos Ax no vazia ( A ),
x P
x

B = {x R | 1 x < 3} ento existe algum que amigo de todas as pessoas da festa.


C = {x R | 3 x < 5} (C) Se Ax , ento existe uma pessoa z, tal que Az = P .
x P
B = {x R | x 0}
(D) Se x Ay e y Az ento, necessariamente, x Az .
B B
O resultado de ( A C ) ( D C ) : (E) ( Ax { x }) pode ser diferente de P, pois pode ocorrer que algum
C A x P

no possua amigos na festa (alm de si prprio).


(A) [3,4]
(B) ]2, 1[ [3, 4]
(C) [2, 1] [3,5]
(D) ]2, 4] [5, + ]
(E) ]3, 1]

164 Vol. 1
Conjuntos

03 (ITA) Denotemos por n(X) o nmero de elementos de um conjunto Podemos concluir que o nmero de sentenas verdadeiras :
finito X. Sejam A, B e C conjuntos tais que n(A B) = 8, n(A C) = 9,
n(B C) = 10, n(A B C) = 11 e n(A B C) = 2. (A) 0.
Ento, n(A) + n(B) + n(C) igual a: (B) 1.
(C) 2.
(A) 11. (D) 18. (D) 3.
(B) 14. (E) 25. (E) 4.
(C) 15.
09 (CN) Numa cidade, 28% das pessoas tm cabelos pretos e 24% possuem
04 (UFU) Sejam A, B e C conjuntos de nmeros inteiros, tais que A tem olhos azuis. Sabendo que 65% da populao de cabelos pretos tm olhos
8 elementos, B tem 4 elementos, C tem 7 elementos e A B C tem castanhos e que a populao de olhos verdes que tem cabelos pretos 10%
16 elementos. Ento, o nmero mximo de elementos que o conjunto do total de pessoas de olhos castanhos e cabelos pretos, qual a porcentagem
D = (A B) (B C) pode ter igual a: do total de pessoas de olhos azuis, que tem os cabelos pretos?

(A) 1. (C) 3. Obs.: Nesta cidade s existem pessoas de olhos azuis, verdes ou
(B) 2. (D) 4. castanhos.

05 (UFSJ) Assinale a alternativa que indica quantos so os nmeros (A) 30,25%.


inteiros de 1 a 21.000, que no so divisveis por 2, por 3 e nem por 5. (B) 31,25%.
(C) 32,25%.
(A) 6.300. (D) 33,25%.
(B) 5.600. (E) 34,25%.
(C) 7.000.
(D) 700. 10 Sejam A, B e C subconjuntos de um conjunto universo U. Das afirmaes:

06 (EN) Considere os conjuntos A = {x} e B = {x,{A}} e as proposies: I. Se A B = A C, ento B = C.


II. ((A B) C) ((A BC) C) = A C
I. {A} B; III. (A B C)C = (A C)C (A B)C
II. {x} A;
III. A B; (so) verdadeira(s):
IV. B A
V. {x , A} B (A) apenas II.
(B) apenas III.
As proposies falsas so: (C) apenas I e II.
(D) apenas II e III.
(A) I, III e V. (E) todas.
(B) II, IV e V.
(C) II, III, IV e V. 11 Sejam A, B, C e D subconjuntos de um conjunto universo U. Das afirmaes:
(D) I, III, IV e V.
(E) I, III e IV. I. Se A B e B C, ento A C
II. Se A B e B C , ento A C
III. P ({A [(BC \ CC) D]} [(DC \ A) (C \ B)] unitrio
1
07 (EN) Se Ah o intervalo 0, , h ento A1 A2 A3 A4 ... : IV. P(A B) = {A1 B1 | A1 P(A) e B1 P(B)}
h
o nmero de afirmaes verdadeiras :
(A) {}
(A) 0. (D) 3.
1 (B) 1. (E) 4.
(B) 0,
h (C) 2.
(C) um conjunto unitrio.
12 (EN) Se 70% da populao gostam de samba, 75% de choro, 80%
1
(D) 0, de bolero e 85% de rock, quantos por cento da populao, no mnimo,
h gostam de samba, choro, bolero e rock?
(E) n.r.a
13 Em Porto Alegre foi feita uma pesquisa com a populao sobre suas
08 Seja (A) o conjunto de todos os subconjuntos do conjunto A. Sobre bebidas prediletas e o resultado foi:
as afirmativas: 60% tomam refrigerante (A)
70% tomam vinho (B)
I. A (A); 80% tomam caf (C)
II. Se A B, ento A (B); 90% tomam chimarro (D)
III. Se A (B) e B (A), ento A = B;
Verifica-se ainda que nenhuma pessoa consome as quatro bebidas. Qual
IV. Se A(B) e B(C), ento A C.
a percentagem das pessoas que consomem refrigerante ou vinho?

AFA-EFOMM 165
Matemtica I Assunto 1

RASCUNHO

166 Vol. 1
Lgicas e tcnicas de demonstrao A ssunto
2
Matemtica I

1. Conceitos Ex.:
I. p: Joo irmo de Roberto.
1.1 Proposio
~p: Joo no irmo de Roberto.
toda orao declarativa que exprime uma (proposio simples) II. p: Todos os homens so elegantes.
ou mais (proposio composta) informaes. Uma proposio sempre ~p: Nem todos os homens so elegantes.
verdadeira ou falsa, nunca ambas simultaneamente (Princpio da no
contradio) e tambm no admite uma terceira hiptese (Princpio do No exemplo 1, em termos de conjuntos, sendo A = {x | x irmo de
terceiro excludo). Roberto}, p significa x A. Sua negao x A.

Ex.: A lua um satlite da Terra (verdadeira); Vasco da Gama descobriu 2.2 Disjuno () / Unio ()
o Brasil (falsa)
Chama-se disjuno de duas p q pq
1.2 Proposio funcional proposies p e q a proposio
V V V
representada por p ou q, cujo
valor lgico a verdade (V) quando V F V
p(x) uma proposio funcional num dado conjunto U quando ela
ao menos uma das proposies F V V
assumir valores verdadeiros ou falsos a partir dos elementos de U. O
conjunto dos valores para os quais uma proposio funcional definida verdadeira e a falsidade quando F F F
denomina-se seu conjunto-universo e o conjunto de valores para os quais as proposies so ambas falsas.
a proposio verdadeira denomina-se seu conjunto-verdade ou soluo. Representamos a disjuno de p e q por p q. Podemos resumir as
informaes em uma tabela verdade:
Ex.: p(x): o nmero natural x par.
Ex.: p: Joo irmo de Roberto.
1.3 Conectivos q: Maria no me de Joo.
p q: Joo irmo de Roberto ou Maria no me de Joo.
Chamam-se conectivos palavras usadas para formar novas
proposies a partir de outras. Os conectivos usuais so: e, ou, no, Considerando as proposies: p: x A, q: x B, veja que p q
se ... ento ..., ... se e somente se .... significa x A B
Ex.: O tringulo ABC equiltero se e somente se equingulo. Obs.: Disjuno Exclusiva()
1.4 Tabela verdade O valor lgico da disjuno exclusiva p q pq
a verdade somente quando p
o dispositivo que permite a determinao dos valores lgicos de V V F
verdadeira ou q verdadeira, mas no
uma proposio composta, isto , que possui mais de uma informao,
quando p e q so ambas verdadeiras. V F V
a partir dos valores lgicos das proposies simples que a compem. O
nmero de linhas de uma tabela verdade 2n, em que n o nmero de Ex.: p: Joo viajou de nibus. F V V
proposies simples (cada proposio simples admite 2 valores). F F F
q: Joo viajou de avio.
Obs.: Quando uma proposio composta s admite valores verdadeiros ou
na ltima coluna, dizemos que ela uma tautologia, quando s admite p q: Joo viajou de nibus ou de avio.
valores falsos, dizemos que uma contradio e quando admite ambos
os valores lgicos, dizemos que uma contingncia. 2.3 Conjuno () / Interseo ()
2. Operaes Lgicas Chama-se conjuno de duas p q pq
2.1. Negao (~) / Complementar (A ) C proposies p e q a proposio V V V
representada por p e q, cujo valor V F F
lgico a verdade (V) quando as F V F
Chama-se negao de uma proposio p proposies so ambas verdadeiras
p ~p F F F
a proposio representada por no p, cujo e a falsidade (F) nos demais casos.
valor lgico verdade (V) quando p falsa V F Representamos a conjuno de p e q por p q. Podemos resumir as
(F) e falso (F) quando p verdadeira. Assim, informaes em uma tabela verdade:
F V
no p tem o valor lgico oposto ao de p.
Representamos a negao de p por ~p. Podemos resumir as informaes Ex.: p: Maria comprou bala.
em uma tabela verdade:
q: Maria comprou chiclete.
p q: Maria comprou bala e chiclete.
Considerando as proposies: p: x A, q: x B, veja que p q
significa x A B.

AFA-EFOMM 167
Matemtica I Assunto 2

2.4 Condicional () / Incluso () 2.5 Bicondicional () / Igualdade (=)

Chama-se condicional uma p q pq Chama-se bicondicional uma p q pq


proposio representada por se p ento V V V proposio representada por p se V V V
q, cujo valor lgico a falsidade (F) no V F F e somente se q, cujo valor lgico
caso em que p verdadeira e q falsa a verdade (V) quando ambas so V F F
F V V
e a verdade (V) nos demais casos. verdadeiras ou ambas so falsas e F V F
F F V
Representamos a condicional por p a falsidade (F) nos demais casos. F F V
q. Podemos resumir as informaes em uma tabela verdade: Representamos a bicondicional por
Ex.: p: Roberto ingressar no IME. p q. Podemos resumir as informaes em uma tabela verdade:
q: Roberto ganhar um carro. Ex.: p: ABCD um paralelogramo
p q: Se Roberto ingressar no IME, ento Roberto ganhar um carro. q: As diagonais de ABCD se cortam ao meio.
Obs. 1: Ateno! Um erro comum achar que se Roberto ganhou um
carro, ele passou no IME. No podemos concluir isso a partir da frase: p q: ABCD um paralelogramo se e somente se as suas diagonais
Se Roberto ingressar no IME, ento Roberto ganhar um carro. se cortam ao meio.
Obs. 2: Para demonstrar um teorema do tipo p q, o que se faz
supor que p verdadeira e a partir da concluir que q tambm . Obs.: Um teorema do tipo p q tem duas par tes a serem
demonstradas: p q e q p.

3. Construo de tabelas verdade


Analisaremos as duas proposies a seguir:
3.1 P: ~((~p) (~q))

p q ~p ~q (~p) (~q) ~((~p) (~q))


V V F F F V
V F F V F V
F V V F F V
F F V V V F

3.2 P(p, q, r): (p ((~q) r)) ~(q (p (~r)))

p q r ~q ~q r p (~q r) ~r p~r q(p~r) ~(q (p~r))


V V V F V V F F V F F
V V F F F F V V V F F
V F V V V V F F F V V
V F F V V V V V V F F
F V V F V V F V V F F
F V F F F V V F V F F
F F V V V V F V V F F
F F F V V V V F F V V

Obs.: Podemos usar tabelas verdade para demonstrar igualdades 5. Negao das operaes lgicas:
entre conjuntos (esta uma sada mecnica, que bastante eficiente
principalmente para igualdades envolvendo apenas 2 conjuntos, pois s I. ~(~p) p
temos 4 possibilidades a testar). Veremos como este mtodo pode ser II. ~(p q) (~p) (~q)
empregado nos exerccios resolvidos. III. ~(p q) (~p) (~q)
4. Quantificadores IV. ~(p q) p (~q)
V. ~(p q) (p (~q)) ((~p) q)
4.1 Quantificador universal VI. A negao do quantificador universal o quantificador existencial, isto
(x) (p(x)). L-se: Para todo x, vale a proposio p(x). , (~)
Ex.: (x > 0); (2 x > x) (Para todo x > 0, tem-se 2x > x) VII. A negao do quantificador existencial o quantificador universal, isto
, (~)
4.2 Quantificador existencial
(x) (p(x)). L-se: Existe x, tal que vale a proposio p(x). Ex.:
I. ~p: Nem todos os homens so elegantes.
Ex.: (k ); (8k + 1 primo). (Existe k natural tal que 8k + 1 primo.) Negao: Todos os homens so elegantes.

168 Vol. 1
Lgicas e tcnicas de demonstrao

II. p q : 2 > 5ou Santos a capital de So Paulo. V. p q: tan = 0 se, e somente se, sen = 0.
p: 2 > 5 ; q: Santos a capital de So Paulo. p: tan = 0; q: sen = 0
~p: 2 5 ; ~q: Santos no a capital de So Paulo. ~p: tan 0; q: sen 0
Negao de p q: 2 5 e Santos no a capital de So Paulo. Negao de p q: (tan = 0) e sen 0 ou (tan 0 e sen = 0).

III. p q: Braslia a capital do Brasil e (20 = 0 ou 30 = 1). VI. p: Todo brasileiro magro.
p: Braslia a capital do Brasil; q: 20 = 0 ou 30 = 1. ~p: Existe brasileiro que no magro.
~p: Braslia no a capital do Brasil; ~q: 20 0 e 30 1.
Negao de p q: Braslia no a capital do Brasil ou (20 0 e 30 1). VII. p: (x) (|x| < 0)
~p: (x) (|x| 0)
IV. p q: Se 3 + 2 = 6, ento 4 + 4 = 9.
p: 3 + 2 = 6; q: 4 + 4 = 9.
~q: 4 + 4 9
Negao de p q: 3 + 2 = 6 e 4 + 4 9

EXERCCIOS RESOLVIDOS
01 (OBM) H trs cartas viradas sobre uma mesa. Sabe-se que em 02 Propriedade distributiva Prove que A (B C) = (A B)
cada uma delas est escrito um nmero inteiro positivo. So dadas a (A C).
Carlos, Samuel e Toms as seguintes informaes:
Soluo
I. todos os nmeros escritos nas cartas so diferentes; Sejam p: x A, q: x B e r: x C. Queremos provar que p (q r)
II. a soma dos nmeros 13; (p q) (p r). Como temos 3 proposies, devemos construir
III. os nmeros esto em ordem crescente, da esquerda para a direita. uma tabela verdade com 23 = 8 linhas:

Primeiro, Carlos olha o nmero na carta da esquerda e diz: No tenho p q r q r p q p r p (q r) (p q) (p r)


informaes suficientes para determinar os outros dois nmeros. Em
V V V V V V V V V
seguida, Toms olha o nmero na carta da direita e diz: No tenho
informaes suficientes para determinar os outros dois nmeros. V V F V V F V V V
Por fim, Samuel olha o nmero na carta do meio e diz: No tenho V F V V F V V V V
informaes suficientes para determinar os outros dois nmeros. V F F F F F F F V
Sabendo que cada um deles sabe que os outros dois so inteligentes e F V V V F F F F V
escuta os comentrios dos outros, qual o nmero da carta do meio? F V F V F F F F V
F F V V F F F F V
Soluo
F F F F F F F F V
Sejam x, y e z os nmeros das cartas da esquerda, do meio e da direita,
respectivamente. Temos que x < y < z e x + y + z = 13. Assim,
Como obtemos uma tautologia, a igualdade segue.
x + x + x < x + y + z x 4. Observemos que x 4 (se x = 4,
teramos y = x). Se x = 3, Carlos concluiria que y = 4 e z = 6, portanto,
03 Para n inteiro, prove que se n2 par, ento, n par.
x 3. Assim, x = 1 ou x = 2 e; portanto, y + z 11. Como 2 < y <
z, conclui-se que 6 z 9. Se z = 6, Toms concluiria que y = 5 e
Soluo
x = 2, portanto z 6. Se z = 9, Toms concluiria que x = 1 e y = 3.
Assim, z = 7 ou z = 8. Temos uma implicao p q e, neste caso, mais fcil demonstrar
Neste momento, Samuel poderia achar todas as possveis solues. Se a sua contrapositiva ~q ~p que se n mpar, ento n2 mpar.
x = 1 e z = 7, teramos y = 5; se x = 1 e z = 8, teramos y = 4; se Veja que se n mpar, temos que n = 2k + 1, com k Z. Da,
x = 2 e z = 7, teramos y = 4; se x = 2 e z = 8, teramos y = 3. n2 = (2k + 1)2 = 4k(k + 1) + 1 e podemos concluir que n2 mpar.
Assim, Samuel saberia que os possveis valores de y so 3, 4 e 5. Ora,
se y = 3 ou y = 5, Samuel descobriria os nmeros (se y = 3, Samuel
concluiria que x = 2 e z = 8; se y = 5, Samuel concluiria que x = 1
e z = 7). Logo, o nmero da carta do meio 4.

EXERCCIOS NVEL 1

01 Julgue como (verdadeiros ou falsos) os itens a seguir: 02 Determine a contrapositiva das proposies abaixo:

a. x2 18x + 81 = 49 x =16 a. Se um quadriltero um quadrado, ento ele um retngulo.


b. x2 x 12 0 x 3 ou x 4 b. Se um nmero mpar, ento seu quadrado mpar.
c. x2 x 12 0 x 3 e x 4 c. x1 x2 (x1) (x2) (aqui, (x) representa um objeto qualquer
associado a x).

AFA-EFOMM 169
Matemtica I Assunto 2

03 Um aluno concluiu que 1 = 0 com a seguinte sequncia de argumentos 10 (OBM) Qual o produto da quantidade de vogais pela quantidade de
consoantes na alternativa correta? (No considere as letras A, B, C, D, E
x = 1 x 2 = x x 2 1 = x 1 ( x + 1)( x 1) = x 1 das alternativas na contagem.)
x + 1= 1 x = 0
Determine quais conectivos foram empregados de forma errada pelo aluno. (A) Vinte e quatro.
(B) Trinta e seis.
04 Prove, usando uma tabela verdade, as leis de De Morgan: (A B)C = (C) Quarenta e dois.
AC BC e (A B)C = AC BC. (D) Quarenta e oito.
(E) Cinquenta e seis.
05 (EN) A negativa da proposio (x) (y) (x + y < 2 (x 0
y < 0)) :
11 (OBM) No Planeta Nrdia, existem trs espcies de nerds: ET-nerds,
(A) (x) (y) (x + y 2 (x < 0 y 0)). UFO-nerds e OVNI-nerds. A primeira mente quando chove e diz a verdade
(B) (x) (y) (x + y < 2 (x < 0 y 0)). quando no chove; a segunda sempre mente; a terceira sempre diz a
(C) (x) (y) (x + y < 2 (x < 0 y 0)). verdade. Certo dia, Bruberson, um nerd muito camarada, se encontra com
(D) (x) (y) (x + y 2 (x 0 y 0)). quatro nerds. E eles falam:
(E) (x) (y) (x + y 2 (x < 0 y 0)).
X: "Hoje est chovendo."
Y: "O nerd que acabou de falar est mentindo."
06 (EN) Dada a proposio p (q r) (p q) (p r), podemos
afirmar que : Z: "Hoje no est chovendo."
W: "O primeiro nerd mentiu ou eu sou um ET-nerd."
(A) logicamente falsa. (D) equivalente a (p q) r. Com quantos ET-nerds Bruberson falou no mximo?
(B) uma tautologia. (E) equivalente a ( p q ) r .
(C) equivalente a (p q) r. (A) 0.
(B) 1.
07 (EN) A negao da proposio x 3 e y < 2 :
(C) 2.
(A) x = 3 e y 2. (D) x 2 e y < 3. (D) 3.
(B) x = 3 e y > 2. (E) x 3 ou y < 2. (E) 4.
(C) x = 3 ou y 2.
12 (OBM) Quatro amigos, Arnaldo, Bernaldo, Cernaldo e Dernaldo esto
08 (EN) Considere a proposio: Se x > 5 ento y = 6. A proposio jogando cartas. So 20 cartas diferentes, cada carta tem uma entre 4
equivalente :
cores (azul, amarelo, verde, vermelho) e um nmero de 1 a 5. Cada amigo
(A) Se x < 5 ento y 6. recebe cinco cartas, de modo que todas as cartas so distribudas. Eles
(B) Se y 6 ento x < 5. fazem as seguintes afirmaes:
(C) Se y > 5 ento x = 5. Arnaldo: Eu tenho quatro cartas com o mesmo nmero.
(D) Se y 6 ento x 5. Bernaldo: Eu tenho as cinco cartas vermelhas.
(E) Se x 5 ento y 6. Cernaldo: As minhas cinco cartas so de cores que comeam com
a letra V.
09 (OBM) O programa Quem no quer o bode? ficou muito famoso
Dernaldo: Eu tenho trs cartas de um nmero e duas cartas de outro
nos Estados Unidos. O programa era como a seguir: o participante deve
nmero.
escolher uma dentre trs portas. Atrs de uma das portas, h um carro
e atrs de cada uma das outras duas, h um bode. O convidado ganhar Sabe-se que somente uma das afirmaes falsa. Quem fez essa afirmao?
o que estiver atrs da porta escolhida. Entretanto, os organizadores do
programa perceberam, com o passar do tempo, que aproximadamente (A) Arnaldo.
dois em cada trs participantes ganhavam o carro e, com isso, decidiram (B) Bernaldo.
mudar o programa. Agora, cada uma das trs portas teria um nmeros (C) Cernaldo.
de 1 a 3 e haveria um porteiro identificado com o nmero da porta. Cada (D) Dernaldo.
porteiro faz uma afirmao que pode ser verdade ou mentira. Em seguida, (E) No possvel definir.
o participante escolhe a porta na qual acredita que o carro est. Em um
dos programas, foram ditas as seguintes afirmaes pelos porteiros:
13 (OBM) Sempre que Agilulfo volta para casa depois da escola com uma
Porteiro 1: O carro no est atrs da porta 3.
Porteiro 2: O carro est atrs da minha porta. advertncia, se sua me est em casa, ela o coloca de castigo. Sabendo-se
Porteiro 3: O carro no est atrs da minha porta. que ontem tarde Agilulfo no foi colocado de castigo, qual das seguintes
Sabe-se que pelo menos uma das afirmaes verdade e que pelo menos afirmaes certamente verdadeira?
uma mentira. Atrs de qual porta est o carro?
(A) Agilulfo recebeu advertncia ontem.
(A) Porta 1.
(B) Agilulfo no recebeu advertncia ontem.
(B) Porta 2.
(C) Ontem tarde a sua me estava em casa.
(C) Porta 3.
(D) No possvel identificar. (D) Ontem tarde a sua me no estava em casa.
(E) No possvel que esteja em nenhuma delas. (E) Nenhuma das afirmaes acima certamente verdadeira.

170 Vol. 1
Lgicas e tcnicas de demonstrao

EXERCCIOS NVEL 2
01 (OBM) A figura a seguir foi recortada em cartolina e depois dobrada
para formar um icosaedro. As faces em branco foram numeradas de modo
que ao redor de cada vrtice (pontas do slido) apaream os nmeros de
1 a 5. Qual nmero est na face com a interrogao?

2 ?

3 1 (A) Escolher a primeira coluna esquerda.


(B) Escolher as duas primeiras colunas esquerda.
4 (C) Escolher a terceira linha, de cima para baixo.
(D) Escolher as duas ltimas linhas, de cima para baixo.
(E) Qualquer uma, j que Fbio forosamente ficar com o amendoim.

03 (IME) No produto abaixo, o * substitui algarismos diferentes de 3 e


no necessariamente iguais. Determine o multiplicando e o multiplicador.

* * 3 *
(A) 1. x * * 3
(B) 2. 3 * * *
(C) 3.
* * * 3 3
(D) 4. Icosaedro
(E) 5. * * * *
* * * * * * *
02 (OBM) A figura representa uma barra de chocolate que tem um
amendoim apenas num pedao. Elias e Fbio querem repartir o chocolate, 04 Na Inglaterra um garoto escreve ao pai a seguinte carta:
mas nenhum deles gosta de amendoim. Eles combinam de dividir o SEND
chocolate quebrando-o ao longo das linhas verticais ou horizontais da MORE +
barra, um depois do outro e retirando o pedao escolhido, at que algum
tenha que ficar com o pedao do amendoim. Por sorteio, coube a Elias MONEY
comear a diviso, sendo proibido ficar com mais da metade do chocolate Quanto dinheiro (money) ele pediu ao pai? (substitua cada letra por um
logo no comeo. Qual deve ser a primeira diviso de Elias para garantir algarismo, letras diferentes por algarismos diferentes)
que Fbio fique com o amendoim ao final?

RASCUNHO

AFA-EFOMM 171
Relaes e funes A ssunto
3
Matemtica I

1. Par ordenado A B

1.1 Conceito
Admitiremos o par ordenado (a, b) como conceito primitivo, levando-se 1 1
em considerao que a ordem em que os nmeros aparecem relevante:
R1:
se a b, ento (a, b) (b, a). 2 2

Obs. 1: Kuratowski, um matemtico polons, definiu (a, b): ={{a}, 3


{a, b}}, mas no h necessidade de se preocupar com esta definio.
Obs. 2: A igualdade entre dois pares ordenados (a, b) = (c, d) ocorre
se, e somente se, a = c e b = d.
A B

2. Produto cartesiano 1
1
2.1 Definio 2
R2: 2
Sendo A e B conjuntos, definimos A B = {(a, b)|a A b B}
( o conjunto dos pares ordenados em que a primeira entrada pertence
ao conjunto A e a segunda entrada pertence ao conjunto B). 3

Ex.:
A = {1, 2, 3} 3.2 Domnio
B = {1, 2} O domnio (Dom) de uma relao R de A em B o conjunto formado
pelos elementos de A que, de fato, se relacionam com algum de B. Ou
A B = {(1, 1), (1, 2), (2, 1), (2, 2), (3, 1), (3, 2)}
seja, Dom(R) = {a A|b B com aRb}. Em termos de flechas, o
domnio composto pelos elementos de A que mandam flechas para B.
Teorema 1 (Nmero de elementos do produto cartesiano): Se A e B
so conjuntos finitos, segue que A B finito e n(A B) = n(A) n(B).
Ex.:
R: {1, 2, 3} {1, 2, 3, 4}, R = {(1, 1), (1, 2), (3, 2)}
Obs. 1: A B l-se como A cartesiano B.
Dom(R) = {1, 3}
Obs. 2: Em geral, A B B A (de fato, a igualdade s ocorre
quando A = B). 3.3 Contradomnio
O contradomnio (Cd) de uma relao R de A em B o prprio B ( o
conjunto dos elementos que podem se relacionar com elementos de A).
3. Relao Em termos de flechas, o contradomnio formado pelos elementos que
3.1 Definio podem receber flechas (no caso, todo o conjunto B).
Dados os conjuntos A e B, chama-se relao de A em B qualquer
subconjunto de A B, isto : Ex.: R: {1, 2, 3} {1, 2, 3, 4}
R = {(1, 1), (1, 2), (3, 2)}
R relao de A em B R A B. Cd(R) = {1, 2, 3, 4}

Ex.: A = {1, 2, 3} e B = {1, 2}


3.4 Imagem
R1 = {(1, 1), (2, 1)}
A imagem (Im) de uma relao R de A em B o conjunto formado
R2 = {(1, 1), (2, 1), (3, 1), (3, 2)} pelos elementos de B que de fato se relacionam com algum de A. Ou
seja, Im(R) = {b B|a A com aRb}. Em termos de flechas, a imagem
Obs. 1: (a, b) R pode ser representado por aRb. composta pelos elementos de B que efetivamente recebem flechas.

Obs. 2: Podemos pensar em uma relao como um diagrama de Ex.: R: {1, 2, 3} {1, 2, 3, 4}
flechas, no qual representamos de um lado o conjunto A e do outro o R = {(1, 1), (1, 2), (3, 2)}
conjunto B. Para representar a relao, ligamos a a b com uma flecha se
aRb. Essa intuio ajudar a entender os conceitos futuros. Im(R) = {1, 2}

172 Vol. 1
Relaes e funes

3.5. Relao composta Demonstrao:


Seja R uma relao de A em B e S uma relao de B em C. Definimos
l: y = x
a relao composta T da seguinte forma: T = S R = {(a, c) A C| b
P(a,b)
B com aRb bRc} Intuitivamente, olhamos para a relao R e buscamos b
os elementos da imagem de R que esto no domnio de S (o conjunto B
funciona como uma ponte).
A(a,a)
a Q(b,a)
Ex.: R: {1, 2, 3} {1, 2, 3, 4}, R = {(1, 2), (1, 3), (2, 1)}
S: {1, 2, 3, 4} {3, 4, 5}, S = {(2, 5), (3, 4), (4, 5)}
S R: {1, 2, 3} {3, 4, 5}, SoR = {(1, 5), (1, 4)}
a b
Os elementos da imagem de R que esto no domnio de S so 2 e 3
(esses elementos faro a ponte da relao composta). Suponha P = (a, b) G(R) (grfico de R). Ento, Q = (b, a) G(R1) .
Vejamos um diagrama de flechas ilustrando o que ocorre: Seja A = (a, a).
I. AP = AQ PAQ issceles 
A B C II. 45 = ang < l, AQ > = ang < AP,  >  bissetriz.
R S 
De I e II, l mediatriz de PQ, donde P e Q equidistam de .
1 1 3 Como o ponto P um ponto qualquer de G(R), tem-se G(R) e G(R1)
2 simtricos em relao reta .
2 4
3 5 3.7 Relao em um conjunto
3 3.7.1 Conceito
4
Seja U um conjunto. Chama-se relao em U toda relao de U em U.
A C Ex.:
R: {1, 2, 3, 4} {1, 2, 3, 4}
T
1 3 R = {(1, 2), (2, 3), (2, 4)}

3.7.2 Propriedades
2 4
Seja R uma relao de U em U:

3 5
I. Reflexiva: R reflexiva (x) (x U xRx)

Ex.:
R S T
R = {1, 2, 3} {1, 2, 3}
125 15 R = {(1, 1), (2, 2), (3, 3), (2, 3), (1, 3)}
R S T
134 14
II. Simtrica: R simtrica (x) (y) (x U y U xRy yRx)

Obs.: A operao de composio associativa, mas no comutativa. Ex.:


R = {1, 2, 3} {1, 2, 3}
3.6 Relao inversa R = {(1, 2), (2, 1), (2, 2)}
Seja R uma relao de A em B. Definimos a relao inversa por R = 1

{(y, x) B A|(x, y) R}. Em termos de um diagrama de flechas, basta III. Antissimtrica: R antissimtrica (x)(y)(x U y U xRy
inverter as flechas. yRx x = y)

Ex.:
Ex.:
R = {1, 2, 3} {1, 2, 3}
R: {1, 2, 3} {1, 2, 3, 4}, R = {(1, 1), (1, 2), (3, 2)}
R = {(1, 2), (2, 2), (3, 1)}
R1: {1, 2, 3, 4} {1, 2, 3}, R1 = {(1, 1), (2, 1), (2, 3)}

IV. Transitiva: R transitiva (x)(y)(z)(x U y U z U


Teorema 2 (Simetria do grfico da relao inversa): Se R relao
xRy yRz xRz)
em e R1 sua inversa, ento os grficos dessas relaes no plano
cartesiano so simtricos em relao reta y = x. Ex.:
R = {1, 2, 3} {1, 2, 3}
R = {(1, 2), (2, 3), (1, 3)}

AFA-EFOMM 173
Matemtica I Assunto 3

4. Funes casos, segue que x = y e, portanto, a funo injetora.


Obs. 2: O conceito de funo injetora depende fortemente do domnio.
4.1 Definio Por exemplo, a funo : [1, 1] [1, 1] dada por (x) = x2 no
injetora, j que (1) = (1) = 1. Entretanto, a funo g: [0,1] [0,1]
Sejam A e B conjuntos. Uma funo de A em B (representamos por :
dada pela mesma lei de formao g(x) = x2 injetora. De fato, se g(x)
A B) um tipo especial de relao em que duas condies so satisfeitas:
= g(y), segue que x2 = y2 x = y x = y. Como x, y 0, a ltima
I. Todo elemento de A est relacionado com um elemento de B: (x possibilidade nos d x = y = 0. Em ambos os casos, ento, segue que
A)(y B)((x, y) ). x = y e g injetora. Isto ressalta a importncia de que o domnio e o
II. Todo elemento de A est relacionado com um nico elemento de B contradomnio de uma funo so partes cruciais de sua definio!
(x A)( y, y B)((x, y) (x, y) y = y).
Obs. 3: No caso de domnio e contradomnio finitos, se : A B
Em termos de flechas, todo elemento de A manda uma e apenas injetora, segue que n(A) n(B).
uma flecha para B.
Nesse caso, em vez de escrevermos x y, escrevemos y = (x) sem
ambiguidade.
4.5 Funo sobrejetora (sobrejetiva)

Ex.: Sejam A e B conjuntos. Uma funo : A B sobrejetora se


: {1, 2, 3} {1, 2, 3} y B, x A, tal que (x) = y (ou seja, Im = Cd = B). Em
termos de flechas, todo elemento de B recebe pelo menos uma flecha.
G() = {(1, 2), (2, 2), (3, 1)}
(G(), denominado grfico de , o conjunto dos pares (x, y) tais Obs. 1: No grfico de uma funo sobrejetora, ao traarmos uma reta
que y = (x)). horizontal, esta reta corta o grfico em pelo menos um ponto.
Obs. 1: O domnio de uma funo : A B o conjunto A e o
contradomnio o conjunto B. A imagem definida como em relaes Como verificar se uma funo sobrejetora? Basicamente, o que
(intuitivamente, so os elementos de B que recebem efetivamente flechas). devemos fazer considerar um elemento y do contradomnio de e tentar
resolver a equao (x) = y. Se para cada y essa equao possuir ao
Obs. 2: Em concursos como a AFA, quando nada for dito sobre o menos uma soluo x pertencente ao domnio de , a funo sobrejetora.
domnio, devemos supor que o domnio o mais amplo possvel nos reais,
isto , o domnio o conjunto dos valores para os quais a funo est 3x 5
bem definida (portanto, devemos fazer restries como denominadores Ex.: A funo : {2} {3} dada por f (x ) =
x2
diferentes de 0, expresses dentro de radicais de ndices pares devem ser
no negativas, condies de existncia de logaritmos). sobrejetora. Para verificarmos isso, devemos considerar um real y 3 e
tentar resolver 3 x 5 = y 3 x 5 = xy 2 y x(y 3) = 2y 5 .
4.2 Funo identidade x 2
Seja A um conjunto no vazio. A funo identidade IdA: A A dada 2y 5
Como y 3, segue que x = . Para a demonstrao ficar completa,
por IdA(a) = a para todo a A. y 3
devemos ainda verificar que essa expresso encontrada para x nunca pode
4.3 Funo constante ser 2 (pois o domnio da funo exclui o nmero 2), o que evidente, pois
Sejam A e B conjuntos. Uma funo : A B constante se (a) = b x = 2 2y 5 = 2y 6, contradio.
para todo elemento a A (ou seja, a funo assume um nico valor: todas
as flechas chegam a um mesmo elemento). Obs. 2: O conceito de funo sobrejetora depende fortemente do
contradomnio. Por exemplo, a funo : [1, 1] [1, 1] dada por
(x) = x2 sobrejetora. Entretanto, a funo g: [1, 1] [1, 1] {4}
4.4 Funo injetora (injetiva)
dada pela mesma lei de formao g(x) = x2 no sobrejetora. De fato,
Sejam A e B conjuntos. Uma funo : A B injetora se(x) = (y) se g(x) = 4, segue que x2 = 4 x = 2. Como 2 e 2 no esto no
x = y (ou seja, a funo no repete valor). Em termos de flechas, domnio da funo, no existe x Dom tal que (x) = 4.
cada elemento da imagem recebe uma nica flecha.

Obs. 1: No grfico de uma funo injetora, ao traarmos uma reta Obs. 3: No caso de domnio e contradomnio finitos, se : A B
horizontal, esta reta corta o grfico em no mximo um ponto (pode no sobrejetora, segue que n(A) n(B).
cortar em ponto algum).
4.6 Funo bijetora (bijetiva)
Como verificar que uma funo injetora? Um mtodo prtico e
eficiente supormos (x) = (y) e, atravs de manipulaes algbricas,
Sejam A e B conjuntos. Uma funo : A B bijetora se injetora
chegar a x = y.
e sobrejetora simultaneamente.
Ex.: A funo : dada por (x) = x3 injetora. De fato, (x) Obs. 1: No grfico de uma funo bijetora, ao traarmos uma reta
= (y) x3 = y3 (x y)(x2 + xy + y2) = 0. Logo, x = y ou horizontal, esta reta corta o grfico em exatamente um ponto.
2
y 3y2
x 2 + xy + y 2 x + + = 0 x = y = 0 . Em ambos os Para verificar que uma funo bijetora, basta seguir os passos de 4.4 e
2 4
4.5 para verificao da injetividade e da sobrejetividade.

174 Vol. 1
Relaes e funes

Ex.: A funo : [0, ] [1, 1] dada por (x) = cos x bijetora. Denotamos a inversa de por 1 : B A. Veja que segue da definio
Obs. 2: Assim como em 4.4 e 4.5, o conceito de funo bijetora que 1 = IdB e 1 = IdA.
depende fortemente do domnio e do contradomnio!
Teorema 5 (Mtodo prtico para calcular a inversa):
Obs. 3: No caso de domnio e contradomnio finitos, se : A B Dada a funo bijetora : A B definida pela sentena y = (x), para
bijetora, segue que n(A) = n(B). obtermos a expresso de 1, procedemos como a seguir:
I. Na sentena y = (x), trocamos as variveis x y, escrevendo
4.7 Funo composta
x = (y).

Sejam : A B e g: B C duas funes. Definiremos a composio II. Transforma-se a expresso x = (y), expressando y em funo de x,
da funo g com a funo (h = g l-se g de ou g bola ) chegando a y = 1(x).
da seguinte maneira:
Formalmente, usamos que (1(x)) = x e, a partir disso, encontra-se a
I. O domnio de h o conjunto A. expresso 1(x).

II. O contradomnio de h o conjunto C. Ex.:


I. Determine a inversa da funo (x) = y = 5x + 3.
III. h(x) = g((x)) (aqui funciona exatamente da mesma forma que na Trocando x y, temos x = 5y + 3. Resolvendo em y, segue que
composio de relaes) x 3
y= e, portanto, f 1 (x) = x 3 .
5 5
Ex.: Sendo g(x) = x2 e (x) = 2x + 1, temos que g((x)) = g(2x + 1) II. Determine a inversa da funo : + [1, +] dada por (x) = y
= (2x + 1)2 = 4x2 + 4x + 1. = x2 + 1.
Trocando x y, temos x = y2 + 1. Resolvendo em y, segue que
Obs. 1: Para existir h = g , o contradomnio de deve ser igual f 1 (x) = x 1, x 1.
ao domnio de g.
Obs.: Se o domnio da funo fosse o conjunto dos reais no positivos,
Obs. 2: A composio de funes no comutativa. Alm disso, pode seguiria que a inversa x 1, pois o contradomnio da inversa seria o
acontecer que g esteja definida, mas g no. conjunto dos reais no positivos! Tenha ateno com isso!

Teorema 3 (Critrio para garantir injetividade e sobrejetividade):


4.9 Operaes entre funes
Sejam : A B e g: B C duas funes. Se g : A C injetora,
Sejam A e B conjuntos e : A B, g: A B funes. Definimos as
ento injetora. Se g : A C sobrejetora, ento g sobrejetora.
seguintes operaes:
Demonstrao:
I. ( g)(x) = (x) g(x)
II. ( g)(x) = (x) g(x)
Parte 1: g : A C injetora
Suponha que (x) = (x'). Logo, g (x) = g (x') e, como g f f (x )
III. (x )=
injetora, segue que x = x'. Logo, injetora. g g(x )
IV. g(x) = (x)g(x)
Parte 2: g : A C sobrejetora
Queremos provar que para qualquer z C, existe y B, tal que g(y)
= z. Como g : A C sobrejetora, existe x A, tal que g((x)) = z.
5. Funes reais de varivel real
Finalmente, como (x) B, basta tomarmos y = (x) e assim g sobrejetora. 5.1 Conceito
4.8 Funo inversa So funes de em .
5.2 Paridade
Dada uma funo : A B, queremos definir uma funo g: B A
(a inversa de ) de forma que se (x) = y, ento g(y) = x. Note que para I. Dizemos que : par se (x) = (x) para todo x .
que g seja de fato uma funo, necessrio que: Graficamente, isso significa que a funo simtrica com relao
ao eixo y.
I. cada elemento de B s mande uma flecha de volta (para isso, a funo
no pode repetir valores e, portanto, deve ser injetora); Ex.: (x) = x2, g(x) = cos x

II. todo elemento de B precise mandar flechas (para isso, todo elemento II. Dizemos que : mpar se (x) = (x) para todo x .
de B deve receber flechas da funo e, portanto, deve ser Graficamente, isso significa que a funo simtrica com relao origem.
sobrejetora).
Ex.: u(x) = x3, v(x) = sen x
Teorema 4 (Condio de existncia da inversa):
Uma funo : A B admite inversa se, e somente se, bijetora.

AFA-EFOMM 175
Matemtica I Assunto 3

5.3 Monotonismo
3 o Passo: Finalmente desenhamos a funo desejada
Funo estritamente crescente
y = 1 + cos x , deslocando a anterior uma unidade para cima.
Dizemos que : estritamente crescente se x < y 3
(x) < (y) . 2.0
Ex.: (x) = 2x + 1
1.5
Funo estritamente decrescente
Dizemos que : estritamente decrescente se x < y (x)
> (y) . 1.0

Ex.: (x) = 2x + 1 0.5

5.4 Periodicidade
Uma funo : dita peridica quando existe T > 0 (dito 6 4 2 2 4 6
um perodo de ) tal que (x + T) = (x) para todo x real. O menor real
6.2 Esticando e contraindo uma funo
positivo T com essa propriedade chamado de perodo de (s vezes
chamado de perodo fundamental).
k > 1 esticaa funoem y
III. k f ( x )
Ex.: 0 < k < 1 contraia funoem y
A funo (x) = sen x peridica de perodo 2, pois (x + 2) =
(x). k > 1contrai 
o grfico em x
IV. f ( kx )
0 < k < 1estica 
a grfico em x
6. Grficos
Ex.: Monte o grfico da funo y = 3sen(2x)
6.1 Deslocando o grfico de uma funo
1o Passo: Montamos o grfico da funo y = sen x.
I. f ( x ) + k  k > 0 desloca 
a funok unidadespracima
1.0

k < 0 
desloca a funok unidadesprabaixo
0.5
k > 0 desloca 
o grfico paraesquerda
II. f ( x + k )
k < 0 desloca 
o grfico paradireita
2 2
Ex.:
0.5
Vejamos o grfico da funo y = 1 + cos x
3
1.0
1o Passo: Primeiro devemos desenhar o grfico da funo y = cos x.
2o Passo: Desenhamos o grfico da funo y = sen (2x), lembrando
1.0 que se estamos multiplicando x por dois, estamos dividindo o perodo
da funo por 2.
0.5 1.0

0.5
2
0.5


2 2
1.0 0.5


2o Passo: Depois desenhamos o grfico da funo y = cos x , 1.0
3

deslocando a funo anterior para a direita. 3o Passo: Finalmente desenhamos o grfico da funo y = 3sen(2x),
3
lembrando que estamos esticando a funo no eixo y.
1.0 3
2
0.5
1

6 4 2 2 4 6
2 1
2 2
0.5
2
1.0 3

176 Vol. 1
Relaes e funes

EXERCCIOS RESOLVIDOS
01 Considere uma turma de 30 alunos (Ablio e Deuclcio so dois 2
x 1 x 1
deles). Seja A o conjunto dos alunos dessa turma e : A N a funo y, podemos dizer que h ( x ) = .
que associa cada aluno sua quantidade de amigos dentro da turma. 2 2
Considere que a relao de amizade recproca (ou seja, se X amigo de
Y, ento Y amigo de X) e que ningum amigo de si mesmo. possvel Obs.: Veja que o fato de a relao valer para todo y, nos permite substituir
que tenhamos simultaneamente (Ablio) = 0 e (Declcio) = 29? o y por qualquer coisa, inclusive por x.

Soluo 04 Sejam e g funes reais tais que (x) = 4x 1 e g(x) =


Se (Deuclcio) = 29, teramos que Deuclcio amigo de todos da 3x2 + 7x + 1. Determine a lei de formao da funo g.
turma (pois ele no amigo de si mesmo). Da, veja que Ablio deveria
ter pelo menos um amigo (Deuclcio) e ento no possvel termos Soluo
(Ablio) = 0. De (x) = 4x 1, tiramos que (g(x)) = 4g(x) 1, portanto, 4g(x) 1
= 3x2 + 7x + 1.
03 Sabendo que h uma funo tal que h(2x + 1) = x2 x, para todo 3x2 + 7x + 2
x real, determine a lei de formao de h. Ento, a funo g tem g ( x ) = como lei de formao.
4

1a Soluo: Como h(2x + 1) = x2 x para todo x real, podemos


3x + 1
substituir x por qualquer nmero ou expresso. Ento, conveniente 05 Seja : {2} {b} uma funo tal que f ( x ) = .
x x2 x x 2
trocar x chegando a h ( x + 1) = . Agora, basta trocar x
2 4 2 Determine o valor de b para que a funo seja bijetora e determine
( x 1) ( x 1) . sua inversa.
2

x 1, chegando a h ( x ) =
4 2
Soluo:
Obs.: claro que poderamos fazer uma nica substituio Antes de tudo, vamos determinar a lei de formao da inversa de . Para a
x 1 3y + 1 2x + 1
x . Apresentamos a soluo em 2 passos para ficar um inversa, temos que x = e, isolando o y, temos que y = (*).
2 y 2 x 3
pouco mais natural.

y 1 Veja, ento, que 3 no pertence ao domnio da inversa; portanto, precisamos


2a soluo: Fazendo 2x + 1 = y, temos que x = . Substituindo
2 excluir 3 do contradomnio de (para que seja sobrejetora). Ento, b = 3.
fcil ver que a funo dada injetora, pois na inversa, cada y est definido
2
y 1 y 1 unicamente a partir de um x, pela equao (*).
na expresso dada, temos que h ( y ) = , para todo
2 2
1 2x + 1
Ento, f (x ) = e b = 3.
y real. Veja que isso define h, pois, j que esta relao vale para todo x 3

EXERCCIOS NVEL 1

01 (OBM) Seja uma funo definida para todo x real, satisfazendo as Nessas condies, a imagem do nmero 3 igual a:
condies:
(A) 1. (D) 4.
f (3) = 2 (B) 2. (E) 5.
(C) 3.
f (x + 3) = f (x ) . f (3)
f (x )
03 (AFA) As funes f e g so dadas por (x) = ax + bx e g(x ) = .
Ento, (3) vale: f (x 2)
Ento, g(3) igual a:
(A) 6. (D) 2.
(A) a2 + b2.
(B) 0. (E) 1.
(B) (a + b)2.
(C) 1/2.
(C) (a b)2.
(D) a2 ab + b2.
02 Seja S = {1, 2, 3, 4, 5} e considere uma funo bijetora de S em S,
tal que:
04 (AFA) A funo f satisfaz a relao: (x + 1) = x (x), x > 0. Se
1 3
I. Se x S, a imagem de x no pode ser igual a x 1, nem igual a x, f = , o valor de ff :
nem igual a x + 1. 2 2
II. Se x S e a imagem de x y, ento a imagem de y no pode ser nem 3
x, nem x + 1. (A) . (C) .
2 2
(B) 2 . (D) .

AFA-EFOMM 177
Matemtica I Assunto 3

n 11 (AFA) Considere as funes f, g e h, todas de domnio [a, b] e


2 se n par contradomnio [c, d], representadas atravs dos grficos abaixo.
05 (AFA) A funo : definida por (x) = :
n + 1 se n mpar f(x) g(x)
2
(A) bijetora. d d
(B) somente injetora. e
(C) somente sobrejetora. c c
(D) no injetora e no sobrejetora.
x x
x 1 0 a b 0 a b
06 (AFA) Se for uma funo real, tal que f = x + 3, ento (x)
x + 1 h(x)
definida por:
d
4 2x 2x + 1
(A) . (C) . e
1 x x 1
c
4x 2 2x 1 .
(B) . (D) x
1+ x 1 x 0 a b

Com base nos grficos, correto afirmar que:


07 (AFA) Seja D = {1, 2, 3, 4, 5} e f: D a funo definida por
f(x) = (x 2)(x 4). Ento, pode-se afirmar que f: (A) f uma sobrejeo, g no uma injeo, h uma sobrejeo.
(B) f uma sobrejeo, g uma injeo, h no uma sobrejeo.
(A) bijetora. (C) f uma injeo, g no uma sobrejeo, h uma bijeo.
(B) somente injetora. (D) f uma bijeo, g no uma injeo, h no uma sobrejeo.
(C) somente sobrejetora.
(D) possui conjunto-imagem com 3 elementos. 12 (EN) dada uma funo tal que:

2+ x I. f(x) f(y) = f(x + y)


08 (AFA) A imagem da funo real definida por f (x ) = :
2 x II. f(1) = 2 e f( 2 ) = 4
(A) {1}. (C) {1}.
Podemos concluir, ento, que f(3 + 2) igual a:
(B) {2}. (D) {2}.
(A) (3 + 2 )2.
09 (EsPCEx) A funo , de domnio real mais amplo possvel, tal
(B) 16.
que f (x ) = ax + b 5 . Sabendo que (3) no existe e (1) = 1, o (C) 24.
ax + 3 b (D) 32.
valor de a2 + b2 : (E) 64.
(A) 50/16. (D) 50/8.
13 (EN) Determine o conjunto-imagem da funo (f g) para:
(B) 25/3. (E) 50/9.
(C) 25/2.
1, se x < 0
0, se x < 0 x
10 (AFA) Seja f a funo real cujo grfico se apresenta a seguir:
f (x) = 2x, se 0 x 1 e g(x) = , se 0 x 1
0, se x > 1 2
1, se x > 1

(A) |0, 1| {2}.


(B) (, +).
(C) |0, 1|.
(D) |0, +).
(E) {1}.

14 (EsPCEx) Seja : uma funo tal que 2 (x) < 5 e g:


dada por g(x) = 1 (x). Ento o conjunto-imagem da funo g(x) :
Analisando o grfico, incorreto afirmar que:
(A) ]4, 3].
(A) f(f(1)) = f(0,5). (B) [4, 3].
(B) f(x) + 1 > 0, x . (C) ]4,3[.
(C) f(0) f(x), x . (D) [3, 4[.
5 (E) ]3, 4].
(D) se g(x) = f(x) 1, ento g(2) = f .
2

178 Vol. 1
Relaes e funes

15 (ITA) Sejam f, g: funes tais que: g(x) = 1 x e (x) + 1


2 (2 x) = (x 1)3 para todo x . Ento [g(x)] igual a: 23 (AFA) Sejam as funes reais definidas por f(x) = x2 1 e g(x ) = .
Ento, f(g(1)) igual a: x

(A) (x 1)3.
(A) 1.
(B) (1 x)3.
(B) 0.
(C) x3.
(C) 1.
(D) x.
(D) 2.
(E) 2 x.
24 (AFA) Sejam A = {0,1,2,3} e f: A A uma funo definida por
16 (ITA) Qual das funes definidas abaixo bijetora?
f(0)= 2, f(1) = 1, f(2) = 3 e f(3) = 0. Calculando f f f f f(1),
encontra-se:
Obs.: + o conjunto dos reais no negativos.
(A) 0.
(A) : + tal que (x) = x2. (B) 1.
(B) : + tal que (x) = x +1. (C) 2.
(C) : [1, 3] [2, 4] tal que (x) = x+1. (D) 3.
(D) : [0, 2] tal que (x) = sen x.
(E) n.d.a. 3x 2
25 (AFA) Se f e g so funes de em definidas por f (3 x + 2) =
e g(x 3) = 5x 2, ento (g(x)) : 2
17 (EN) Seja uma funo e x um ponto do seu domnio. Diz-se que
um ponto fixo de se (x) = x. Considere a funo g: definida
x x 4
f (2x + 1) = (A) .
por x 2 + 1. correto afirmar que: 5
2x + 9
(B) .
(A) g no possui ponto fixo em [0, 1]. 5
(B) g possui um ponto fixo em [0, 1].
(C) 5x + 13.
(C) g possui dois pontos fixos em [0, 1].
(D) g possui trs pontos fixos em [0, 1]. 5 x + 11
(E) g possui quatro pontos fixos em [0, 1]. (D) .
5
18 (OBM) A funo dada pela tabela a seguir. 1/ x , se x *
26 (EsPCEx) Sendo : definida por f (x) = e
2,se x - *
1, se x Q
x 1 2 3 4 5 g: definida por g(x) = , ento ( g g)
1/ 2, se x Q
(x) 4 1 3 5 2 (2 + 2 ) igual a:

f ( f (...( f ( f (4))...))? (A) 1.


Por exemplo, (2) = 1. Quanto vale    (B) 1/2.
2004 vezes
(C) 2.
(A) 1. 2
(B) 2. (D) 1 .
(C) 3. 2
(E) 2.
(D) 4.
(E) 5.
27 (AFA) Observe os grficos abaixo, das funes f e g, definidas no
intervalo [0,1].
19 Sejam as funes reais de varivel real e g, definidas por
5 x 3 e g(x) = 3 . Pede-se:
f (x) =
4x + 1 x
a. obter as leis que definem g e g;
b. calcular g (2) e g(2).

20 Dada a funo real de varivel real (x) = ax2 + bx + c, pede-se:

a. obter (x + 1);
b. obter (x);
c. determinar a, b e c de modo que se tenha (x + 1) = (x).
Com base nos grficos, assinale a alternativa falsa:
x
21 Se f (2x + 1) = , determine (x 1).
x2 + 1 (A) g(f(0,4)) g(f(x)), x [0,1]
22 Se (x) = 4x + 1 e (g(x)) = x2 + 1, determine a funo g(x). (B) g(f(0,05)) > g(f(0,1))
(C) g(g(x)) = x, x [0,3; 0,8]
(D) g(f(0,6)) > g(f(1))

AFA-EFOMM 179
Matemtica I Assunto 3

28 Determine a funo inversa de (x) = x5 + 1. No grfico, tem-se o nvel da gua armazenada em uma barragem ao longo
dos ltimos anos, que foi construda para represar gua a fim de mover
3x 1 as turbinas de uma usina hidreltrica.
29 Sendo f (x) = 3 2 x + 3 1 e g(x) = , ache 1, g1 e g g.
2x + 5
nvel(m) o nvel mximo
30 Seja a funo : [2, ) I, (x) = x2 x + 1, determine qual deve 120
ser o intervalo I para que admita uma funo inversa. 80

x 1
31 (AFA) Determine a funo inversa de f (x)= .
x
30
1
(A) . o nvel mnimo para gerar energia
1 x 10
1
(B) .
1+ x 1989 1995 2000 temp
1 x
(C) .
1+ x Analise as alternativas e marque a opo correta:
1+ x
(D) .
1 x (A) O nvel da gua permaneceu constante num perodo de 8 anos.
(B) O nvel de 80 metros foi atingido exatamente duas vezes at o ano
32 (AMAN) Sejam e g funes de A em A com grficos f* = {(1, 2), 2000.
(2, 1), (3, 5), (4, 4), (5, 2)} e g* = {(1, 1), (2, 3), (3, 5), (4, 3), (5, 1)}. (C) Aps o ano de 2000, o nvel da gua da barragem foi insuficiente para
Logo, 1(4) g1(5) vale: gerar energia.
(D) No perodo de 1995 a 2000, o nvel da gua s diminuiu.
(A) 0. (D) 6.
(B) 2. (E) 12. 4 x 6 x 1 se
2
x1
37 (AFA) Considere as funes reais ( f g )( x ) =
(C) 25. 4 x + 3 se x <1
e g(x) = 2x 3. Com base nessas funes, classifique cada afirmativa
33 ( ITA) Seja a funo f: {2} {3} definida por abaixo como verdadeira ou falsa.
2x 3 . Sobre sua inversa podemos garantir que:
f (x) = +1
x 2 I. (x) par;
II. (x) admite inversa em todo o seu domnio;
(A) no est definida pois f no injetora.
III. (x) crescente em {x / x < 1 ou x 1};
(B) no est definida pois f no sobrejetora. IV. se x < 6, ento (x) > 3.
y2
(C) est definida por f 1 (y ) = , y 3.
y3 A sequncia correta :
y+5
(D) est definida por f 1 (y) = 1 , y 3. (A) V V F V. (C) F F V V.
y 3
2y 5 (B) F F V F. (D) F V V F.
1
(E) est definida por f (y ) = , y 3.
y3
38 (EN) Sabendo que f, g e h so funes reais de varivel real e que f e
34 Se (x) peridica de perodo T, determine o perodo de g(x) = (ax g no se anulam, considere as afirmaes abaixo:
+ b), sendo a 0.
I. (g + h) = g + h;
35 (AFA) Indique a alternativa correta:
II. (g + h) = g + h ;
(A) Se uma funo par, ento bijetora.
(B) Se (x) ( x) = 0, ento pode ser relao, mas no funo. 1 1
(C) Se uma funo par e x *, ento * par s quando x for primo. III. = g;
f g f
(D) Se : uma funo real qualquer, ento pode ser escrita
como soma de duas funes reais g: e h: , em que g
1 1
par e h impar. IV. = f
f g g
36 (AFA)
O Brasil tem um encontro marcado com o caos. No dia 1o de junho Podemos afirmar que:
comea o plano de racionamento de energia.
(A) todas as afirmativas acima so verdadeiras.
O modelo energtico brasileiro baseado quase que exclusivamente (B) somente I e II so verdadeiras.
em hidreltricas, que produzem 97% da energia consumida no Pas. Sem (C) somente IV falsa.
chuva, entra em colapso. (D) somente II e III so verdadeiras.
(Revista Veja 16/5/2001.) (E) somente I falsa.

180 Vol. 1
Relaes e funes

39 (ITA) Consideremos as seguintes afirmaes sobre uma funo f: . 04 Determine o valor da expresso
1 2 3 1998 1999 2000
I. Se existe x tal que (x) ( x), ento f no par. f
2000 + f 1999 + f 1998 + ... + f 3 + f 2 + f 1 ,
II. Se existe x tal que f( x) = f(x), ento f mpar.
III. Se f par e mpar, ento existe x tal que f(x) = 1. x2
IV. Se f mpar, ento f f (f composta com f) mpar. em que f ( x ) = .
1+ x 2

Podemos afirmar que esto corretas as afirmaes de nmeros: 05 (OBM) Seja uma funo real de varivel real que satisfaz a condio

2002
(A) I e IV. (D) III e IV. f ( x ) + 2f = 3 x para x > 0. O valor de (2) igual a:
(B) I, II e IV. (E) I, II e III. x
(C) I e III. (A) 1.000. (D) 4.000.
(B) 2.000. (E) 6.000.
40 (ITA) Considere a funo y = f(x) definida por f(x) = x3 2x2 + 5x, para (C) 3.000.
cada x real. Sobre esta funo, qual das afirmaes abaixo verdadeira?
06 (OBM) A funo real , definida nos inteiros, satisfaz
(A) y = f(x) uma funo par. (n) (n + 1) (2 n) = (n + 3)2, para todo n inteiro . Quanto vale (0)?
(B) y = f(x) uma funo mpar.
(C) (x) 0 para todo real x. (A) 17. (D) 2.
(D) (x) 0 para todo real x. (B) 0. (E) 9.
(E) f(x) tem o mesmo sinal de x, para todo real x 0. (C) 1.

41 Esboce no plano cartesiano os grficos das seguintes funes: 07 (OBM) Seja : Z Z uma funo tal que (0) = 0, (1) = 1,
(2) = 2 e (x + 12) = (x + 21) = (x) para todo x Z. Ento (2009) :
a. : { 1, 1}
x y = signx, funo sinal, signx = {1 se x 0 e 1 se x < 0} (A) 0. (D) 3.
b. : (B) 1. (E) 2009.
x y = x = (n / n x< n + 1), funo parte inteira. (C) 2.
c. : [0,1[
x {x} = x x, funo parte fracionria. n
08 (OBM) Para todo n natural definimos a funo por: f ( n ) =
se n par, (n) = 3n + 1 se n mpar. 2

EXERCCIOS NVEL 2 O nmero de solues da equao (((n))) = 16 :

01 Determine o conjunto-imagem das funes abaixo: (A) 2. (D) 5.


(B) 3. (E) 6.
| x| (C) 4.
a. : {0} , x y =
x
09 (OBM) Seja uma funo real que tem as seguintes propriedades:
b. : [4, +[ , x y = x + x 4
1 I. Para todos x, y reais, (x + y) = x + (y);
c. : , x y = II. (0) = 2.
x2 + 1
x
d. : , x y = Quanto vale (2000)?
x2 + 1
(A) 0. (D) 2000.
02 (IME) Sejam q e r funes cujo domnio so os inteiros maiores (B) 2. (E) 2002.
que zero. Sabe-se que q(1) = 1, r(1) = 0 e: (C) 1998.

r( n + 1) = r( n) + 1 10 (OBM) A funo definida para todos os pares ordenados (x; y) de


se r(n) < 2q(n) + 1, ento
q( n + 1) = q( n) inteiros positivos e tem as seguintes propriedades:
r( n + 1) = 0
se r(n) = 2q(n) + 1, ento (x; x) = x; (x; y) = (y; x); (x + y)(x; y) = (2x + y)(x; x + y). Qual
q( n + 1) = q( n) + 1 o valor de (21; 12)?
Determine q(5) e r(5). 7 6
(A) . (D) .
4 11
03 Classifique a funo : N N N, (m, n) = 2m 3n quanto a
4 1
injetividade e sobrejetividade. (B) . (E) .
7 2003

(C) 11.
6

AFA-EFOMM 181
Matemtica I Assunto 3

11 (OBM) Seja uma funo de Z em Z definida como 16 (ITA) Sejam A e B subconjuntos no vazios dos nmeros reais e
(x) = x/10 se x divisvel por 10 e (x) = x + 1 caso contrrio. : A B, g : B A duas funes tais que f g = Id B. Ento podemos
Se a0 = 2001 e an+1 = f(an), qual o menor valor de n para o qual an = 1? afirmar que:

(A) 20. (D) 2000. (A) sobrejetora. (D) g injetora e par.


(B) 38. (E) an nunca igual a 1. (B) injetora. (E) g bijetora e mpar.
(C) 93. (C) bijetora.

12 (EN) O conjunto-imagem da funo f ( x ) = x 2 4 + 4 x 2 : 17 ( I TA ) S e j a : u m a f u n o s a t i s f a z e n d o


(x + y) = (x) + (y) para todos a, x, y reais. Se a1, a2, a3,..., an uma
(A) {x / x > 0}. progresso de razo d, ento podemos dizer que (a1), (a2), (a3),..., (an)
(B) {x / 2 < x < 2}.
(C) {0}. (A) uma progresso aritmtica de razo d.
(D) {x / x 2 ou x 2}. (B) uma progresso aritmtica de razo f(d) cujo primeiro termo a1.
(E) +. (C) uma progresso geomtrica de razo f(d).
(D) uma progresso aritmtica de razo f(d).
13 (EN) Considere a funo real definida por: (E) nada se pode afirmar.

x 2 1 se x < 2
x 3
18 (EN) Seja x {1, 0, 1}. Se f1( x ) = e n+1(x)= 1(n(x)) para
todo n natural, ento f1988(x) igual a: x +1
3 se 2 x < 1
x2 1
3 x
3 se 1 < x < 1 (A) x 3 . (D) .
x2 1 x +1 x +1
f (x) =
3 (B) x. (E) x + 3 .
se 1 < x < 2
x2 1 x 1
x 3 se x 2 (C) x + 3 .

2 se x = 1 1 x

2 se x = 1 19 (EN) Determine o conjunto-imagem da funo (f g) para:
1 se x < 0
A imagem da funo o conjunto: 0 se x < 0
x
f ( x ) = 2x se 0 x 1 e g( x ) = se 0 x 1
(A) ] , 3] U [1, +[. 0 se x > 1 2
1 se x > 1
(B) ], 1[ U [2, +[.
(C) ], 3[ U ]1, 1[ U ]1,+[. (A) |0,1| {2}. (D) |0, +).
(D) ], 2[ U ]2, 1[ U ]1, +[. (B) (, +). (E) {1}.
(E) R {1, 1} (C) |0, 1|.

14 ( I TA ) S e j a m t r s f u n e s f, u , v : t a i s q u e f 20 (ITA) Sejam f, g : funes tais que:


1 1
x + = f( x ) + para todo x no nulo e (u(x))2 + (v(x))2 = 1 g(x) = 1 x e (x) + 2 (2 x ) = (x 1)3 para todo x .
x f ( x)
para todo x real. Sabendo-se que x0 um nmero real tal que u(x0) v(x0) Ento [g(x)] igual a:
1 1 u (x 0 )
0 e f . = 2, o valor de f : (A) (x 1)3. (D) x.
u (x ) v (x )
0 0 v (x 0 ) (B) (1 x)3. (E) 2 x.
(C) x3.
(A) 1. (D) 1 .
2 21 (ITA) Considere x = g(y) a funo inversa da seguinte funo:
(B) 1. (E) 2. 1
y = f(x) = x2 x + 1, para cada nmero real x . Nestas condies,
(C) 2. 2
a funo g assim definida:
15 (ITA) Sejam o conjunto dos nmeros reais e f uma funo de
1 3 3
1
{ }
em . Se B e o conjunto f ( B ) = x ; f ( x ) B , ento: (A) g( y ) =
2
+ y , para cada y .
4 4

( ) B
(A) f f 1 ( B ) (B) g( y ) =
1
2
1 1
+ y , para cada y .
4 4
( ) = B, se injetora.
(B) f f 1 ( B )
( ) = B,
(C) f f 1 ( B ) 3 3
(C) g( y ) = y , para cada y .
( ) = B, se f injetora.
(D) f 1 f ( B ) 4 4
(E) n.d.a

182 Vol. 1
Relaes e funes

1 26 (ITA) Dadas as funes f: e g: , ambas estritamente


1
(D) g( y ) = y , para cada y . decrescentes e sobrejetoras, considere h = fg. Ento podemos afirmar que:
4 4
3 1 1 (A) h estritamente crescente, inversvel e sua inversa estritamente
(E) g( y ) = + y , para cada y .
crescente.
4 2 2
(B) h estritamente decrescente, inversvel e sua inversa estritamente
crescente.
22 (ITA) Seja f uma funo real definida para todo x real tal que: f mpar;
(C) h estritamente crescente, mas no necessariamente inversvel.
(D) h estritamente crescente, inversvel e sua inversa estritamente
f ( x ) f (1)
f(x y) = f(x) f(y); e f(x) 0, se x 0. Definindo g( x ) = , decrescente.
x (E) n.d.a.
se x 0, e sendo n um nmero inteiro positivo, podemos afirmar que:

(A) f no decrescente e g uma funo mpar. 27 (ITA) Considere uma funo f: no constante e tal que
(B) f no decrescente e g uma funo par. f(x + y) = f(x)f(y), x, y .
(C) g uma funo par e 0 g(n) f(1).
Das afirmaes:
(D) g uma funo mpar e 0 g(n) f(1).
(E) f no decrescente e 0 g(n) f(1) I. f(x) > 0, x .
II. f(nx) = [f(x)]n, x , n N*.
23 (EsPCEx) Seja a funo : { 1, 1} , definida por III. f par.
x3
f ( x ) = 2 . Podemos afirmar que essa funo : (so) verdadeira(s):
x 1
(A) bijetora e no par nem mpar. (A) apenas I e II. (D) todas.
(B) par e injetora. (B) apenas II e III. (E) nenhuma.
(C) mpar e injetora. (C) apenas I e III.
(D) par e sobrejetora.
(E) mpar e sobrejetora. 28 Seja (x) uma funo real, definida em e satisfazendo a equao

24 Seja f: uma funo estritamente decrescente, isto , quaisquer x 1


x e y reais com x < y tem-se f(x) > f(y). Dadas as afirmaes: funcional f ( x ) + f = 1 + x . A expresso de (x) :
x
I. f injetora;
II. f pode ser uma funo par; x3 x2 1
(A) .
III. se f possui inversa ento sua inversa tambm estritamente 2 x ( x 1)
decrescente. x3 + x2 + 1
(B) .
Podemos assegurar que: x ( x 1)
x3 x2 + 1
(C) .
(A) Apenas as afirmaes I e III so verdadeiras. x ( x 1)
(B) Apenas as afirmaes II e III so falsas.
x3 + x2 1
(C) Apenas as afirmaes I falsa. (D) .
(D) Todas as afirmaes so verdadeiras x ( x 1)
(E) Apenas a afirmao II verdadeira. x3 + x2 1
(E) .
x ( x + 1)
25 (ITA) Considere as afirmaes:
1+ x
I. Se f: uma funo par e g: uma funo qualquer, 29 A funo real definida por f ( x ) = pode ser decomposta, de
1 x
ento a composio gf uma funo par.
II. Se f: uma funo par e g: uma funo mpar, ento maneira nica, como uma soma da forma P( x ) + I( x ), onde P(x) uma
a composio fg uma funo par. funo par e I(x) uma funo mpar. A expresso de I(x) :
III. Se f: uma funo mpar e inversvel, ento f1: uma
funo mpar. x
(A)
1 x 2
Ento: 2x
(B)
1 x 2
(A) Apenas a afirmao I falsa.
(B) Apenas as afirmaes I e II so falsas. 3x
(C)
(C) Apenas a afirmao III verdadeira. 1 x 2
(D) Todas as afirmaes so falsas. 4x
(E) n.d.a. (D)
1 x 2
5x
(E)
1 x 2

AFA-EFOMM 183
Matemtica I Assunto 3

30 (OBM) Se : uma funo tal que, para todo x real, 31 A funo tal que, para cada nmero real x, vale a relao
(x) ((x) x) = 0, ento: (x) + (x 1) = x2. Se (19) = 94, ento (94) vale:

(A) a funo nula. (A) 3227. (D) 4245.


(B) a funo identidade, ou seja, (x) = x para todo x real. (B) 3572. (E) 4561.
(C) a funo nula ou a funo identidade. (C) 3763.
(D) H 4 possveis funes .
(E) H infinitas funes .

RASCUNHO

184 Vol. 1
lgebra bsica A ssunto
1
Matemtica II

Neste material, estudaremos os princpios bsicos da lgebra. No Teorema 2 (Multiplicao por zero):
decorrer das seguintes semanas, ficaro claras sua importncia e suas
aplicaes em todas os outros assuntos da matemtica. Podemos dizer que Para todo x real, tem-se que x 0 = 0.
o aluno que no alcanar um domnio mnimo neste assunto certamente
ter dificuldades com as outras reas.
Demonstrao: A ideia utilizar que x (0 + 0) = x 0. Fazendo
distributiva do lado esquerdo, temos x 0 + x 0 = x 0 + 0. Pela lei do
1. Axiomas e conceitos corte, segue que x 0 = 0.
bsicos dos nmeros reais ()
Comentrio: Por isso no se define a diviso por 0. Se por um
1
No conjunto dos reais, so definidas duas operaes: a soma e o momento aceitssemos = x , teramos que 1 = 0 x, o que no
produto. Essas operaes possuem propriedades bsicas que no podem possvel. 0
ser demonstradas e, por isso, as chamamos de axiomas.
Teorema 3 (Produto igual a zero)
I. Comutativa
x+y=y+x
xy=yx Se x e y so reais tais que x y = 0, ento x = 0 ou y = 0.

II. Associativa Demonstrao: Pelo teorema 2, fcil ver que se x = 0, a equao


(x + y) + z = x + (y + z) x y = 0 satisfeita. Caso x 0, sabemos que existe o seu inverso x1.
(x y) z = x (y z) Multiplicando os dois lados de x y = 0 por x1, temos x1 (x y) = x1 0.
Utilizando a associativa do lado esquerdo e o teorema 2 do lado direito, temos que
III. Elemento Neutro y = 0. Portanto, ou x = 0 ou y = 0 .
x + 0 = x
x1=x Essa uma das principais propriedades da lgebra e uma das
grandes motivaes para se aprender a fatorar. Em algum problema em
IV. Elemento Simtrico / Inverso que uma expresso igual a zero, se conseguirmos fatorar essa expresso,
x + (x) = 0 (simtrico) podemos transformar o problema em dois geralmente mais simples.
x x1 = 1, para x 0 (inverso)

V. Distributiva Teorema 4 (Regra dos sinais):


x (y + z) = x . y + x z
(y + z) x = x y + x z I. (x) y = x (y) = (x y)

Assim, definimos tambm: II. (x) (y) = x y

Diferena: x y = x + (y) Esse o famoso "menos com menos d mais e menos com mais
d menos".
x
Diviso: = x y 1, para y 0
y Demonstrao: Para (1), a ideia usar que (x + ( x)) y = 0 y = 0
Axiomaticamente, podemos, em uma igualdade, somar uma mesma Fazendo a distributiva do lado esquerdo, temos x y + ( x) y = 0. Da,
quantidade dos dois lados, como tambm podemos multiplicar os dois basta somar (x y) dos dois lados e ficamos com ( x) y = (x y). A
lados por uma mesma quantidade. Com isso e as regras iniciais, j outra parte de (1) anloga.
possvel demonstrar alguns teoremas.
Para (2), usaremos que ( x) (( y) + y) = ( x) 0 = 0. Fazendo
a distributiva, temos que ( x) ( y) + ( x) y = 0. Usando (1), temos
Teorema 1 (Lei do corte soma): ( x) ( y) + ((x y)) = 0. Agora, basta somar (x y) e ficamos com
( x) (y) = x y.
Se x, y e z so nmeros reais tais que x + y = x + z, ento y = z.
Teorema 5 (Lei do corte produto)
Demonstrao: Na equao x + y = x + z, podemos somar (x)
dos dois lados: (x) + (x + y) = (x) + (x + z). Utilizando a propriedade
a = 0
associativa, podemos somar antes o x ao seu simtrico: (x + x) + y =
Se ax = ay, ento ou .
(x + x) + z. Da, 0 + y = 0 + z, o que nos d y = z.
x = y

AFA-EFOMM 185
Matemtica II Assunto 1

Demonstrao: Em ax = ay, somamos ay dos dois lados, ou x > y> 0


seja, se "passarmos o ay para o outro lado", temos ax ay = 0, ou seja, V. (multiplicar inequaes) xz > yw
a(x y) = 0. Como j vimos, temos que a = 0 ou x y = 0 (ou seja, x = y). z > w > 0
1 1
VI. (inverso) x > y > 0 <
importante observar que voc no dever fazer o passo a passo x y
de nenhum desses teoremas durante os exerccios. Fazemos essas Demonstraes:
demonstraes apenas para a teoria ficar completa e para que voc
aumente sua capacidade de abstrair e utilizar conceitos j dados para I. Como (x + z) (y + z) = x y, a ordenao no se altera;
chegar a novos resultados. II. (x + z) (y + w) = (x y) + (z w) a soma de dois positivos,
ento positivo;
Teorema 6 (Tirando raiz quadrada em uma equao) III. Basta somar as duas inequaes e cancelar o y;
IV. xz yz = (x y)z positivo quando x y e z tm o mesmo sinal;
x > y xz > yz
Se x2 = y2, ento x = y ou x = y. (em que x2 = x x) V. e termina pela transitividade;
z > w yz > yw
Demonstrao: Aqui utilizaremos um produto notvel que ser visto 1 1 xy
mais frente: usaremos que x2 y2 = (x y) (x + y) (*). Ento, em VI. = o quociente entre dois positivos
y x xy
x2 = y2, somemos y2 dos dois lados: x2 y2 = 0. Da, por (*), temos
(x y) (x + y) = 0. Pelo teorema 3, segue que x = y ou x = y. 2.2 Teorema 7 (Quadrado maior ou igual a zero)

Para todo x real, tem-se que x2 0. (* em que x2 = x x)


2. Inequaes
Demonstrao: Caso x seja positivo, x2 = x x o produto de dois
O conjunto dos nmeros reais pode ser dividido em 3 partes: positivos, portanto positivo. Caso x seja negativo, x2 = x x o produto
de dois negativos, portanto positivo. Caso x seja nulo, x2 = x x nulo.
reais positivos (*
+
); Portanto, x2 = x x sempre maior ou igual a zero.
zero (0);
reais negativos (*).
Muitas desigualdades famosas decorrem dessa propriedade e isso
ser cobrado ao longo do material.
* *
+

Erros comuns em inequaes


0
I. No permitido subtrair inequaes
Axiomaticamente, temos que se x e y so reais positivos, ento x + y
e x y tambm so. Alm disso, temos tambm que se x real positivo, 8 > 6
Por exemplo, verdade, mas 8 7 > 6 3 (1 > 3) no !
ento x real negativo. Da, pela regra de sinais, podemos ver que o 7 > 3
produto de dois negativos um positivo e o produto de um positivo por
II. S se pode elevar ao quadrado se os 2 lados so positivos
um negativo negativo.
Por exemplo, 1 > 2 verdade, mas 12 > ( 2)2 (1 > 4) no !
2.1 Relao de ordem
III. No se pode passar varivel multiplicando para o outro lado
Dizemos que:
x > y se x y *+ ; x < y se x y * 1 1 x
Considere as inequaes > (* ) e 1 > (**).
x 2 2
Alm disso, x y se x > y ou x = y; x y se x < y ou x = y. Veja que de (*) para (**), a inequao foi multiplicada por x. No
entanto, caso x fosse negativo, o sinal da inequao deveria ser modificado
Propriedades (propriedade 4).
Esse o erro mais comum neste assunto, tome muito cuidado!
I. (somar dos dois lados / lei do corte) x > y x + z > y + z;
3. Potncias e razes
x > y
II. (somar inequaes) x + z > y + w;
z > w 3.1 Potncia
x > y
III. (transitividade) x > z; Para a real no nulo (base) e n inteiro positivo (expoente) definimos
y > z 1
a = a an1 e a0 =1. Para expoentes negativos, definimos a n = n .
n

x > y , se z > 0 a
IV. (lei do corte produto) xz > yz
x < y , se z < 0

186 Vol. 1
lgebra bsica

Propriedades: Assim, fazendo essas aproximaes cada vez mais precisas, temos
n o valor de 3 2 = 4 ,72880...
an a
I. am an = am + n IV. n
=
b b Com essa expanso da definio de potncia, carregamos todas as
am m n propriedades vistas inicialmente para expoentes inteiros.
II. =a V. (a ) = a
m n mn

an
III. anbn = (ab)n
4. Produtos notveis e fatoraes iniciais
Todas as propriedades podem ser rapidamente demonstradas.
Determinadas expresses aparecem muitas vezes em matemtica. As
primeiras desse tipo so os chamados produtos notveis:
3.2 Razes
1
(0) (distributiva/colocar em evidncia) (a + b) x = ax + bx
Para a real e n inteiro positivo, definimos n a = a n . (1) (distributiva/agrupamento) (a + b)(x + y) = ax + ay + bx + by
(2) (produto de Stevin) (x + a)(x + b) = x2 + (a + b)x + ab
Essa definio bastante natural, j que, de forma habitual, definimos (3) (quadrado da soma) (a + b)2 = a2 + 2ab + b2
n
1 (4) (quadrado da soma de 3 termos) (a + b + c)2 = a2 + b2 + c2 + 2ab
n
como x = a um nmero tal que x = a e a
n n = a. + 2bc + 2ac
(5) (diferena de quadrados) (a + b) (a b) = a2 b2

(6) (cubo da soma) (a + b)3 = a3 + 3a2b + 3ab2 + b3
Observao: Caso n seja par, pelo teorema 6, a equao xn = a nos (7) (soma de cubos) (a + b) (a2 ab + b2) = a3 + b3
d que a 0. Alm disso, para no haver duplo sentido, acrescentamos (8) (cubo da soma de 3 termos) (a + b + c)3 = a3 + b3 + c3 + 3(a + b)
definio que n a 0 para n par. (b + c) (c + a)

No difcil fazer a distributiva no lado esquerdo de cada uma das


Por exemplo, 9 = 3.
equaes acima e chegar ao lado direito. Caso a necessidade fosse
sempre essa, voc poderia refazer isso a cada problema. No entanto, a
Imagine, por um momento, que aceitssemos que radicais de ndice maior utilidade desse ponto j conhecer de antemo essas expresses
par pudessem ter 2 valores (no exemplo anterior, +3 e 3). Neste caso, para que, rapidamente, se possa substitu-las pela sua forma fatorada.
que valor assumiria a expresso 4 + 9 + 16 ? Vrios valores seriam
possveis: 2 + 3 + 4, 2 3 +4, 2 3 4... Com isso, uma expresso Como essas propriedades valem para todos a, b, c e x, podemos
simples geraria uma grande confuso! Para isso no acontecer, aceitamos substituir essas letras como quisermos. Em particular, trocando b por b
apenas o sinal de +. em (1), (4) e (5), obtemos:

Propriedades (3) (quadrado da diferena) (a b)2 = a2 2ab + b2


Para a e b positivos, temos: (6) (cubo da diferena) (a b)3 = a3 3a2b + 3ab2 b3
(7) (diferena de cubos) (a b)(a2 + ab + b2) = a3 b3
n
p a n a
I. n III. =
ap = a n n
b b Todas essas expresses devem ser memorizadas e, para que se tenha
n n n mais facilidade nisso, sugere-se que muitos exerccios sejam feitos.
II. a b = ab IV.
m n
a = mn a
Comentrios:
Dessa forma, pela propriedade (1), definimos potncias para expoentes
racionais (fracionrios). Como j vimos, no difcil partir de cada lado esquerdo at chegar
ao lado direito correspondente. No entanto, em alguns momentos isso
pode parecer artificial. Um bom exemplo (5), j que ser muito mais
Expoentes irracionais
comum aparecer a3 + b3. Caso no soubssemos que a3 + b3 pode ser
Vamos entender este conceito atravs de um exemplo. escrito como (a + b)(a2 ab + b2) , como poderamos chegar a esse
resultado? Em lgebra, assim como em toda matemtica, a maneira mais
Qual seria o valor de 3 2 ? eficiente de se resolver um problema associ-lo a alguma situao j
vista anteriormente. Podemos ver que a3 + b3 est no desenvolvimento
Este um problema computacional. Para chegar a este valor, de (a + b)3, que uma expresso muito comum.
precisamos fazer aproximaes (pelo truncamento da representao
decimal do expoente) por cima e por baixo
Da, (a + b)3 = a3 + 3a2b + 3ab2 + b3 implica a3 + b3 =
31 = 3 32 = 9 = (a + b)3 (3a2b + 3ab2) = (a + b)3 3ab(a + b) =
31,4 = 4,65554... 31,5 = 5,19615...
31,41 = 4,70697... 31,42 = 4,75896... (a + b)2 3 ab = (a + b)(a2 ab + b2 )
  
31,414 = 4,72770... 31,415 = 4,73289... a2 + 2 ab + b2

31,4142 = 4,72873... 31,4143 = 4,72925...
possvel abordar de forma similar outros produtos notveis.

AFA-EFOMM 187
Matemtica II Assunto 1

EXERCCIOS RESOLVIDOS
2
a + ab + ac + bc
01 Simplifique a expresso A = .
b2 + ab + ac + bc

Soluo
Antes de fazer qualquer tipo de cancelamento, precisamos fatorar o numerador e o denominador. E, para fatorar essas expresses com
4 parcelas, normalmente usamos o que chamado de agrupamento:
a2 + ab + ac + bc = a(a + b) + c(a + b) = (a + c)(a + b)
2
b + ab + ac + bc = b(a + b) + c((a + b) = (b + c)(a + b)

(a + c)(a + b) a + c
Substituindo, temos que A = = .
(b + c)(a + b) b + c

a2 1 a2 1
02 Para a 1, prove que a expresso E = no depende de a.
a1 a+1
Soluo
Primeiramente, vejamos que a expresso a2 1 pode ser vista como uma diferena de quadrados e, por isso, pode ser fatorada: a2 12
= (a + 1)(a 1). Da, temos que:

(a + 1)(a 1) (a + 1)(a 1)
E= = (a + 1) (a 1) = 2 , que no depende de a.
a1 a+1

x+y
03 (Desigualdade das mdias para 2 termos) Prove que xy para todo x e y positivos.
2
Soluo
Para provar uma desigualdade, uma das ideias olhar para a diferena entre os dois lados:

( x) ( y) = ( )
2 2 2
x+y x 2 xy + y 2 x y + x y
xy = = .
2 2 2 2
x+y
Como todo quadrado de nmero real 0, ento, segue que xy 0, o que finaliza o problema.
2

5. Tcnicas de fatorao

5.1 Completando o quadrado 5.2 Teorema 8 (frmula de Bhaskara)

Em algumas situaes, no possvel fatorar a expresso dada b


apenas agrupando as parcelas. Uma das sadas pode ser se aproveitar da Para a 0, se ax2 + bx + c = 0, ento x = , em que
= b2 4ac. 2a
semelhana entre a expresso e algum quadrado perfeito.

Ex.: Fatorar a expresso a4 + 4. Demonstrao: A ideia completar o quadrado na equao para que
a varivel x aparea apenas um vez e no duas, como na situao inicial.
Soluo
b c
J que temos uma expresso com apenas 2 parcelas, no possvel Primeiramente, dividimos tudo por a: x 2 + x + = 0.
apenas colocarmos fatores em evidncia. a a
Ento, percebemos que a expresso se assemelha a (a2 + 2)2 e, por b2
isso, fazemos a4 + 4 = a4 + 4a2 + 4 4a2 = (a2 + 2)2 (2a)2 o que Para completar o quadrado, somando , temos:
4 a2
nos leva a uma diferena de quadrados.
Por isso, a4 + 4 = (a2 + 2 + 2a)(a2 + 2 2a). Por uma questo de b b2 b2 c b2 4 ac
x2 + x+ 2 = 2 = .
organizao, comum colocarmos as potncias em ordem decrescente a 4a 4a a 4 a2
de expoente:
2
a4 + 4 = (a2 + 2a + 2)(a2 2a + 2). Da, x + b = e x + b = e est finalizado.

2a 4 a2 2a 2a

188 Vol. 1
lgebra bsica

5.2. Distributiva inteligente 5.3 Quebrando parcelas


O popular agrupamento precisa de 4 parcelas para ser feito. Em
comum o vcio de fazer a distributiva termo a termo quando duas algumas ocasies, temos apenas 3 parcelas e podemos quebrar uma
expresses esto sendo multiplicadas. No entanto, em alguns momentos delas em duas.
interessante agrupar alguns termos antes disso.
Ex.: Fatorar = 3x2 + 10 xy + 3y2.
Ex.: Fatorar E = (a2 + 3a + 3)(a2 + 3a + 5) 15
Soluo
Soluo Aqui, a ideia ser quebrar o coeficiente 10 em duas partes. De nada
Aqui, no vale a pena fazer toda a distributiva termo a termo, porque seria til escrev-lo como 8 + 2, 7 + 3, 6 + 4 ou 5 + 5, pois 8, 7, 6 e 5
ficaramos com uma expresso com muitas parcelas e perderamos a no tm fator comum com 3 e, por isso, no conseguiramos colocar nada
repetio de parcelas que acontece na expresso dada. Por isso, fazemos em evidncia em U. A boa ideia quebrar 10 = 9 + 1. Assim:
E = ((a2 + 3a) + 3)((a2 + 3a) + 5) 15 e temos uma distributiva de
apenas 4 parcelas em vez de 9. Da: U = 3x2 + 9xy + xy + 3y2
U = 3x(x + 3y) + y(x + 3y)
E = (a2 + 3a)2 + 3(a2 + 3a) + 5(a2 + 3a) + 15 15 U = (3x + y)(x + 3y)
E = (a2 + 3a)2 + 8(a2 + 3a)

e podemos colocar a2 + 3a em evidncia, ficando com E = (a2 + 3a)(a2 +


3a + 8). Alm disso, ainda podemos fatorar o 1o fator e E = a(a + 3)(a2
+ 3a + 8).

EXERCCIOS RESOLVIDOS
n+1 n x Para concluir o problema, precisamos fatorar o numerador e, para isso,
01 Sendo xn = 22 + 22 + 1, prove que n +1 inteiro para todo n
natural. xn vamos completar o quadrado.

Soluo Temos a4 + a2 + 1 = a4 + 2a2 + 1 a2 = (a2 + 1)2 a2 = (a2 + 1


n n+1 n n
Inicialmente, faa 22 = a. Veja, ento, que 22 = 22 2 = (22 )2 = a2 + a)(a2 + 1 a). Substituindo em (*), temos
n+2 n n
e 2 2
= 2 2 2 2
= (2 ) = a . Com isso, temos que
2 4 4

n+2 n+1 x n +1 a4 + a2 + 1 (a2 + a + 1)(a 2 a + 1)


x n +1 22 + 22 + 1 a4 + a2 + 1 = 2 = = a2 a + 1, que
= n+1 n
= 2 (* ) . xn a + a+1 a2 + a + 1
xn 22 + 22 + 1 a + a+1
inteiro.

6. Radicais
Uma situao muito comum encontrarmos uma expresso que Soluo
contenha radicais no denominador. Veja que efetuar uma diviso por um Aqui, faremos uso do produto notvel (a + b)(a b) = a2 b2 e
nmero irracional pode nos levar a erros de aproximao, dependendo da multiplicaremos o numerador e o denominador por 4 3 :
preciso com que se tenha o denominador.
1 4 3 4 3 4 3
1 = = =
Por exemplo, considere o clculo do nmero . Caso utilizemos a 4+ 3 ( 4 + 3 )( 4 3 ) 16 3 13
3
1 1 Comentrio: Uma outra forma de tratar o problema encontrar uma
aproximao 3 1,7, encontraremos 0,5888. Caso refinemos
3 1, 7 equao que tenha a expresso do denominador como raiz. No exemplo,
1 1 fazendo 4 + 3 = x, temos x 4 = 3 e, elevando ao quadrado, x2 8x +
a aproximao com 3 1,73, obteremos 0,5780.
3 1, 73 16 = 3 , o que nos d a equao x2 8x + 13 = 0, que era o nosso objetivo.

Por isso, como na prtica no temos todas as casas de 3, a diviso 1


1 Agora, podemos isolar :
no deve ser feita dessa maneira. x
3
1 8 x
Por outro lado, se multiplicarmos o numerador e o denominador de ( x 8) x = 13 =
x 13
1 3 1, 73205 Veja que, fazendo isso, eliminamos o radical do denominador.
por 3 , teremos 0,5773.
3 3 3
Substituindo o valor de x, temos
1 8 x 8 4+ 3
= =
(=
4 3
.
)
A esse processo de eliminao de radicais em denominadores, damos x 13 13 13
o nome de racionalizao.

1
Ex.: Racionalizar .
4+ 3

AFA-EFOMM 189
Matemtica II Assunto 1

6.1 Teorema 10 (radicais duplos): Demonstrao: Basta comparar os quadrados dos dois lados.
(Para uma motivao de como chegar a essa expresso, faa
A B = x y e escolha x e y convenientes.)
A+C A C , em que
A B = C= A2 B Em geral, conveniente usar essa frmula quando A2 B for quadrado
2 2 perfeito, para que transformemos um radical duplo em uma soma de
radicais simples.

EXERCCIOS RESOLVIDOS
01 Prove que o nmero x = 3 8 3 + 8 inteiro e negativo.

Soluo
bvio que x negativo, pois 3 8 < 3 + 8 . Para provar que x inteiro, elevamos ao quadrado:

( ) =( ) ( )
2 2 2
x2 = 3 8 3+ 8 3 8 2 3 8 3+ 8 + 3+ 8

( 8)
2
x 2 = 3 8 2 32 + 3 + 8 x 2 = 4.

Como x negativo, temos que x = 2, que inteiro.

Tambm possvel utilizar a frmula do radical duplo.

02 Qual o valor de x = 6 + 6 + 6 + 6 + ... , com infinitos radicais?

Soluo
2
Elevando ao quadrado, temos que x = 6 + 6 + 6 + 6 + ... . Como h uma quantidade infinita de radicais, ficamos com x2 = 6 + x, ou
seja, x2 x 6 = 0. As razes dessa equao so x = 3 e x = 2. Como x um radical, x positivo.
Por isso, x = 3.

Obs.: A rigor, antes de darmos os argumentos acima deveramos provar que x um nmero real. Como x o limite de uma sequncia, neste caso,
poderia ser que x tendesse ao infinito. Aqui, para formalizar, basta provar (por induo no nmero de razes) que x menor que 3.

Agora, veremos outras tcnicas de fatorao. 8.2 Teorema 12 (teste das razes racionais)
7. Diferena e soma de potncias Considere uma equao polinomial em x de coeficientes inteiros

Dois produtos notveis que tambm podem ser muito teis so: E = bmxm + bm1xm1 +...+b1x + b0
p
Se x = anula a expresso E (frao irredutvel, ou seja, p e q inteiros e
(1) xn an = (x a)(xn1 + axn2 + a2xn3 +...+ an1), para n natural q
(2) xn + an = (x + a)(xn1 axn2 + a2xn3 ...+ an1), para n natural mpar
p divisor de b0
Para demonstrar as duas relaes, basta fazer a distributiva nos lados primos entre si), ento .
q divisor de bm
direitos. A demonstrao deste teorema ser vista em outro assunto mais frente.
8. Razes Fatores Esse teste muito til pois nos d uma lista de fraes que podem
8.1 Teorema 11 (raiz implica fator) anular a expresso em questo. Da, usando o teorema 11, podemos
encontrar uma fatorao diretamente.
Se x = a anula uma expresso polinomial em x, ento (x a) um Ex.: Fatorar K = 2x3 + 5x2 x 1 .
fator dessa expresso.
Soluo
Demonstrao: Considere a expresso E = bmxm + bm1xm1 +...+b1x Pelo teste das razes racionais, as possveis razes racionais de K
+ b0. Se x = a anula a expresso E, ento bmam + bm1am1 +...+ b1a + 1 1 1
so + 1, 1, + , . Testando uma a uma, vemos que x = anula a
b0 = 0. Subtraindo uma equao da outra, ficamos com E = bm(xm am) 2 2 2
1
+ bm1(xm1 am1) +...+ b1(x a). expresso K. Por isso, sabemos que x um fator de K, o que
2
Agora, veja que pelo produto notvel da diferena de razes, cada
equivalente a 2x 1 ser fator K.
parnteses tem (x a) como fator e, assim, E possui (x a) como fator.
Agora, basta forar o surgimento desse fator somando e subtraindo
os termos corretamente:

190 Vol. 1
lgebra bsica

K = 2x3 + 5x2 x 1 = 2x3 x2 + 6x2 3x + 2x 1 Formalmente, o Lema de Gauss diz o seguinte: "se um polinmio de
x (2x 1) + 3x (2x 1) + 1 (2x 1) = (2x 1) (x + 3x + 1).
2 2 coeficientes inteiros pode ser fatorado como produto de polinmios de
coeficientes racionais, ento ele tambm pode ser fatorado como produto
de polinmios com coeficientes inteiros".
9. Expresses homogneas Na prtica, isso significa que, no seu rascunho, voc deve supor
que os coeficientes dos fatores so inteiros, porque se no forem, sero
Em uma expresso algbrica, definimos o conceito de grau de uma irracionais e ser difcil encontr-los.
parcela como a soma dos expoentes nas suas variveis. Por exemplo, o
grau de x3 y5 z2 10. Ex.: Fatorar U = x4 + 3x3 + 3x2 2 .

Dizemos que uma expresso homognea quando todas as suas Soluo


parcelas tm o mesmo grau. Por exemplo, x3 + 3x2y + 7xy2 4y3 Primeiramente, veja que os candidatos a razes racionais de U so
homognea pois todas as suas parcelas tm grau 3. +1, 1, +2, 2. Testando, veja que nenhuma delas funciona. Com isso,
Caso haja uma expresso homognea, h um artifcio, muitas vezes no temos razes racionais e, por isso, no h fatores de grau 1 em U.
com vantagens at mais psicolgicas, que pode ser muito til. Ento, devemos escrever U como um produto de dois fatores de grau 2:

Ex.: Considere a equao x3 + 3x2y + 7xy2 11y3 = 0. Veja que U = (ax2 + bx + c)(dx2 + ex + f)
x = y = 0 soluo. Para y 0, divida tudo por y3:
A ideia fazer a distributiva e igualar os coeficientes aos da expresso
3 2 original. No coeficiente de x4, temos ad = 1. Agora que temos a
x x x vantagem de considerar que os coeficientes so inteiros. Veja que temos
+ 3 + 7 11 = 0
y
y
y a = d = 1 ou a = d = 1. Podemos considerar que a = d = 1, pois no
x 2o caso bastaria multiplicar os 2 fatores de U por 1. Ento, ficamos com:
Fazendo = t , reduzimos a equao original de duas variveis a
y
U = (x2 + bx + c)(x2 + ex + f)
equao de apenas uma varivel t3 + 3t2 + 7t 11 = 0. Agora, poderamos
seguir utilizando as tcnicas de fatorao j vistas.
Agora, analisando o termo independente de x, temos que cf = 2. Para
c e f inteiros, temos 2 casos: c = 1, f = 2 ou c = 1, f = 2.
10. Mudana de variveis
1o caso: c = 1, f = 2
Em muitos problemas, interessante fazer uma mudana de variveis
para simplificar a soluo. U = (x2 + bx + 1)(x2 + ex 2) = x4 + (b + e)x3 + (1 + be)x2 +
(e 2b) x 2

12
Ex.: Determinar as razes reais de x 2 + 3 x + 1 = 2
. b + e = 3
x + 3x + 2
Igualando os coeficientes, temos o sistema e 2 b = 0 .
Soluo 1 + be = 3

Inicialmente, repare que se multiplicarmos, chegaremos a uma
equao do 4a grau, o que no seria bom. Ao perceber a semelhana entre As duas primeiras equaes nos do b = 1, e = 2 que no funcionam
12 na terceira equao.
as expresses, faamos x2 + 3x + 2 = a. Da, a equao a 1 = ,
a 2o caso: c = 1, f = 2
que equivale a a2 a 12 = 0, que tem solues a = 4 e a = 3. U = (x2 + bx 1)(x2 + ex + 2) = x4 + (b + e)x3 + (1 + be)x2 +
Substituindo de volta, ficamos com as equaes x2 + 3x 2 = 0 e x2 + (2b e) x 2.
3 17 b + e = 3
3x + 5 = 0. Resolvendo-as, temos que x = .
2 Igualando os coeficientes, temos o sistema 2 b e = 0
1 + be = 3

11. Lema de Gauss (mtodo dos
As duas primeiras equaes nos do b = 1, e = 2, que funcionam na
coeficientes a determinar) terceira equao.

Em geral, ao tentar fatorar uma expresso algbrica, tentamos usar Portanto, temos que U = (x2 + x 1)(x2 + 2x + 2).
as ferramentas estudadas na seguinte ordem:
Veja que este processo pode ser muito trabalhoso caso o coeficiente
colocar em evidncia/agrupamento; independente de x possua muitos divisores.
quebrar parcelas/completar quadrado;
chutar uma raiz racional para obter um fator de grau 1;
fazer alguma mudana de variveis.
Caso nenhuma dessas tentativas d certo, podemos utilizar o mtodo
deste tpico.

AFA-EFOMM 191
Matemtica II Assunto 1

EXERCCIOS RESOLVIDOS
01 Resolva a equao x4 + 3x3 2x2 + 3x + 1 = 0 nos reais. Substituindo na equao, temos (t2 2) + 3t 2 = 0, que d

Soluo t2 + 3t 4 = 0. Essa equao tem razes t = 4 e t = 1. Colocando


Essa equao polinomial chamada de recproca (isso acontece quando 1
em x + = t , temos as equaes do 2o grau x2 + 4x + 1 = 0 e x2 x
os coeficientes equidistantes do centro so iguais). Nesse caso, temos uma x
+ 1 = 0. A primeira d as razes x = 2 3 e a segunda no tem
3 1
soluo padro. Dividindo tudo por x2 temos que x 2 + 3 x 2 + + = 0, razes reais.
x x2
{
Ento, S = 2 3 . }
que pode ser escrita como x 2 + 1 + 3 x + 1 2 = 0. Agora,
x
x2
2
1 1 1
fazendo x + = t, temos x + = t 2 , o que nos d x 2 + 2 = t 2 2.
x x x

EXERCCIOS NVEL 1

x 3 1010 + 1020 + 1030


01 Quantas solues tem a equao = ? 09 (UFF) Qual o valor simplificado da frao ?
x 3 x 3 1020 + 1030 + 1040
02 Resolva a equao (x + 1)(x 4)(x 2)(x2 + 3x + 2) = (x + 1) 10 (OCM) Qual dos nmeros maior:
(x 4)(x 2)(x2 + 8x + 3).
999
3 123456 + 10999 ou 123457 + 10 ?
03 Resolva = 2. 123457 + 10999 123458 + 10 999
3
1+
3
1+ 11 (ITA-2002) Considere as seguintes afirmaes sobre nmeros reais
1 x
positivos:
2 2
04 Resolva = . I. Se x > 4 e y < 2, ento x 2y > 12.
1 5
5+ II. Se x > 4 ou y < 2, ento x 2y > 12.
1
4+ II. Se x < 1 e y > 2, ento x 2y < 0.
1
5 Ento, destas (so) verdadeira(s):
x+2
1 1 (A) apenas I. (D) apenas I e III.

1 x 1+ x (B) apenas I e II. (E) todas.
05 Determine as solues de = 1. (C) apenas II e III.
x 1
+
1 x 1+ x
12 (OBM 1a fase 2008) Sendo x = 102008, assinale a alternativa que
1 apresenta o maior valor.
06 A expresso 1 igual a:
a
1+ 1
1 a (A) . (D) x.
x
(A) a, se a 0. (D) 1 a, para todo a.
1
(B) 1, para todo a. (E) a, se a 1. (B) . (E) x .
(C) a, se a 1. x( x + 1) 1
x+
x
a y y a 1
07 ( E N ) S e j a + = 1, a 0 . (C) .
a+ y a y a+ y a y 1
1+
1
A igualdade vlida: 1+
x
(A) para todos, exceto dois, valores de y.
(B) s para dois valores de y. 13 (OBM 1a fase 2011) Sendo a e b reais tais que 0 < a 1 e 0 < b 1,
(C) para todos os valores de y.
o maior valor que a b pode assumir :
(D) s para um valor de y. a+ b
(E) para nenhum valor de y.
1
(A) 0. (D) .
2
x2 + 1 + x2 1 x2 + 1 x2 1 1
08 Calcule + . (B) . (E) 1.
2
x +1 2
x 1 2
x +1+ 2
x 1 4
1
(C) .
3

192 Vol. 1
lgebra bsica

2 3
14 (OBM 1a fase 2010) Qual das seguintes fraes mais prxima de 7? 21 (EFOMM 03) Que termo se deve acrescentar ao binmio x + b x
4 3
3 13 de modo que se obtenha um trinmio que seja quadrado perfeito?
(A) . (D) .
1 5 6 b3
5 18 (A) b . (D) .
(B) . (E) . 3 3
2 7
b4 b6 .
8 (B) . (E)
(C) . 9 9
3
6
15 (OBM 1a fase 2011) Qual o valor da expresso 201120112 + (C) b .
201120032 16 20112007? 2
22 (OBM 1a fase 2006) Os dois nmeros reais a e b so no nulos e
(A) 2 201120072. (D) 2 20112003. satisfazem ab = a b. Assinale a alternativa que exibe um dos possveis
(B) 2 201120032. (E) 2 201120112. a b
valores de + ab.
(C) 2 20112007. b a
(A) 2. (D) 1 .
16 (SPIA) Fatore as expresses algbricas abaixo: 2
1
(B) . (E) 2.
a. a4 1; f. a5 + a3 a2 1; 2
b. a6 1; g. a4 + 2a3 2a 1; 1
c. a6 + 1; h. 4b2c2 (b2 + c2 a2)2; (C) .
d. a4 18a2 + 81; i. c4 (1 + ab)c2 + ab; 3
e. a12 2a6 + 1; 23 (SPIA) Fatore as expresses algbricas a seguir:

2 a. a4 + a2b2 + b4; f. a4 + a2 + 1;
17 Se x + 1 = 3, ento, x 3 + 1 igual a: b. a + 4a 5;
4 2
g. a4 + 9;
x x3 c. 4a4 + 5a2 + 1; h. a4 + 4b4;
d. a5 + a4 + a3 + a2 + a +1; i. (a + b + c)3 (a3 + b3 + c3);
(A) 0. (D) 3. e. a + 324;
4
j. (a b)3 + (b c)3 (a c)3;
(B) 1. (E) 4.
(C) 2.
( a2 b2 )3 + ( b2 c2 )3 + ( c2 a2 )3
24 Simplifique .
18 (SPIA) Simplifique as expresses abaixo: ( a b)3 + ( b c)3 + ( c a)3

a a2 + a 1 a2 a 1 2 a3 25 Calcule a2 .
a. 2 + 3 2
+ 3 2
4 ;
a 1 a a + a1 a + a + a+1 a 1
26 Calcule a4 .
1 1
+
a b+ c b 2 + c 2 a2 27 Calcule 2 2 3 (1 + 3 ).
b. 1 + ;
1 1 2 bc

a b+c
28 Calcule 2 3 ( 6 2 )( 2 + 3 ).
1 1 1
c. + + ;
( a b)( a c) ( b c)( b a) ( c a)( c b) 29 Racionalize as fraes:

1 5+ 3
a+b b+c c+a (A) . (C) .
d. + + ; 2+ 3 5 5 3
( b c)( c a) ( c a)( a b) ( a b)( b c)
1 1
(B) 4 . (D) .
5 3 4 14 + 21 + 15 + 10
19 (OBM 1a fase 2009) Se x2 = x + 3, ento x3 igual a:

(A) x2 + 3. (D) 4x + 3. 30 Sendo a, b e c positivos, simplifique a2 + 4 ab + 6 ac + 4 b2 + 12 bc + 9c2 .


(B) x + 4. (E) x2 2.
(C) 2x + 2. 31 (OBM 1a fase 2006) Sejam x e y nmeros racionais. Sabendo que
x 5 2006
tambm um nmero racional, quanto vale o produto xy?
20 (CN) Sabe-se que a3 3a + 1 = 93 e K = a4 6a + 1. Logo, K 4 y 2006
tambm pode ser expresso por:
(A) 20.
(A) 3a2 + 86a + 1. (D) 6a2 + 84a + 1.
(B) Pode ser igual a 20, mas tambm pode assumir outros valores.
(B) 3a2 + 84a + 1. (E) 9a2 + 86a + 1.
(C) 1.
(C) 6a2 + 86a + 1.
(D) 6.
(E) No se pode determinar.

AFA-EFOMM 193
Matemtica II Assunto 1

32 (SPIA) Simplifique as expresses abaixo: 04 (CN) Sejam A = 72011, 112011 e B = {x / x = (1 t) 72011 + t


1 112011, com t [0,1]}, o conjunto A B igual a:
1 4
1 1 2 a 3 2 1 3 (A) A B. (D) A.
(A) + a ;
1 1 1 1 2 1
a3 a6 + 1 a3 + a6 + 1 a3 a 3 + 1 4 (B) B {112011}. (E) .
(C) A {72011}.
1/ 2
4 4 4 4
1
(B) b ( a b ) + 2 ab 4 b + 1 + 1 8 ab ; 05 (OBM 1a fase 2010) Os nmeros a e b so reais no negativos tais
( 4 b + 4 a )2 a que a3 + a < b b3. Ento:

1
2 2
1
1 1
2
(A) b < a < 1. (D) a < b < 1.
( a + b) a 3 b 3 ( 3 a2 b 3 ab2 ) b 3 a 3
(B) a = b = 1. (E) 1 < a < b.
(C) + 26 a ; (C) a < 1 < b.


6 6
( 3 6
( a + b ) b + ab 2 a 3
)
2 2 3 3 3
06 Para quais valores reais de a e b vale a + b a + b ?

1
6 5 6 2 3 6 3 2 6 5 6 9 6 10 07 ( A FA 1 9 9 7 ) O p r o d u t o d a s r a z e s d a e q u a o
(D) b a b + a b a ab + a + 1.
( ) +( ) = 4 pertence ao conjunto dos nmeros:
x x
6
b+6a a ab + b 2+ 3 2 3

33 (OBM 1a fase 2006) Quantos ternos de nmeros reais x, y, z satisfazem (A) naturais e primo.
o sistema abaixo? (B) inteiros e mltiplo de 4.
(C) complexos e imaginrio puro.
x( x + y + z ) = 2005 (D) racionais positivos e uma frao imprpria.
y( x + y + z ) = 2006
( )
2011
z( x + y + z ) = 2007 08 (OBM 1a fase) Sendo a e b inteiros tais que 1 + 2 = a + b 2,
(1 2 )
2010
igual a:
(A) Nenhum. (D) 3.
(B) 1. (E) 2006. (A) a + 2 b + ( a b) 2 . (D)
2 b a + ( b a) 2 .
(C) 2. (B) a 2 b + ( a b) 2 . (E)
a + 2 b ( a + b) 2 .
(C) a + 2 b + ( b a) 2 .
34 (OBM 1a fase 2013) Determine x + y, onde x e y so reais, sabendo
que x3 + y3 = 9 e xy2 + x2y = 6. 09 (CN) O nmero real 3 26 15 3 igual a:

(A) 1. (D) 4. (A) 5 3 . (D)


13 3 3 .
(B) 2. (E) 5.
(C) 3. (B) 7 4 3 . (E) 2.
(C) 3 2 .
EXERCCIOS NVEL 2
(3 + 2 2 )
2008

10 (CN) O valor de + 3 2 2 um nmero


(5 2 + 7)
1338
01 (OBM 1a fase 2010) Os nmeros x e y so distintos e satisfazem
1 1
x =y . Ento xy igual a:
x y (A) mltiplo de 11. (D) mltiplo de 3.
(B) mltiplo de 7. (E) primo.
(A) 4 (D) 4 (C) mltiplo de 5.
(B) 1 (E) So necessrios mais dados.
(C) 1
11 (OBM 1a fase 2005) O nmero ( 2 + 2 )3 (3 2 )4 + ( 2 2 )3
02 Sendo a, b e c nmeros distintos, simplifique a expresso abaixo: (3 + 2 )4 :

2 bc 2 ca 2 ab (A) inteiro mpar. (D) irracional positivo.


+ + + + +
b c ( c a) ( a b) c a ( a b) ( b c) a b ( b c) ( c a) (B) inteiro par. (E) irracional negativo.
(C) racional no inteiro.
(A) a + b + c (D) 2abc.
(B) a b (E) 0. 12 (CN) Se a, b, c e d so nmeros reais no nulos tais que ad2 + bc2
(C) (a b)(b c)(c a) = 0, pode-se afirmar que:

03 (OBM 1a fase 2012) Se x2 = 2x + 4, ento (x + 1)1 igual a: a c a+c c b b+c


(A) + = ; b + d 0. (D) + = ; a + d 0.
b d b+d a d a+d
(A) x + 2. (D) 2x + 5.
(B) x 3. (E) 3x + 5. a b a+ b c d c+d
(B) + = ; c + d 0. (E) + = ; a + b 0.
(C) x 1. c d c+d b a a+ b
a b a+ b
(C) + = ; c + d 0.
d c c+d
194 Vol. 1
lgebra bsica

1 1 1 14 (CN) Sejam y e z nmeros reais distintos no nulos tais que


13 (CN) Sejam a, b e c nmeros reais no nulos tais que + + = p,
a b c a b c ab bc ac 4 y2 z2
+ + + + + = q e ab + ac + bc = r. O valor de q + 6q 2 + + = 3. Qual o valor de y + z?
b a a c c b yz 2 z 2 y

sempre igual a:
(A) 2. (D) 2.
2 2 (B) 1. (E) 3.
p r +9 (C) 0.
(A) .
4
p2 r 2 9 p 15 (OBM 1a fase 2010) Qual o maior valor de xy2 se x e y so reais
(B) .
12 positivos cuja soma 3?
(C) p2r2 9. (A) 3. (D) 6.
2 2 (B) 4. (E) 7.
p r 10 (C) 5.
(D) .
4r
(E) p2r2 12 p.

RASCUNHO

AFA-EFOMM 195
Sequncias A ssunto
1
Matemtica III

1. Introduo Para ver que essa relao verdadeira, basta pensar que, cada vez
que andamos para frente na sequncia, somamos a razo uma vez. Como
O presente assunto tem por objetivo definir o que uma sequncia, queremos chegar ao termo na posio n, partindo do primeiro termo,
estudar os principais tipos (progresses aritmticas e geomtricas), e devemos dar n 1 passos na sequncia, somando ento n 1 vezes r.
determinar os seus termos, conhecendo-se os termos iniciais.
De forma geral, vale a seguinte relao:
Alm disso, estudaremos a soma de seus elementos, as propriedades
da P.A. e da P.G. e aplicaes em matemtica financeira, como juros an = ap + (n p)r
simples e compostos.
Finalmente veremos outros tipos de sequncias, como defini-las de
forma recursiva (em funo de termos anteriores), e alguns mtodos para 3.2 Propriedades da P.A.
obter um termo geral, como a soma telescpica e outros truques algbricos.
Uma das principais propriedades da P.A. a simetria em relao
ao centro. Assim, quando temos uma P.A. com um nmero pequeno
2. Sequncias de termos, podemos escrev-la a partir do termo central para facilitar
algumas contas.
De forma intuitiva, uma sequncia uma ordenao dos elementos de
um conjunto, ou seja, devemos associar cada elemento a uma posio, de P.A.s com um nmero mpar de termos:
modo que exista um primeiro elemento (a1), um segundo elemento (a2), 3 termos: (x r, x, x + r)
um terceiro (a3) e assim por diante.
5 termos: (x 2r, x r, x, x + r, x + 2r)
Chamaremos de ai o termo na posio i. Veja que associamos cada
elemento a um nmero natural i (sua posio). E assim sucessivamente.

Ex.: P.A.s com um nmero par de termos:


(1, 2, 3, 4, 5, ..., n) (sequncia dos n primeiros nmeros naturais); 4 termos: (x 3r, x r, x + r, x + 3r)
(1, 1, 2, 3, 5, 8, ...) (sequncia de Fibonacci: cada termo a soma 6 termos: (x 5r, x 3r, x r, x + r, x + 3r, x + 5r)
de dois anteriores);
(3, 2, 7, 12, 17, 22) (diferena entre termos consecutivos constante);
(5, 10, 20, 40, 80) (razo entre termos consecutivos constante). Ateno: Repare que, ao escrever uma P.A. com um nmero par de
termos nessa forma, a razo da P.A. 2r.

3. Progresso aritmtica (P.A.) Escrevendo a P.A. com esse formato, conseguimos visualizar outra
propriedade importante da P.A.: a soma de termos equidistantes do centro
Chamamos de progresso aritmtica toda sequncia (a1, a2, a3, ..., an) (ou das pontas) constante, ou seja:
cuja diferena entre termos consecutivos constante:
a1 + an = a2 + an 1 = a3 + an 2 = = ap + an p + 1
ak ak 1 = r = cte, k {2, 3, ..., n}
Alm disso, se a P.A. possuir um termo central ento esse termo a mdia
Neste caso, dizemos que r a razo da P.A. aritmtica das extremidades.

Ex.: (3, 7, 11, 14, ...) P.A. de razo 4. 3.3 Soma da P.A.
Veja que a P.A. fica bem definida se dermos um termo e sua razo, uma Dada uma P.A. (a1, a2, a3, ..., an), definimos Sn como a soma dos n
vez que qualquer termo pode ser obtido atravs desses dois parmetros. primeiros termos da P.A., ou seja,

Dizemos que a P.A. crescente se r > 0 e decrescente se r < 0. No Sn = a1 + a2 + + an


caso em que r = 0 dizemos que a P.A. estacionria.
Como podemos calcular essa soma?
3.1. Termo geral
Ex.:
Como dito anteriormente, todo termo de uma P.A. pode ser obtido S = 1 + 2 + 3 + ... + 100
atravs de outro termo e da razo. Por exemplo, se tivermos o termo S = 100 + 99 + 98 + ... + 1
inicial a1 e a razo r podemos determinar an atravs da seguinte relao:
Somando ambas as equaes: 2S = 101 + 101 + ... + 101 =
an = a1 + (n 1)r 100 vezes
101 100 S = 101 50 = 5050

196 Vol. 1
Sequncias

Repare que escrevendo a P.A. ao contrrio juntamos os termos Mais importante que entender a demonstrao da frmula lembrar
equidistantes das pontas, e como vimos anteriormente, a soma desses da ideia por trs dela. Quando queremos calcular uma soma, podemos
termos sempre igual soma das extremidades, assim: perturb-la, ou seja, podemos pensar em algum meio de achar uma
expresso muito parecida com ela, para subtrair (ou fazer outra operao)
Sn = a1 + a2+ + an de modo que a maioria dos termos cancele.
Sn = an + an1+ + a1 Veja que essa foi a motivao principal para multiplicar pela razo, uma
vez que ao multiplicar os termos de uma P.G. por q, apenas andamos
Somando e utilizando a propriedade anterior: com a P.G. para frente.

( a1 + an ) n
2Sn = (a1 + an) n Sn = Outro modo de enxergar a frmula da soma da P.G. usar a prpria
2 definio de P.G.:
Ex.:
I. Sn = 1 + 2 + 3 + ... + n = (n(n + 1))/2 a2 = a1 q
II. Sn = 1 + 3 + 5 + ... + (2n 1) = n2 a3 = a2 q
a4 = a3 q
..............
4. Progresso geomtrica an = an-1 q

Chamamos de progresso geomtrica toda sequncia (a1, a2, a3, ..., an) Somando tudo: Sn a1 = q(Sn an ) qSn Sn = qan a1 = a1
cuja razo entre termos consecutivos constante:
q n 1
qn a1 Sn = a1 , q 1
ak / a k 1 = q = cte, k {2, 3, ..., n} q 1

Obs.: Se a razo for igual a 1, a P.G. constante e sua soma igual


Neste caso, dizemos que q a razo da P.G. a Sn = a1 + a1 + + a1 = na1

Ex.: (3, 6, 12, 24, ...) P.G. de razo 2


4.3 Produto dos termos da P.G.
Veja que, como na P.A. a P.G. fica bem definida se tivermos um termo
e sua razo; assim podemos achar o termo geral, utilizando a mesma Sendo Pn o produto dos n primeiros termos de uma P.G., tem-se:
ideia da P.A..
Pn = a1a2 ... an = a1(a1q)(a1q2) (a1qn1) Pn = a1n q1 + 2 +...+ (n 1)
Dizemos que a P.G. crescente se ak ak1 > 0.
Para que isso ocorra devemos ter: a1 > 0 e q > 1 ou a1 < 0 e 0 < q < 1.
n ( n 1)

Pn = a1n q 2

Dizemos que a P.G. decrescente se ak ak1 < 0.


Para que isso ocorra devemos ter: a1 > 0 e 0 < q < 1 ou a1 < 0 e q > 1.
Outra maneira de calcular o produto usando uma propriedade antes
Caso q < 0, dizemos que a P.G. alternante e, se q = 1, dizemos citada em P.A., que continua valendo para P.G: o produto dos termos
que esta estacionria. equidistantes das pontas igual ao produto dos extremos.

4.1. Termo geral a1an = a1an


a a = aa
2 n 1 1 n
Em funo do primeiro termo e da razo: an = a1qn 1 a3 an 2 = a1an Multiplicando as equaes: Pn2 = ( a1an )n
.

Em funo de um termo qualquer e da razo: an = apqn p an a1 = a1an

4.2. Soma dos n primeiros termos da P.G. Obs.: Deve-se tomar cuidado ao extrair a raiz quadrada na relao,
pois o produto dos termos pode ser negativo (dependendo da quantidade
Considere uma P.G. (a1, a2, a3, ..., an) de razo q 1, seja: de termos negativos na sequncia).

Sn = a1 + a2 + + an = a1 + a1 q + a1q2+ + a1qn1 (1) 4.4 Progresso geomtrica infinita


Multiplicando por q: qSn = a1q + a1q2 + a1q3+ + a1qn (2)
Chamamos de P.G. infinita toda P.G. com um nmero infinito de termos.
Subtraindo (2) (1): Sn(q 1) = a1(q 1)
n Dependendo da razo desta P.G., podemos calcular a soma de seus
elementos, ou seja, existem alguns casos em que a soma infinita converge
q n 1 (resultando em um nmero finito).
Sn = a1 , q 1
q 1

AFA-EFOMM 197
Matemtica III Assunto 1

Sabemos que numa P.G. finita vale a seguinte frmula: fcil ver que isso ocorre para todo q, com |q| < 1. Nesse caso,
iremos trocar qn na P.G. finita por zero, assim:
q 1 n
Sn = a1 , q 1;
q 1 0 1 a1
S = a1 S = , q <1
em que n representa o nmero de termos a serem somados. Queremos q 1 1 q
saber o que ocorre quando esse n tende a infinito.
Vamos analisar, por exemplo, o caso q = 1/2: +;a1 > 0
2 3 10
1 1 1 1 1 1 Obs.: Caso q 1, temos: S = ;a1 < 0 ,
2 = 4 ; 2 = 8 ; 2 = 1024 0;a = 0
1

Veja que quanto maior o expoente menor o valor de qn; assim, se n


basta pensar no que ocorre com qn na frmula da P.G. finita. Se q 1,
tende a infinito, podemos ver que qn tende a zero.
a soma infinita no existe.

EXERCCIOS RESOLVIDOS
01 Calcule o valor da soma 2 + 5 + 8 + 11 + ... + 92. A distncia total percorrida por ele ao trmino de 15 dias representa
uma soma de P.A., logo:
Soluo ( a1 + a15 ) . 15 = 67500 a
Esta a soma de uma P.A. com a1 = 2, an = 92 e r = 3. Antes de 1 + ( a1 + 14 500 ) =
2
calcularmos a soma, precisamos saber quantos termos h. Usando que
2 67500
an = a1 + (n 1)r, temos 92 = 2 + (n 1) 3, que nos d n = 31. = = 9000 a1 = 2000
15
( a + a ) n ( 2 + 92).31 = 1457.
Portanto, segue que a soma igual a: 1 n = Desse modo: a3 = a1 + 2r = 3000 m
2 2
02 Prove que se (a, b, c) simultaneamente uma P.A. e uma P.G., 05 A soma dos 11 primeiros termos da progresso aritmtica (a1, a2,
ento, a = b = c. ..., an, ...) 176. Se a11 = a1 + 30, ento, para qualquer n * temos:

Soluo (A) an = 3n 2.
a+c (B) an = 2n 3.
Como P.A., temos b = . Como P.G., temos b2 = ac.
2 (C) an = n + 3.
Substituindo a 1 na 2 , temos que:
a a
(D) an = 2n + 3.
a+c
2 (E) an = 3n + 2.
2 2
2 = ac a + 2 ac + c = 4 ac
Soluo
a2 2 ac + c2 = 0 ( a c ) = c a = c
2
Letra A.

S11 =
( a1 + a11 ).11 = 176 a + a11 = 32, porm, como a11 = a1 + 30,
a+c 1
Como b = , segue que b = a = c. 2
2
temos: a1 = 1 e a11 = 31. Usando que a11 = a1+10r, tm-se: r = 3,
03 O produto dos 15 primeiros termos da progresso geomtrica, de logo:
primeiro termo 1 e razo 10, vale: an = a1 + (n 1)r = 1 + (n 1) 3 = 3n 2.

Soluo 06 A soma de trs nmeros em P.G. 26 e o produto 216. Ento,


n ( n 1) 15 14
os termos da P.G. valem:
Usando que Pn = a1n q 2
P15 = 10 2
= 10105

04 Um atleta corre sempre 500 metros a mais do que no dia anterior. Soluo
Sabendo-se que ao final de 15 dias ele correu um total de 67.500 Trs nmeros em P.G. (x/q, x, xq), multiplicando: x3 = 216 x = 6.
metros, qual o nmero de metros percorridos no terceiro dia?
1 1 10
Somando: x q + 1 + = 26 q + = 3q2 10q +
Soluo q q 3
Veja que, como o atleta sempre corre 500 m a mais que no dia 1
3 = 0 q = 3 ou q = .
anterior, a sequncia formada pelas distncias percorridas diariamente 3
por ele uma P.A. de razo 500. Logo: (2, 6, 18) ou (18, 6, 2).

198 Vol. 1
Sequncias

07 Em um certo jogo de azar, apostando-se uma quantia X, tem-se uma 09 Em uma P.G. com n termos, sejam S a soma dos termos, I a soma
n
das duas possibilidades seguintes: S
dos inversos e P o produto. Demonstre que P 2 = :
I
I. Perde-se a quantia X apostada;
Soluo
II. Recebe-se a quantia 2X.
fcil ver que a relao vlida caso a razo seja igual a 1. Supondo
Uma pessoa jogou 21 vezes da seguinte maneira: na primeira vez, q n 1
que a razo no seja igual a 1, temos que S = a1 . Agora, veja
apostou 1 centavo; na segunda vez, apostou 2 centavos, na terceira q 1
vez, apostou 4 centavos e assim por diante, apostando em cada vez o 1 1
dobro do que havia apostado na vez anterior. que I uma soma de P.G. de 1o termo igual a e razo igual a :
Nas 20 primeiras vezes, ela perdeu. Na 21a vez, ela ganhou. Sendo T a a1 q
quantidade total por ela desembolsada e Q a quantidade recebida na 21a 1
1
jogada, determine uma relao entre T e Q: 1 1 1 1 1 qn
I= + + + ... + n 1 =
a1 a1q a1q 2 a1q a1 1 1
Soluo q
Veja que as apostas dele crescem como P.G. de razo 2, assim, o total
desembolsado :
T = 1 + 2 + 4 + 8 + ... + 220. Como T uma soma de P.G.:
1 q n 1 1 q n 1
q n 1 221 1 21 I= =
T = a1 = = 2 1. Na 21a jogada ele recebe o dobro do a1q n 1 q 1 an q 1
q 1 21
que investiu: Q = 2.220 = 221, logo: Q = T + 1. S
n
S
= a1an , ento, = ( a1an ) .
n
Portanto, temos que
08 Em um paraleleppedo retngulo a soma das medidas de todas as I I

arestas 52 e a diagonal mede 91. Se as medidas das arestas esto S
n

Como sabido que Pn2 = ( a1an ) , segue que P 2 = .


n
em progresso geomtrica, ento o seu volume :
I
Soluo 10 Seja Q um quadrado de lado 4. A partir dos pontos mdios de Q,
a construmos o quadrado Q1. Prosseguindo da mesma forma, a partir
Sejam os lados 1 ,a1,a1q temos: dos pontos mdios dos lados do quadrado Qi, construmos o quadrado
q
Qi+1. Determine a soma das reas de todos os quadrados citados no
1 52 1 enunciado.
a1 + 1 + q = = 13 e a12 2 + 1 + q 2 = 91 .
q 4 q
Soluo
1 1 1
Fazendo t = q + , tm-se: t 2 = q 2 + 2 + 2 q 2 + 2 = t 2 2 . fcil ver que a rea de cada quadrado a metade da rea do
substituindo: q q q 1
quadrado anterior. Portanto, temos uma P.G. de razo igual a . Como
( )
a12 t 2 1 = 91
. Dividindo a1(t 1) = 7 a1t = a1 + 7, a1t = 13 a1
2
16
a1 ( t + 1) = 13  o 1 quadrado tem rea 16, a soma das reas = 32.
o
1
1
2
= a1 + 7 a1 = 3, logo: V 3
= a=
1 27 u.v.

5. Matemtica financeira
5.1. Porcentagem Obs.: Um aumento de x% seguido de uma reduo de x% no traz o
valor de volta ao inicial.
As fraes (ou razes) que possuem denominadores iguais a 100 Isto ocorre porque a reduo feita sobre um valor maior que o inicial.
so conhecidas por razes centesimais e podem ser representadas pelo
smbolo %. Ex.: Se aumentarmos o preo de uma camisa em 10% e depois
O smbolo % lido como por cento. 5% l-se 5 por cento. reduzirmos em 10%, voltamos ao valor original? Vejamos:
25% l-se 25 por cento. Seja x o preo da camisa, aps o aumento de 10% ficamos com 1,1x.
Para se calcular uma porcentagem de um dado valor, basta multiplicar Ao reduzirmos esse valor em 10% ficamos com 0,9 1,1. x = 0,99x, que
a razo pelo valor desejado. corresponde a 99% do valor original.

30
Ex.: 30% de 1500 1500 = 450 5.2. Juros
100
Juro a remunerao cobrada pelo emprstimo de dinheiro.
Aumento Percentual (ou reduo): Dado um valor x (o preo de expresso como um percentual sobre o valor emprestado (taxa de juro)
uma camisa, por exemplo), para calcular o valor aps um aumento de e pode ser calculado de duas formas: juros simples ou juros compostos.
i
i%, basta multiplicarmos x por 1 + . No caso de reduo deve-se
i 100
multiplicar por 1 .
100

AFA-EFOMM 199
Matemtica III Assunto 1

O juro pode ser compreendido como uma espcie de aluguel sobre 5.2.2 Juros compostos
o dinheiro. A taxa seria uma compensao paga pelo tomador do No regime de juros compostos a taxa de juros aplicada sempre sobre
emprstimo para ter o direito de usar o dinheiro at o dia do pagamento. o montante atual, ou seja, temos juros sobre juros.
Como a taxa de juros feita sobre o ltimo valor, estamos sempre
5.2.1. Juros simples i
No regime de juros simples, a taxa de juros sempre aplicada sobre pegando o capital atual e multiplicando por 1 + de modo que o
100
o valor inicial, ou seja, no leva em considerao o capital acumulado. i
Como a taxa de juros ocorre sempre sobre o mesmo valor, estamos montante acumulado cresce como P.G. de razo 1 + .
100
sempre somando uma constante, ou seja, o montante acumulado cresce Logo, para calcularmos o montante, podemos usar a seguinte frmula:
como P.A. Deste modo, para o clculo do montante, podemos usar a
seguinte frmula: n
i
M = C 1+
100
ni
M = C 1+
100
Obs.: Na verdade, tanto nos juros simples como nos juros compostos,
Em que M o montante, C o capital inicial, i a taxa de juros e n o os juros (J) adquiridos (ou cobrados) a diferena entre o montante (M)
perodo de aplicao. e o capital inicial (C)

J=MC

EXERCCIOS RESOLVIDOS
01 Em uma turma de Cincia da Computao formada de 40 rapazes Soluo
e 40 moas, tem-se a seguinte estatstica: 20% dos rapazes so Basta lembrar que aumentos e redues percentuais so feitos
fumantes; 30% das moas so fumantes. Logo, a porcentagem dos atravs de multiplicao (no soma). Para aumentar 10% devemos
que no fumam na turma de: multiplicar por 1,10 e para reduzir 10% multiplicar por 0,90, deste modo:
p
Soluo: p f = 1,1 0 ,9 pi f = 0 ,99
pi
20% dos rapazes = 0,2 40 = 8 fumantes; 30% das moas = 04 (CN 99) As vendas de uma empresa foram, em 1998, 60%
0,3.40 = 12 fumantes. superiores s vendas de 1997. Em relao a 1998, as vendas de 1997
Logo o total de fumantes 20 e o de no fumantes 60 que corresponde foram inferiores em:
a 60/80 = 75%.
(A) 62,5%. (D) 44,5%.
02 Aps se fazer uma promoo em um clube de dana, o nmero de (B) 60%. (E) 37,5%.
frequentadores do sexo masculino aumentou de 60 para 84 e, apesar (C) 57,5%.
disso, o percentual da participao masculina passou de 30% para
24%. Considerando-se essas informaes, correto afirmar que o Soluo:
nmero de mulheres que frequentam esse clube, aps a promoo, Letra E.
teve um aumento de: Se x representa as vendas de 1998 e y as vendas de 1997, tm-se:
1
x = 1,6 y y = x = 0 ,625 x , assim as vendas foram inferiores em
Soluo: 1,6
Seja x o total de frequentadores do clube antes da promoo, 1 0,625 = 37,5%
tm-se:
0,3x = 60 donde x = 200. Assim, o nmero de frequentadores do 05 (UF-PI) Uma quantia foi aplicada a juros simples de 6% ao ms,
sexo feminino era 140. durante 5 meses e, em seguida, o montante foi aplicado durante mais
Sendo y o nmero de frequentadores do clube aps a promoo, tm-se 5 meses, a juros simples de 4% ao ms. No final dos 10 meses, o novo
0,24y = 84 donde y = 350. Assim, o nmero de frequentadores do montante foi de R$ 234,00. Qual o valor da quantia aplicada inicialmente?
sexo feminino passou a ser 266.
Como 266/140 = 1,9 o aumento foi de 90%. Soluo:
Sendo x a quantia inicial aplicada, aps os cinco meses o montante era de:
03 (VUNESP) Uma mercadoria teve seu preo acrescido de 10%. x(1 + 0,06 5) = 1,3x. Tendo aplicado esse montante com taxa de juros
Tempos depois, esse novo preo sofreu um desconto de 10%. de 4% a.m. durante 10 meses, o montante foi de 1,3x(1 + 0,04 10) =
Denotando-se por pi o preo inicial e por pf o preo final da mercadoria, 1,3 1,4x = 234 donde x = R$ 128,57 (aproximadamente).
p
determine f : ni
pi
Obs.: Usamos a frmula dos juros simples: M = C 1 +
100

200 Vol. 1
Sequncias

06 (AFA 03) Em julho de 2001, uma pessoa gastava 27,3% do seu dvida foi sendo rolada, com capitalizao anual dos juros. Qual dos
salrio com o pagamento da prestao da casa prpria. Em 2002, valores abaixo est mais prximo do valor da dvida em 1989?
houve dois reajustes no seu salrio: 40% em janeiro e 30% em junho. Adote (1,09)8 2.
Se, em julho de 2002, o aumento daquela prestao foi de 130%, que
porcentagem de seu salrio a pessoa passou a gastar? (A) 14 milhes de dlares. (D) 80 bilhes de dlares.
(B) 500 milhes de dlares. (E) 1 trilho de dlares.
Soluo: (C) 1 bilho de dlares.
Sendo x o valor inicial do salrio e y o valor da inicial da prestao,
sabe-se que y = 0,273x. Soluo:
Aps os dois reajustes: 1,4 1,3x = 1,82x. Letra E.
J a prestao: 2,3y = 2,3 0,273x = 0,6279x. De 1829 at 1989 passaram-se 160 anos. Usando a frmula de
0 ,6279 x juros compostos:
Desse modo a prestao representa: = 0= ,345 34 ,5% do
salrio. 1,82 x
M = C(1 + i)n = 106(1 + 0,09)160 = 106 (1,098)20 = 106 220 =
(1024)2 106 > 1012
07 (FUVEST 90) Um pas contraiu em 1829 um emprstimo de 1
milho de dlares, para pagar em cem anos taxa de juros de 9% ao Logo, a dvida passa de 1 trilho de dlares.
ano. Por problemas de balana comercial, nada foi pago at hoje, e a

6. Somatrios
Ex.: (1, 3, 6, 10, 15, 21) P.A. de 2a ordem, pois suas diferenas so:
O smbolo de somatrio serve para representar uma soma de parcelas (2, 3, 4, 5, 6) P.A. no estacionria.
com mesma lei de formao, podendo ser uma soma finita ou infinita: De modo geral, uma P.A. de ordem k(k > 2) uma sequncia na qual
as diferenas entre cada termo e o termo anterior formam uma P.A. de
ordem k 1.
n

a i =1
i = a1 + a2 + ...+ an
Ex.: (0, 0, 6, 24, 60, 120, 210, ...) P.A. de 3a Ordem, pois suas diferenas
valem:
6.1 Propriedades dos somatrios (0, 6, 18, 36, 60, 90, ...) P.A. de 2a Ordem, uma vez que olhando para
as diferenas temos:
Existem duas propriedades bsicas de somatrios: (6, 12, 18, 24, 30, ...) P.A. no estacionria.

I. O somatrio da soma a soma dos somatrios:


7.1 Teoremas importantes
n n n

( a + b ) = a + b
i =1
i i
i =1
i
i =1
i Teorema 1:
Se (an) uma P.A. no estacionria ento an um polinmio em n de
grau um e, reciprocamente, todo polinmio em n de grau um termo geral
Dem.: de alguma P.A. no estacionria.
n

(a + b ) = (a + b ) + (a
i =1
i i 1 1 2 + b2 ) + ... + ( an + bn ) =
Teorema 2:
n n
Seja Sn a soma dos n primeiros termos de uma sequncia (an). Se
= ( a1 + a2 + ... + an ) + ( b1 + b2 + ... + bn ) = ai + bi
i =1 i =1
(an) uma P.A. no estacionria, ento Sn um polinmio em n de grau
dois desprovido de termo independente e reciprocamente, todo polinmio
de grau dois desprovido de termo independente, o valor da soma dos n
II. Podemos colocar uma constante multiplicando para fora do somatrio
primeiros termos de alguma P.A. no estacionria.
n n

a i = ai Teorema 3:
i =1 i =1 Se (an) uma P.A. de ordem k ento an um polinmio em n de grau
k e, reciprocamente, todo polinmio em n de grau k termo geral de
Dem.: Basta pensar que colocamos em evidncia. alguma P.A. de ordem k.

Teorema 4:
7. P.A. de ordem superior A soma dos n primeiros termos de uma P.A. de ordem k um polinmio
em n de grau k + 1 e termo independente nulo, reciprocamente, se Sn a
Dizemos que uma sequncia (a1, a2, a3, ..., an) forma uma P.A. de soma dos n primeiros termos de uma sequncia e Sn um polinmio de
2a ordem se as diferenas ai formam uma P.A. no estacionria. grau k + 1 em n, ento a sequncia forma uma P.A. de ordem k.

AFA-EFOMM 201
Matemtica III Assunto 1

EXERCCIOS RESOLVIDOS
01 Obter o termo geral da sequncia (1, 3, 7, 13, 21, ...) 1 3 5 1 5
Assim, Sn = n + n S20 = 203 + 20 = 2700
3 3 3 3
Soluo
Olhando para a sequncia formada pelas diferenas de termos 03 Determine o valor da soma S = 1 2 + 2 3 + 3 4 + ... + 99
consecutivos (2, 4, 6, 8, ...) que uma PA no estacionria, donde a 100.
sequncia original uma PA de 2a ordem.
Como toda PA de 2a ordem tem termo geral do 2 grau: Soluo (1):
an = an2 + bn + c. Fazendo n = 1, 2 e 3: Veja que estamos somando termos da forma k(k + 1) = k2 + k,
polinmio do 2 grau. Assim, estamos somando termos de uma PA
a + b + c = 1(1) de ordem 2, donde a soma um polinmio de grau 3 sem termo
( 2) (1): 3 a + b = 2 independente.
4 a + 2 b + c = 3 ( 2 ) a = 1, b = 1, c = 1
9 a + 3 b + c = 7 ( 3 ) ( 3 ) ( 2): 5a + b = 4 Sn = an3 + bn2 + cn. Fazendo n = 1, 2 e 3:
a + b + c = 2(1)
( 2) 2 (1): 6 a + 2b = 4 
8 a + 4 b + 2c = 2 + 6 = 8 ( 2 )
Assim, an = n2 n + 1. 27 a + 9 b + 3c = 2 + 6 + 12 = 20 ( 3 ) ( 3 ) 3 (1): 24 a + 6 b = 14

02 Determine a soma dos 20 primeiros termos da sequncia (2, 4, 8, 1 2
a = ,b = 1,c =
14, 22 ...), que uma PA de 2a ordem. 3 3

Soluo:
1 3 2 1 2
A diferena dos termos consecutivos forma a sequncia 2, 4, 6, 8, Assim,Sn = n + n2 + n S99 = 993 + 992 + 99 = 333300.
(...), que uma PA no estacionria, donde a sequncia original 3 3 3 3
uma PA de 2a ordem. Soluo (2):
Assim, Sn (soma dos n primeiros termos) um polinmio do 3o grau Podemos utilizar as conhecidas somas:
sem termo independente. n
n ( n + 1) ( 2 n + 1) n
n ( n + 1)
k 2
=
6
e k =
2
Sn = an3 + bn2 + cn. Fazendo n = 1, 2 e 3. k =1 k =1

junto s propriedades de somatrio:


a+ b+c=2 (1)
( 2) 2 (1): 6 a + 2b = 2 99 99 99
. .
99100199 .
99100
8 a + 4 b + 2c = 2 + 4 = 6 ( 2) S = k ( k + 1) = k 2 + k = + =
27 a + 9 b + 3c = 2 + 4 + 8 = 14 ( 3 ) 3 (1) : 24 a + 6 b = 8 6 2
(3) k =1 k =1 k =1

= 328.350 + 4.950 = 333.300


a = 1/3, b = 0, c = 5/3

EXERCCIOS NVEL 1

01 (AFA-90) Quantos nmeros no mltiplos de 11 h no conjunto {x 5n 4


04 (AFA 88) O termo geral de uma progresso aritmtica .A
| 51 x 1.500}? 3
soma dos n primeiros termos da progresso vale:
(A) 1210. (D) 1412.
(B) 1318. (E) n.r.a. n2 5 n
(A) .
(C) 1406. 3
2
(B) 5 n 3 n .
02 (AFA-94) O nmero formado por 3 algarismos em progresso 6
aritmtica com soma 15 e que, adicionado a 396, d como resultado ele 2
5 n 16 n
mesmo escrito em ordem inversa : (C) .
3
(A) par. 10 n2 8 n
(B) primo. (D) .
6
(C) mltiplo de 7.
(D) divisvel por 13. 05 (ITA-96) As dimenses x, y e z de um paraleleppedo retngulo esto
em progresso aritmtica. Sabendo que a soma dessas medidas igual
03 (AFA-88) A soma dos 15 primeiros termos da sequncia (2, 1, 4, a 33 cm e que a rea total do paraleleppedo igual a 694 cm2, ento o
7,...) vale: volume deste paraleleppedo, em cm3, igual a:

(A) 260. (A) 1200. (D) 728.


(B) 285. (B) 936. (E) 834.
(C) 330. (C) 1155.
(D) 345.

202 Vol. 1
Sequncias

06 (ITA -88) Suponha que os nmeros 2, x, y e 1.458 esto, nesta ordem, 17 Em uma sequncia (an), a soma dos n primeiros termos , para todo
em progresso geomtrica. Desse modo o valor de x + y : n, Sn = n2 + 2n . Determine an.

(A) 90. (D) 360. 18 (Fuvest-10) Os nmeros a1, a2, a3 formam uma progresso aritmtica
(B) 100. (E) 1.460. de razo r de modo que a1+ 3, a2 3, a3 3, estejam em progresso
(C) 180. geomtrica. Dado ainda que a1 > 0 e a2 = 2, conclui-se que r igual a:

07 (Fuvest) 500 moedas so distribudas entre trs pessoas, Antnio, 3


Pedro e Cristian, sentadas em crculo, da seguinte maneira: Antnio recebe (A) 3 + 3 . (D) 3 .
uma moeda, Pedro recebe duas, Cristian trs, Antnio quatro, Pedro cinco, 2
3
Cristian seis, Antnio sete, e assim por diante, at no haver mais moedas (B) 3 + 3 3 .
. (E)
2
suficientes para continuar o processo.
Quantas moedas sobraram ao final do processo? (C) 3 + 3 .
4
1 1 1
08 Sabendo que (a, b, c) e , , esto em progresso aritmtica, 19 A espessura de uma folha de estanho 0,1 mm. Forma-se uma pilha
b c d
de folhas colocando-se uma folha na primeira vez e, em cada uma das
demonstre que 2ad = c(a + c). vezes seguintes, tantas quantas j houverem sido colocadas anteriormente.
Depois de 33 dessas operaes, a altura da pilha ser, aproximadamente:
09 Dada uma progresso aritmtica na qual o primeiro termo 12 e a
razo 4, qual o valor de n, se a mdia aritmtica dos n primeiros termos (A) a altura de um poste de luz.
dessa progresso 50? (B) a altura de um prdio de 40 andares.
(C) o comprimento da praia de Copacabana.
10 (Fuvest) Em uma P.A. de termos positivos, os trs primeiros termos (D) a distncia Rio-So Paulo.
so: (1 a, a, 11 a ). Qual o quarto termo desta P.A.?
20 Determine trs nmeros em progresso geomtrica, sabendo que a
(A) 2. (D) 5. sua soma igual a 52 e que o maior deles excede em 20 unidades a soma
(B) 3. (E) 6. dos outros dois.
(C) 4.
21 Determine as geratrizes das dzimas peridicas:
11 Determine cinco nmeros em progresso aritmtica sabendo que sua
soma 40 e a soma dos inversos dos extremos, 1/3. a. 0,141414141...
b. 0,345454545...
12 (FGV) Quantos termos devemos tomar na P.A. 7, 3, ... a fim de que c. 0,999999999...
a soma valha 3.150? d. 1,030503050...
e. 1,711111111...
13 (FFCL USP-65) A soma de quatro termos consecutivos de uma P.A. f. 1,488888888...
6 e o produto do primeiro deles pelo quarto 54 . Determine esses
termos. 25 a2 2 2
22 (IME) Em uma P.G., tem-se a1 = e a4 = 2( a + 1) , com
4( a2 + 1) 5a
14 (Olimpada de Matemtica de Natal 95) Os inteiros de 1 a 1.000 a > 0.
so escritos ordenadamente em torno de um crculo. Partindo de 1, cada
dcimo quinto nmero riscado (isto , so riscados 1, 16, 31,...). O a. Quais os valores de a para os quais a P.G. decrescente?
processo continua at se atingir um nmero j previamente riscado. 1
b. Qual o limite da soma dos termos para q = a ?
Quantos nmeros sobraro sem riscos? 5
23 Uma pessoa pagou 20% de uma dvida. Se R$ 4.368,00 correspondem
(A) 800.
a 35% do restante a ser pago, determine a dvida total.
(B) 934.
(C) 933.
24 Os capitais de R$ 20.000,00, R$ 30.000,00 e R$ 50.000,00 foram
(D) 862.
aplicados mesma taxa de juros simples mensal durante 4, 3 e 2
(E) Nenhuma correta.
meses, respectivamente. Obtenha o prazo mdio de aplicao desses
capitais, ou seja, o tempo por que seria necessrio aplicar o capital total
15 Em uma progresso aritmtica com um nmero par de termos, a soma
(R$ 100.000,00) mesma taxa anterior para obtermos o mesmo retorno.
dos termos de ordem mpar 70 e a soma dos termos de ordem par 85.
A soma dos extremos 31. Forme a progresso.
25 (Enem-2010) Em maro de 2010, o Conselho Nacional de
Desenvolvimento Cientfico e Tecnolgico (CNPq) reajustou os valores de
16 Devemos colocar 500 bolas formando um tringulo, com uma bola
bolsas de estudo concedidas a alunos de iniciao cientfica, que passaram
na primeira linha, duas na segunda linha, trs na terceira etc.
a receber R$ 360,00 mensais, um aumento de 20% com relao ao que
era pago at ento. O rgo concedia 29 mil bolsas de iniciao cientfica
a. Quantas bolas sobraro?
at 2009, e esse nmero aumentou em 48% em 2010.
b. Quantas linhas haver?
(O Globo. 11/3/2010.)

AFA-EFOMM 203
Matemtica III Assunto 1

Caso o CNPq decidisse no aumentar o valor dos pagamentos aos bolsistas, (A) 12%. (D) 20%.
utilizando o montante destinado a tal aumento para incrementar ainda mais o (B) 18%. (E) 16%.
nmero de bolsas de iniciao cientfica no Pas, quantas bolsas a mais que (C) 14%.
em 2009, aproximadamente, poderiam ser oferecidas em 2010?
33 (UFMG -2010) O preo de venda de determinado produto tem a
(A) 5,8 mil. (D) 51,5 mil. seguinte composio: 60% referentes ao custo, 10% referentes ao lucro
(B) 13,9 mil. (E) 94,4 mil. e 30% referentes a impostos. Em decorrncia da crise econmica, houve
(C) 22,5 mil. um aumento de 10% no custo desse produto, porm, ao mesmo tempo,
ocorreu uma reduo de 20% no valor dos impostos. Para aumentar as
26 (UFPE) Segundo pesquisa recente, 7% da populao brasileira vendas do produto, o fabricante decidiu, ento, reduzir seu lucro metade.
analfabeta, e 64% da populao de analfabetos do sexo masculino. Qual correto afirmar, portanto, que, depois de todas essas alteraes, o preo
percentual da populao brasileira formado por analfabetos do sexo feminino? do produto sofreu reduo de:
(A) 2,52%. (D) 4,48%.
(A) 5%. (D) 25%.
(B) 5,20%. (E) 3,20%.
(B) 19%. (E) 11%.
(C) 3,60%.
(C) 10%.
27 (PUC CAMP-05) Um trabalhador comprou uma bicicleta, conseguindo
34 (PUC SP-97) Um veculo de transporte de passageiros tem seu
um abatimento de 10% sobre o preo marcado. Do valor a ser pago, 40%
valor comercial depreciado linearmente, isto , seu valor comercial sofre
foi dado como entrada e o restante foi pago em 5 parcelas sem juros, no
desvalorizao constante por ano. Veja a figura seguinte.
valor de R$ 41,04 cada. O valor do abatimento obtido foi:

(A) R$ 32,00. (D) R$ 40,00. valor (R$)


(B) R$ 35,00. (E) R$ 42,00.
(C) R$ 38,00.

28 (Fuvest) Em certa populao, 18% das pessoas so gordas, 30% dos 0 20 tempo
homens so gordos e 10% das mulheres so gordas. Qual a porcentagem
de homens na populao?
Esse veculo foi vendido pelo seu primeiro dono, aps 5 anos de uso, por R$
24.000,00. Sabendo-se que o valor comercial do veculo atinge seu valor
29 (ESPM) Uma pessoa fez um investimento em aes. No primeiro
mnimo aps 20 anos de uso, e que esse valor mnimo corresponde a 20%
semestre, ela perdeu 30% do capital aplicado e no segundo semestre ela
do valor que tinha quando era novo, ento esse valor mnimo , em reais:
recuperou 60% do que havia perdido. Em relao ao investimento inicial,
seu prejuzo nesses dois semestres foi de:
(A) menor que 4.500.
(A) 22%. (D) 16%. (B) maior que 4.500 e menor que 7.000.
(B) 24%. (E) 18%. (C) mltiplo de 7.500.
(C) 12%. (D) um nmero que NO divide 12.000.

30 (Enem-11) Uma pessoa aplicou certa quantia em aes. No primeiro EXERCCIOS NVEL 2
ms, ela perdeu 30% do total do investimento e, no segundo ms,
recuperou 20% do que havia perdido. Depois desses dois meses, resolveu
01 (IME) Seja uma progresso aritmtica de primeiro termo a10 e ltimo
tirar o montante de R$ 3.800,00 gerado pela aplicao. 1
termo a10 a1 0. Seja a progresso aritmtica de primeiro termo b1 =
A quantia inicial que essa pessoa aplicou em aes corresponde ao valor de: a1
1
e ltimo termo b10 = . possvel determinar a5 em funo de a1 e a10?
(A) R$ 4.222,22. (D) R$ 13.300,00. a10 b6
(B) R$ 4.523,80. (E) R$ 17.100,00.
(C) R$ 5.000,00. 02 (Uerj-05) O quadriculado abaixo deve ser preenchido por inteiros
positivos de forma que cada linha e cada coluna formem uma progresso
31 (UFCE) Jos e Joo possuem uma empresa cujo capital de aritmtica. Qual deve ser o nmero na posio *?
R$ 150.000,00. Jos tem 40% de participao na sociedade e deseja
aumentar a sua participao para 55%. Se Joo no deseja alterar o valor, *
em reais, de sua participao, o valor que Jos deve empregar na empresa : 74
(A) R$ 110.000,00. (D) R$ 50.000,00. 186
(B) R$ 90.000,00. (E) R$ 82.500,00. 103
(C) R$ 170.000,00.
0
32 (Uerj-08) Joo abriu uma caderneta de poupana e, em 10 de janeiro
de 2006, depositou R$ 500,00 a uma taxa de juros, nesse ano, de 20%. 03 Calcule a soma de todos os inteiros compreendidos entre 100 e 400
Em 10 de janeiro de 2007, depositou mais R$ 1.000,00. Para que Joo que no so divisveis nem por 2, nem por 3, nem por 5.
tenha, nessa poupana, em 10 de janeiro de 2008, um montante de
R$ 1.824,00, a taxa de juros do segundo ano deve corresponder a:

204 Vol. 1
Sequncias

04 (ITA-93) Em uma progresso aritmtica com 2n + 1 termos, a soma 11 (FGV) Uma empresa desconta do salrio anual de seus funcionrios
dos n primeiros igual a 50 e a soma dos n ltimos 140. Sabendo que certa porcentagem para um plano de previdncia privada. O desconto
a razo desta progresso um nmero inteiro entre 2 e 13, qual o ltimo de p% sobre R$ 28.000,00 de renda anual, mais (p + 2)% sobre
nmero? o montante anual do salrio que excede R$ 28.000,00. Joo teve
desconto total de (p + 0,25)% do seu salrio anual para o plano de
05 Calcule o maior valor possvel para a razo de uma P.A. que admita previdncia privada. O salrio anual de Joo, em reais, sem o desconto
os nmeros 32, 227 e 942 como termos da progresso. do plano de previdncia :

06 (Fuvest-09) A soma dos cinco termos de uma P.G., de razo negativa, (A) R$ 28.000,00.
1 (B) R$ 42.000,00.
. Alm disso, a diferena entre o stimo termo e o segundo termo da
2 (C) R$ 32.000,00.
(D) R$ 56.000,00.
P.G. igual a 3. Nessas condies, determine:
(E) R$ 35.000,00.
a. a razo da P.G.;
b. a soma dos trs primeiros termos da P.G. 121
12 Qual a P.G. de cinco termos cuja soma e o produto 243?
3
07 Larga-se uma bola de uma altura de 5 m. Aps cada choque com o
13 As medidas dos lados de um tringulo so expressas por nmeros
solo, ela recupera apenas 4/9 da altura anterior. Determine:
inteiros em P.G. e seu produto 1.728. Calcule as medidas dos lados:
a. a distncia total percorrida pela bola.
14 Calcule o valor da soma Sn = 1.3 + 2.4 + 3.5 +...+ n(n+2).
b. o tempo gasto pela bola at parar.
15 Os nmeros (4, 6, 13, 27, 50, 84) esto em uma P.A. de ordem k.
08 (AIME-89) Um determinado dgito d tal que 0,d25d25d25d25...=
Determine o 30o termo.
n , em que n um inteiro positivo. Determine n.
810 16 Seja A = {1,2, ... , p}. Calcule o valor da soma dos produtos que se
k
podem obter usando como fatores dois elementos distintos de A.
09 Calcule 1
k .
2
k =1
17 Determine o valor da seguinte expresso: 1 100 + 2 99 + 3 98+
10 (UFMG-09) No perodo de um ano, certa aplicao financeira obteve +100 1.
um rendimento de 26%. No mesmo perodo, porm, ocorreu uma inflao
de 20%. Ento, correto afirmar que o rendimento efetivo da referida
aplicao foi de:

(A) 3%. (D) 4%.


(B) 6%. (E) 5,2%.
(C) 5%.

RASCUNHO

AFA-EFOMM 205
Anlise combinatria A ssunto
2
Matemtica III

1. Introduo III. Caso no problema anterior os algarismos fossem todos distintos?

Talvez voc j tenha se perguntado quantos so os resultados diferentes Soluo


em uma loteria como a mega-sena ou quanto tempo seria necessrio para __ __ __ __ 9 9 8 7 = 4536 possibilidades
acertar uma senha caso fosse tentar todas as possibilidades. 9 9 8 7
Com o intuito de determinar o nmero de maneiras de ocorrer um
dado evento e resolver problemas desse tipo, criou-se um ramo na Repare que comeamos a escrever o nmero da esquerda para a
matemtica conhecido como anlise combinatria. Sua ideia principal direita, assim, na primeira casa temos nove possibilidade pois o zero
agrupar problemas com ideias comuns, determinando assim os conceitos no entra, na casa seguinte continuamos com nove, uma vez que s no
principais necessrios para resoluo dos mesmos. podemos utilizar o algarismo da primeira casa (o zero pode entrar) e depois
Neste assunto veremos os conceitos principais de combinatria, que vamos sempre perdendo um algarismo.
so o Princpio Fundamental da Contagem (PFC) e o Princpio Aditivo, O que ocorreria se comessemos da direita para a esquerda no
que basicamente so as ferramentas para o desenvolvimento de toda problema anterior?
teoria. Alm disso, veremos as ideias principais de combinatria, como
as permutaes, os arranjos e as combinaes. __ __ __ __
Encerrando este assunto, encontram-se tpicos menos tradicionais ? 8 9 10
como a Incluso-Excluso, os Lemas de Kaplansky e a Permutao Catica.
Veja que, nesse caso, o nmero de possibilidades para o algarismo
Obs.: Este assunto possui diversos exemplos diferentes, uma vez que das unidades de milhar no est definido, pois dependeria se o zero foi
boa parte do aprendizado em Combinatria est associado ao nmero de utilizado anteriormente ou no. Se o zero tiver sido utilizado teremos sete
questes j vistas anteriormente. possibilidades, caso contrrio, teremos seis.
Isso ocorre uma vez que esta a casa com maior restrio (o zero
no entra), logo importante que toda deciso com maior restrio seja
2. Princpio fundamental da contagem tomada primeiro.
(PFC)
Considere o seguinte problema: Joo decide sair de casa, abre ento 3. Princpio aditivo
seu armrio e percebe que possui trs calas e cinco blusas. De quantos
modos diferentes Joo pode se vestir? Para resolver o tipo de problema que ocorreu no ltimo exemplo,
quando comeamos pelo algarismo das unidades, podemos usar o
Basta ver que para cada opo de cala Joo tem cinco opes de
princpio aditivo:
blusa. Como Joo pode escolher trs calas diferentes, temos 3 5 =
15 possibilidades.
Dados dois conjuntos disjuntos A e B, temos: n(AB) = n(A) + n(B),
De modo geral, se podemos tomar uma deciso de m maneiras e, se onde n(X) denota o nmero de elementos de X.
uma vez tomada essa deciso, podemos tomar outra de n maneiras, ento
o nmero de maneiras de tomar ambas as decises mn. Vejamos ento o exemplo anterior!
Ex.: Para determinar o nmero de possibilidades nas unidades de milhar,
I. Srgio deve viajar de uma cidade A para uma cidade B. Para isso, comeando pelo algarismo mais a direita, devemos saber se o zero foi
ele possui oito opes distintas de estradas. Sabendo que ele deve utilizado ou no anteriormente, assim temos dois casos:
ir de A para B em uma estrada e voltar por outra, de quantos modos
diferentes Srgio pode fazer o seu trajeto de ida e volta? 1o Caso - se utilizarmos o zero antes.
Soluo
Primeiro, temos que definir aonde aparece o zero: 3 possibilidades,
Temos oito opes para ir e, uma vez escolhida essa opo, temos Segundo, completar o nmero:
sete opes para voltar, logo: 8 7 = 56
__ 0 __ __ 7 1 8 9 = 504 possibilidades.
II. Quantos nmeros naturais de 4 algarismos existem no nosso sistema 7 1 8 9
de numerao?
Logo nesse caso temos: 3 504 = 1512 nmeros.
Soluo
2o Caso sem utilizar o zero.
Uma ideia muito comum em combinatria representar cada deciso
a ser tomada por um tracinho, colocando o nmero de possibilidades de Basta escolher os algarismos do nmero.
realiz-la abaixo de cada trao.
__ __ __ __ 6 7 8 9 = 3024 possibilidades.
__ __ __ __ 9 10 10 10 = 9000 possibilidades 6 7 8 9
9 10 10 10

206 Vol. 1
Anlise combinatria

Como os casos so disjuntos: 1512 + 3024 = 4536 nmeros, mesmo 2o Caso 1oQ 3oQ
resultado achado anteriormente.

Ex.:
I. Quantos nmeros pares de 5 algarismos distintos podem ser formados? 1
( 1)( 2)2
Soluo
2 2
Repare que como o nmero deve ser par, ele deve terminar com 0, 2,
4, 6 ou 8. Como o fato de usar ou no o zero influencia na primeira casa
(da esquerda para a direita), devemos abrir em casos:
Somando: ( 1) ( 2)2 + ( 1)2 = ( 1)(72 3 + 3)
1 Caso - terminando em zero.
o
4. Permutaes simples
0 9 8 7 6 1= 3024 possibilidades.
__ __ __ __ __
9 8 7 6 1 Muitos problemas de combinatria esto associados a determinar o
nmero de ordenaes que se podem fazer dados n objetos. Chamamos
2o Caso - no terminando em zero. cada ordenao possvel de uma permutao simples e representamos o
nmero de permutaes simples por Pn.
__ __ __ __ __ 8 8 7 6 4= 10752 possibilidades.
8 8 7 6 4 Veja que na verdade isso uma aplicao direta do PFC, uma vez que
basta determinar que elemento ocupa cada posio:
Somando: 3024 + 10752 = 13776 nmeros.
... Pn = n .( n 1)... .1 := n ! Pn = n!
II. (Morgado) A figura abaixo mostra um mapa com quatro pases: n n 1 1

(chamado tambm de fatorial n)

Obs.: 0! = 1 por definio.

Ex.:
I. Dada uma palavra qualquer, chamamos de anagrama qualquer
permutao simples de suas letras, mesmo que essa permutao
no tenha significado.

Sabendo disso, determine o nmero de anagramas da palavra


De quantos modos esse mapa pode ser colorido (cada pas com uma CADERNO?
cor, pases com uma linha fronteira comum no podem ter a mesma cor)
se dispomos de cores diferentes? Soluo
Basta permutar suas letras, como existem 7 letras distintas:
Soluo P7 = 7! = 5040.
Faremos meno a cada pas como um quadrante do ciclo
trigonomtrico. II. Cinco casais desejam ocupar uma escada com cinco degraus para
Se comearmos a pintar o 1oQ, depois o 2oQ e assim por diante, o tirar uma foto. Sabendo que cada degrau deve ser ocupado por
nmero de possibilidades para o 4oQ no fica definido, uma vez que o 1oQ exatamente um casal, determine o nmero de maneiras desses casais
e o 3oQ podem ser pintados da mesma cor ou no. Assim, devemos abrir se organizarem para essa foto:
em casos:
Soluo
1o Caso 1oQ = 3oQ Primeiro devemos permutar os casais nos degraus: P5 = 5! = 120.
Segundo devemos decidir em cada casal quem fica direita: 25 = 32.
Pelo PFC: 120 32 = 3840 possibilidades.

1 III. Determine o nmero de anagramas da palavra HORTEL que possuem
todas as vogais juntas?
( 1) 2

1 1 Soluo
Sempre que um problema pedir para que alguns objetos fiquem juntos
podemos trat-los como um nico objeto, j que podem ser vistos como
uma nica caixa (bizu da caixinha).

AFA-EFOMM 207
Matemtica III Assunto 2

Assim, podemos considerar que essa palavra possui apenas cinco 6.1 Permutaes completas (com repetio)
letras, sendo estas, as quatro consoantes e a caixinha com as vogais.
Primeiro: Permutando essas letras P5 = 5! = 120. O que ocorre quando queremos determinar o nmero de anagramas de
Segundo: Devemos permutar as vogais dentro da caixa: P3 = 3! = 6. uma palavra com letras repetidas? Por exemplo, quantos so os anagramas
Pelo PFC: 120 6 = 720. da palavra CASA?
Considerando os As como letras distintas, temos 4! = 24
IV. Considerando a palavra do exemplo anterior, determine o nmero de
permutaes. Porm como os As so iguais, cada permutao contada
anagramas que possuem H e R separados:
duas vezes (CA1SA2 = CA2SA1), assim tm-se 12 anagramas.
Soluo De modo geral, se uma letra aparece n vezes, basta pegar o total de
Outra ideia importante em combinatria olhar para o complementar permutaes e dividir por n!, uma vez que fixadas as demais letras temos
de um conjunto em relao ao total, uma vez que em muitos problemas n! maneiras de permutar essas letras iguais.
mais fcil fazer o contrrio do que pedido na questo. Chamamos de permutao completa cada ordenao de n objetos com
Neste problema, por exemplo, queremos saber o nmero de anagramas elementos repetidos ou no, e representamos por Pn1, 2 , ..., k o nmero
que possuem H e R separados, porm pelo exemplo anterior j sabemos de permutaes completas de n objetos, sendo 1 o nmero de objetos
resolver isso quando os objetos esto juntos, sendo assim: do 1o tipo, 2 do 2o tipo e assim por diante. Como os s representam a
multiplicidade de cada objeto devemos ter 1 + 2 +...+ k = n.
Total de anagramas: P7 = 7! = 5040.
Anagramas com H e R juntos: P6 P2 = 6! 2!= 720 2 = 1440. Assim, o nmero de permutaes completas ser dado por:
Resposta: 5040 1440 = 3600 anagramas.
, 2 ,..., k n!
Pn1
=
Obs.: Essa ideia s foi rpida, porque ele queria apenas duas letras 1 ! 2 !...k !
separadas, no caso em que o problema pea mais de duas letras no
adjacentes existe outra maneira de resolver que ser vista mais a frente. 6.2 Permutaes circulares
5. Arranjos simples E no caso de determinar o nmero de maneiras de colocar n pessoas
em um crculo com seus lugares equiespaados, considerando iguais
Muitos problemas em combinatria esto associados a determinar disposies obtidas atravs de rotao?
o nmero de ordenaes em que alguns objetos podem ser distribudos. Fazendo o caso n = 4 (o caso geral igual): Se a fila formada fosse
Porm, nem sempre estamos interessados em utilizar todos os objetos em linha reta teramos 4! = 24 maneiras de orden-los, porm repare
disponveis, por exemplo, considere que em um parque de diverses que em um crculo temos configuraes iguais (por rotao) como as
existem 20 pessoas querendo entrar em numa montanha russa. Essa representadas abaixo:
montanha russa possui apenas quatro assentos, com disponibilidade
para exatamente uma pessoa. De quantos modos essa montanha russa B A A D
pode ser composta?
Repare que, na verdade, assim como nas permutaes, esse apenas
mais um exemplo de aplicao direta do PFC: = =
20 19 18 ... 1 20 !
20 19 18 17 = =
20 19 18 17 16 15 ... 1 16 ! C D B C

Em um caso geral, se temos n objetos e queremos ordenar p desses


D C C B
objetos, chamamos de arranjo de n escolhe p (An,p) o nmero de maneiras
de fazer essa ordenao:
n! =
An, p = Anp = n ( n 1) ( n p + 1) =
n n 1 n p+1 ( n p)!

n! A B D A
An, p =
( n p)!
Nesse caso devemos dividir o total de permutaes simples por 4,
obtendo 6 permutaes. No caso geral, com n objetos, teramos n rotaes
no crculo de onde basta dividir o total por n.
6. Outras permutaes
Denotamos por (PC)n o nmero de permutaes circulares que podem
ser obtidos com n objetos, assim:
Ao resolver muitos problemas de combinatria comum contarmos
elementos que inicialmente so iguais erroneamente como distintos.
Nesses casos, devemos tentar agrupar as solues iguais, vendo quantas n!
vezes cada objeto est sendo contado repetidamente. ( PC)n = = ( n 1)!
n
Os casos mais clssicos em que isso ocorre so nas permutaes
completas (com repetio) e nas circulares.

208 Vol. 1
Anlise combinatria

7. Combinaes simples Ou seja, de modo geral, basta considerar as escolhas com ordem,
e depois dividir pelo fatorial da quantidade de termos escolhidos para
Existem alguns casos em que estamos interessados apenas em consertar isso.
escolher um subconjunto de objetos dentre um conjunto maior disponvel, Sendo assim, se temos n objetos e queremos escolher p chamamos
no importando a ordem com que isso feito. Um exemplo disso, supondo de combinaes simples o nmeros de escolhas distintas:
que voc possui um grupo de 10 amigos e deseja escolher trs para fazer
uma viagem com voc, de quantos modos isso pode ser feito? An,p n!
Cn, p = =
Se a ordem fosse importante, a resposta seria A10,3 = 10 9 8 = p! ( n p)! p!
720. Porm, como nesse caso a ordem no importa s escolhas: (A1, A2,
A3); (A1, A3, A2); (A2, A1, A3); (A2, A3, A1); (A3, A1, A2); (A3, A2, A1) so todas
iguais, ou seja, cada escolha est sendo contada seis vezes, logo, basta
dividir o total por 6, obtendo 120 escolhas possveis.

EXERCCIOS RESOLVIDOS
01 (UFRJ) A mala do Dr. Z tem cadeado cujo segredo uma
combinao com cinco algarismos, cada um dos quais podendo variar 1a etapa: Diviso das bolas: 3 opes (3,5); (4,4); (5,3)
de 0 a 9. Ele esqueceu a combinao que escolhera como segredo, 2a etapa: Diviso das camisas: 3 opes (2,4); (3,3); (4,2)
mas sabe que atende s condies: 3a etapa: Diviso das caixas: 7 opes (2,8); (3,7); (4,6);...; (8,2)
Resposta: 3 3 7 = 63 modos.
se o 1o algarismo mpar, ento o ltimo tambm mpar;
se o 1o algarismo par, ento o ltimo igual ao 1o; 03 (UFRJ) Uma partcula desloca-se sobre uma reta, percorrendo
a soma do 2o com o 3o 5. 1cm para a esquerda ou para a direita a cada movimento. Calcule de
quantas maneiras diferentes a partcula pode realizar uma sequncia
Quantas combinaes diferentes atendem s condies de 10 movimentos terminando na posio de partida.
estabelecidas pelo Dr. Z?
Soluo
Soluo Representemos cada movimento para esquerda por E e cada
Sejam ABCDE os dgitos (nessa ordem). Vamos dividir o problema movimento para a direita por D. Veja que para terminar no ponto de
em casos: partida, necessrio ter 5D e 5E. Alm disso, veja que cada maneira de
realizar os 10 movimentos pode ser vista como uma sequncia desses
1o caso A mpar. 5D e 5E. Ento, o nmero de maneiras de se realizar esses movimentos :
A: 5 opes (1, 3, 5, 7, 9) 5,5 10 !
=
P10 = 252
B: 6 opes (0, 1, 2, 3, 4, 5) 5 !5 !
C: 1 opo (fica determinado pela escolha de B)
D: 10 opes 04 Oito crianas vo se dividir em dois times de 4 para disputar uma
E: 5 opes (tambm mpar nesse caso) partida de futebol. De quantas maneiras isso pode ser feito se:

No 1o caso, temos o total de 5 5 6 1 10 = 1500 combinaes. a. um time joga com camisa e o outro joga sem?
b. os dois times jogam sem camisa?
2o caso A par.
A: 5 opes (0, 2, 4, 6, 8) Soluo
B: 6 opes a. Dentro de cada time, no importa a ordem na qual feita a escolha,
C: 1 opo portanto precisamos usar combinao. Para o time com camisa,
D: 10 opes
8!
E: 1 opo (E = A) C84
temos = = 70 . Para o time sem camisa, colocamos as
4 !4 !
4 4!
No 2o caso, temos o total de 5 1 6 1 10 = 300. crianas restantes, o que s pode ser feito de=
C4 = 1 maneira
0 !4 !
Como dividimos em casos (disjuntos), devemos somar as respostas possvel.
dos casos, obtendo assim 1500 +300 =1800 como resposta.
Ento, so 70 1 = 70 maneiras.
02 Dois irmos gmeos ganharam de aniversrio 8 bolas de futebol
iguais, 6 camisas iguais e 10 caixas de chocolate tambm idnticas. b. O que muda aqui que a diviso passa a ser feita em grupos
De quantos modos pode-se dividir esses presentes entre os dois de indistinguveis. Veja que se 1, 2, 3, 4, 5, 6, 7, 8 so as crianas,
modo que cada um receba, pelo menos, 3 bolas de futebol, 2 camisas as divises 1234/5678 e 5678/1234 so iguais (s porque os
e 2 caixas de chocolate? dois times esto sem camisa no item a, essas duas divises
so diferentes). Portanto, estamos contando duas vezes cada
Soluo configurao. Logo, precisamos dividir por 2 para contar apenas
Considere os irmos A e B. Repare que, como os objetos do mesmo 70
uma vez, o que nos d a resposta = 35 .
tipo so idnticos, no importante quais vamos escolher, o importante 2
quantos vamos escolher. Ento, temos:

AFA-EFOMM 209
Matemtica III Assunto 2

9. Solues inteiras no-negativas Uma interpretao para esse problema pensar que queremos
distribuir cinco objetos idnticos entre trs pessoas, porm uma delas
Os conceitos iniciais de combinatria, como a permutao e a deve receber pelo menos dois. Podemos ento entregar primeiro dois
combinao, envolvem a ordenao ou a escolha de determinados objetos. objetos a essa pessoa e depois distribuir trs objetos de qualquer maneira
E se quisssemos distribuir objetos idnticos a um grupo de pessoas? entre as trs pessoas.
Como isso deve ser feito?
II. (Se a soma for menor ou igual a um nmero) Qual o nmero de
Como os objetos so idnticos, estamos interessados apenas em
solues inteiras no-negativas de x + y + z 5?
determinar quantos objetos cada pessoa ir ganhar. Ento, por exemplo,
se temos cinco bombons e queremos distribuir a duas pessoas, basta ver
Soluo
o nmero de solues inteiras no-negativas da equao: x + y =5, onde
x e y representam o nmero de bombons que cada um ganhou. Podemos interpretar esse problema da seguinte forma: temos cinco
objetos e devemos entregar esses objetos (no necessariamente todos)
Nesse caso, temos as seguintes solues: (5, 0); (4, 1); ...; (0, 5)
a trs pessoas.
6 solues.
Nesse caso, os objetos que no esto sendo entregues ficaro com
Porm, como em muitos problemas de combinatria, se os nmeros
o dono, logo, na prtica, estamos distribuindo esses objetos entre quatro
aumentarem um pouco fica impraticvel listar todos os casos.
pessoas, assim o problema equivalente a:
Ento a pergunta como determinar o nmero de solues inteiras
x + y + z + f = 5 (onde f a folga da equao, ou seja, o quanto
no-negativas da equao:
falta para a soma chegar a cinco).
x1 + x2 + ... + xn = p 5,3 8!
Desse modo, temos: P=
8
= 56 solues.
5 !3 !
Repare que basicamente o que se quer fazer separar os p objetos
iguais em n grupos. Podemos representar cada objeto por um ponto, III. (Limitando uma varivel por cima): Qual o nmero de solues inteiras
e como eles devem ser separados em n grupos, deve-se inserir n 1 no-negativas de x + y + z = 8, se x 3 ?
divisrias entre esses pontos. O nmero de solues o nmero de trocas
possveis entre as posies das divisrias (que iremos representar por Soluo
barras) e dos pontos. Nesse caso, como j se sabe limitar uma varivel por baixo (ver ex. I),
Para facilitar a visualizao, considere o seguinte exemplo: podemos calcular o total de solues, sem restries, e subtrair aquelas
que possuem x 4 .
Quantas solues existem para a equao: x + y +z = 3 vamos 10 !
Total de solues (sem restrio): = 45 solues.
representar cada soluo de acordo com o que foi exposto acima: 8 !2!
Solues com x 4 Temos x = x + 4, donde x + y + z = 4 que
(3, 0, 0) (0, 2, 1)
possui 6 ! = 15 solues.
4 !2!
(0, 3, 0) (1, 0, 2)
Resposta: 45 15 = 30.
(0, 0, 3) (1, 2, 0)
Obs.: No caso de termos mais de uma varivel limitada por cima, o
problema fica um pouco mais difcil. Esse tipo de problema ser visto num
(1, 1, 1) (2, 1, 0)
tpico mais a frente chamado de incluso-excluso.
(0, 1, 2) (2, 0, 1)
10. Combinaes com repetio (ou completa)
fcil entender que existe uma bijeo entre as solues e as
representaes por pontos e barras, ou seja, para determinar o nmero de J vimos o nmero de maneiras de escolher p objetos distintos
solues da equao basta determinar o nmero de permutaes existentes. dentre n disponveis. E se pudssemos escolher um mesmo objeto mais
de uma vez?
No caso geral, temos p pontos e n 1 barras, logo temos Pnn+p1,p1
solues inteiras no-negativas. Nesse caso, a pergunta a ser respondida quantas vezes cada objeto
ser escolhido, podendo alguns deles no serem escolhidos.
Ex.: O problema pode ser visto basicamente como o nmero de solues
I. (Limitando as variveis por baixo) Qual o nmero de solues inteiras inteiras no-negativas de uma equao, assim se xi o nmero de vezes
no-negativas de x + y + z = 5, se x 2 ? que o objeto i escolhido, e se chamarmos o nmero de escolhas possveis
de ( CR ) p (combinao com repetio de n objetos tomados p a p) tm-se:
n
Soluo
Podemos simplesmente substituir variveis. Se x 2 , ento existe p
x1 + x2 + ... + x n = p ( CR ) n = Pnn+p1,p1 =
( n + p 1)! = C p
p!( n 1) ! n + p1solues.
x ' 0 inteiro tal que x = x + 2, assim a equao fica:
3,2 5!
x + y + z = 3 que possui P= 5 = 10 solues. Ex.:
3 !2!
De quantos modos podemos comprar 3 refrigerantes em uma loja
onde h 5 tipos de refrigerantes?

210 Vol. 1
Anlise combinatria

Soluo EXERCCIOS NVEL 1


Devemos determinar quantas vezes cada refrigerante ser escolhido,
01 (IME) Determine quantos nmeros de 4 algarismos diferentes podem
ou seja, devemos ver quantas solues possui a equao:
ser formados com os algarismos 0, 1, 2, 3, 4, 5.
x1 + x2 + x3 + x4 + x5 = 3.
n = (CR)5, 3 = C7, 3 = 35 Obs.: Considere os nmeros iniciados com o algarismo 0 (por exemplo,
0123), nmero de 3 algarismos.
EXERCCIOS RESOLVIDOS
02 (UFRJ) Quantos nmeros de 4 algarismos podemos formar nos quais
01 (UFPB) Deseja-se pintar 6 esferas, recebendo cada uma tinta de o algarismo 2 aparece ao menos uma vez?
uma s cor escolhida entre 3 disponveis. De quantas maneiras pode-se
pintar o conjunto de esferas? 03 (UFRJ) Um construtor dispe de quatro cores (verde, amarelo, cinza
e bege) para pintar cinco casas dispostas em uma rua, lado a lado. Ele
(A) 30. (C) 28. deseja que cada casa seja pintada com apenas uma cor e que duas
(B) 27. (D) N.R.A. casas consecutivas no possuam a menor cor. Determine o nmero de
possibilidades diferentes de pintura.
Soluo:
Letra: B. 04 (Morgado) De um baralho comum (52 cartas) sacam-se sucessivamente
p
Trata-se de uma combinao completa ( CR )n = Cn + p1, p com n = 3 e sem reposio 3 cartas. Quantas so as extraes nas quais a primeira
6 6 8! carta de copas, a segunda um rei e a terceira no uma dama?
e p = 6, assim: ( CR )=
3 C=
8 = 28 solues.
6 !2!
05 (EFOMM) O cdigo Morse usa palavras contendo de 1 a 4 letras, as
02 Quantas ss as solues inteiras no negativas de x + y + z + w < 8? letras sendo ponto e trao. Quantas palavras existem no cdigo Morse?

Soluo: 06 (Morgado) Quantos so os anagramas da palavra CAPTULO:


Como estamos interessados nas solues inteiras se x + y + z +
w < 8, devemos ter: x + y + z + w 7que equivalente a x + y + z a. que comeam por consoante e terminam por vogal?
+ w + i = 7. b. que tm as letras C, A, P juntas nessa ordem?
Neste caso devemos permutar 7 pontos e 4 barras: c. que tm as letras C, A, P juntas em qualquer ordem?
7,4 11! d. que tem as vogais e consoantes intercaladas?
P= = 330 solues.
11 7 !4 ! e. que tm a letra C no 1o lugar e a letra A no 2o lugar?
f. que tm a letra C no 1o lugar ou a letra A no 2o lugar?
03 Quantas so as solues inteiras positivas de x + y + z = 8? g. que tm a letra C no 1o lugar ou a letra A no 2o lugar ou a letra P
no 3 lugar?
Soluo:
Como as solues so positivas, devemos ter: 07 (Olimpada Belga) Um nmero inteiro no negativo dito palndromo
se ele lido da esquerda para a direita igual quando lido da direita para a
x 1; y 1; z 1, em que x = x '+ 1; y = y '+ 1; z = z '+ 1 com x, y esquerda. Por exemplo, 121, 0, 2002 e 4 so palndromos. O nmero de
e z inteiros no negativos, logo: x + y + z = 5. palndromos que so menores que 1.000.000 :
5,2 7!
Permutando cinco pontos e duas barras: P= 7 = 21
5 !2! (A) 900. (D) 1999.
solues. (B) 1991. (E) 2220.
(C) 1993.
04 (UFRJ) Uma estante de biblioteca tem 16 livros: 11 exemplares
08 (ITA) Se colocarmos em ordem crescente todos os nmeros de 5 (cinco)
do livro Combinatria fcil e 5 exemplares de Combinatria no
algarismos, obtidos com 1, 3, 4, 6 e 7, a posio de nmero 61473 ser:
difcil. Considere que os livros com mesmo ttulo sejam indistinguveis.
Determine de quantas maneiras diferentes podemos dispor os 16
(A) 76o.
livros na estante de modo que dois exemplares de Combinatria no
(B) 78o.
difcil nunca estejam juntos.
(C) 80o.
(D) 82o.
Soluo:
(E) N.D.A.
Representemos por F os livros Combinatria fcil e por D os
livros Combinatria no difcil. Normalmente, em combinatria,
09 (ITA) Quantos nmeros de seis algarismos distintos podemos formar
comeamos pela restrio. Este aqui um dos poucos casos em
usando os dgitos 1, 2, 3, 4, 5 e 6 nos quais o 1 e o 2 nunca ocupam
que a ordem de execuo contrria. Coloque os 11 F lado a lado:
posies adjacentes, mas o 3 e o 4 sempre ocupam posies adjacentes?
_F_F_F_F_F_F_F_F_F_F_F_
(A) 144. (D) 288.
Dos 12 espaos determinados acima, precisamos escolher 5 para
(B) 180. (E) 360.
colocarmos os D. Isso garante que os D no ficaro juntos, que a
(C) 240.
restrio do problema. Veja que a ordem da escolha no importante,
5 12!
portanto, usamos combinao e temos = C12 = 792 .
5 !7 !

AFA-EFOMM 211
Matemtica III Assunto 2

10 (ITA 83) Um general possui n soldados para tomar uma posio 17 Uma criana possui 96 blocos distintos. Cada bloco pode ser de
inimiga. Desejando efetuar um ataque com dois grupos, um frontal com r 2 materiais (plstico ou madeira), 3 tamanhos (pequeno, mdio ou grande),
soldados e outro da retaguarda com s soldados (r + s = n), ele poder 4 cores (azul, verde, vermelho e amarelo) e 4 formatos (crculo, hexgono,
dispor seus homens de: quadrado e tringulo). Quantos desses blocos diferem do bloco plstico
mdio vermelho crculo em exatamente dois quesitos? (Um exemplo seria
n! 2n ! o bloco madeira mdio vermelho quadrado.)
(A) maneiras (D) maneiras
( r + s)! ( r + s)!
18 (AFA) Em uma demonstrao de paraquedismo, durante a queda livre,
n! 2n ! participam 10 paraquedistas. Em certo momento, 7 deles devem dar as
(B) maneiras (E) maneiras
r ! s! r ! s! mos e formar um crculo. De quantas formas distintas eles podero ser
n! escolhidos e dispostos nesse crculo?
(C) maneiras
( )!
rs (A) 120.
(B) 720.
11 (ITT JEE) Uma classe tem n alunos, temos que formar uma equipe
(C) 86400.
com eles, incluindo pelo menos dois estudantes e excluindo tambm, pelo
(D) 151200.
menos dois alunos. O nmero de maneiras de formar a equipe :
19 (AFA) Dez bales azuis e oito brancos devero ser distribudos em
a. 2n 2n c. 2n 2n 4
trs enfeites de salo, de modo que um deles tenha 7 bales e os outros
b. 2n 2n 2
dois, no mnimo 5. Cada enfeite dever ter 2 bales azuis e 1 branco, pelo
menos. De quantas maneiras distintas pode-se fazer os enfeites, usando
12 (ITA ) O nmero de arranjos de n + 2 objetos tomados cinco a cinco
simultaneamente todos os bales?
vale 180n. Nestas condies, conclumos que:
(A) 9.
(A) n nmero par. (B) 10.
(B) n um nmero primo. (C) 11.
(C) n est compreendido entre 100 e 200. (D) 12.
(D) n um nmero mpar.
(E) n divisvel por 5. 20 Quantas so as peas de um domin comum?

13 (ITA) Quantos anagramas com 6 caracteres distintos podemos formar 21 (UFRJ) Um campeonato de futebol foi disputado por 10 equipes em
usando as letras da palavra QUEIMADO, anagramas estes que contenham um nico turno, de modo que cada time enfrentou cada um dos outros
duas consoantes e que, entre as consoantes, haja pelo menos uma vogal? apenas uma vez. O vencedor de uma partida ganha 3 pontos e o perdedor
no ganha ponto algum; em caso de empate, cada equipe ganha 1 ponto.
(A) 7.200. (D) 3.600. Ao final do campeonato, tivemos a seguinte pontuao:
(B) 7.000. (E) 2.400. Equipe 1 20 pontos; Equipe 2 10 pontos;
(C) 4.800. Equipe 3 14 pontos; Equipe 4 9 pontos;
Equipe 5 12 pontos; Equipe 6 17 pontos;
14 (Morgado) Quantos so os nmeros naturais de 7 dgitos nos quais Equipe 7 9 pontos; Equipe 8 13 pontos;
o dgito 4 figura exatamente 3 vezes, e o dgito 8, exatamente 2 vezes? Equipe 9 4 pontos; Equipe 10 10 pontos.
15 (Vunesp) A figura a seguir mostra a planta de um bairro de uma cidade. Determine quantos jogos desse campeonato terminaram empatados.
Uma pessoa quer caminhar do ponto A ao ponto B por um dos percursos
mais curtos. Assim, ela caminhar sempre nos sentidos de baixo para 22 (AFA) O nmero de solues inteiras e no negativas da equao
cima ou da esquerda para a direita. O nmero de percursos diferentes x + y + z + t = 6 igual a:
que essa pessoa poder fazer de A at B : (A) 84.
(B) 86.
(C) 88.
(A) 95.040. (D) 90.
(B) 40.635. (E) N.R.A.
(C) 924.
(D) 792. 23 (EN) Uma livraria vai doar 15 livros iguais a 4 bibliotecas. Cada
(E) 35. biblioteca deve receber ao menos dois livros. O nmero de modos que
esses livros podem ser repartidos nessa doao igual a:
16 (UFRJ) Um grupo constitudo por 4 mulheres e 4 homens deve ocupar
(A) 1.365.
as 8 cadeiras dispostas ao redor de uma mesa circular. O grupo deve
(B) 840.
ser acomodado de modo que cada homem sente entre duas mulheres.
(C) 240.
Joo e Maria esto nesse grupo de pessoas; entretanto, por motivos
(D) 120.
de ordem estritamente pessoal, no podem sentar-se lado a lado.
(E) 35.
Duas acomodaes das pessoas ao redor da mesa so consideradas
diferentes quando pelo menos uma no tem o mesmo vizinho direita,
24 (UESPI) Um supermercado oferece 10 variedades de sopas em
nas duas acomodaes. Determine o nmero de diferentes acomodaes
pacotes. De quantas maneiras um consumidor pode escolher 4 pacotes
possveis dessas 8 pessoas ao redor da mesa circular.
de sopas, se pelo menos 2 pacotes devem ser da mesma variedade?

212 Vol. 1
Anlise combinatria

(A) 500. 05 (Morgado) No quadro abaixo, de quantos modos possvel formar a palavra
(B) 505. MATEMTICA, partindo de um M e indo sempre para a direita ou para baixo?
(C) 510.
(D) 515. M
(E) 520. M A
M A T
25 (UFF) Quinze (15) pessoas, sendo 5 homens de alturas diferentes e
M A T E
10 mulheres tambm de alturas diferentes, devem ser dispostas em fila,
obedecendo ao critrio: homens em ordem crescente de altura e mulheres M A T E M
em ordem decrescente de altura. De quantos modos diferentes essas 15 M A T E M A
pessoas podem ser dispostas nessa fila? M A T E M A T
M A T E M A T I
26 (ITT) O nmero de solues inteiras no-negativas de x1 + x2 + x3 + M A T E M A T I C
4x4 = 20 :
M A T E M A T I C A
(A) 530. 06 Para a Seleo Brasileira de Futebol foram convocados 22 jogadores,
(B) 534. os quais jogam em todas as posies, exceto dois deles, que s jogam
(C) 532. no gol. De quantos modos se podem selecionar os 11 titulares?
(D) 536.
07 (ITA) Dispomos de seis cores diferentes. Cada face de um cubo ser
27 (Jos Plnio PRC) De quantas maneiras as letras da palavra pintada com uma cor diferente, de forma que as seis cores sejam utilizadas.
INDIVISIBILIDADE podem ser permutadas de modo que duas letras I De quantas maneiras isto pode ser feito, se uma maneira considerada idntica
nunca fiquem juntas? outra, desde que possa ser obtida a partir desta por rotao do cubo?

EXERCCIOS NVEL 2 08 (Jos Plnio PRC) De quantas maneiras 8 contas distintas podem
01 (Morgado) Um campeonato disputado por 12 clubes em ser colocadas num cordo elstico de modo a formar uma pulseira?
rodadas de 6 jogos cada. De quantos modos possvel selecionar os
jogos da primeira rodada? 09 (UFRJ 08) Para montar um sanduche, os clientes de uma lanchonete
podem escolher:
02 (ITA) Uma escola possui 18 professores, sendo 7 de Matemtica, 3 de um dentre os tipos de po: calabresa, organo e queijo;
Fsica e 4 de Qumica. De quantas maneiras podemos formar comisses de um dentre os tamanhos: pequeno e grande;
12 professores de modo que cada um contenha exatamente 5 professores de um at cinco dentre os tipos de recheio: sardinha, atum, queijo, presunto
de Matemtica, com no mnimo 2 de Fsica e no mximo 2 de Qumica? e salame, sem possibilidade de repetio de recheio em um mesmo
sanduche.
(A) 875.
(B) 1877. Calcule:
(C) 1995.
(D) 2877. a. quantos sanduches distintos podem ser montados;
(E) N.D.A. b. o nmero de sanduches distintos que um cliente pode montar, se ele
no gosta de organo, s come sanduches pequenos e deseja dois
03 (Jos Plnio - PRC) De quantas maneiras um grupo de 7 pessoas recheios em cada sanduche.
pode ser agraciado com 4 prmios diferentes: (todos os prmios devem
ser distribudos) 10 Tem-se 5 pontos sobre uma reta r e 8 pontos sobre uma reta r paralela a r.
Quantos quadrilteros convexos com vrtices em 4 desses 13 pontos existem?
a. se nenhuma pessoa puder receber mais que um prmio;
b. se cada pessoa puder receber qualquer nmero de prmios (at quatro 11 (ITA 95) Considere todos os nmeros de cinco algarismos formados
naturalmente); pela justaposio de 1, 3, 5, 7 e 9 em qualquer ordem, sem repetio.
c. se o vencedor do primeiro prmio no puder receber outro prmio, mas Calcule a soma de todos esses nmeros.
vencedores de outros prmios puderem receber mais de um prmio.
(A) 5 106 e 6 106.
04 (OBM) Cinco amigos, Arnaldo, Bernaldo, Cernaldo, Dernaldo e Ernaldo, (B) 6 106 e 7 106.
devem formar uma fila com outras 30 pessoas. De quantas maneiras (C) 7 106 e 8 106.
podemos formar esta fila de modo que Arnaldo fique na frente de seus 4 (D) 9 106 e 10 106.
amigos? (Obs.: Os amigos no precisam ficar em posies consecutivas) (E) 10 106 e 11 106.
35 !
(A) 35! (D) 5! 12 (ITA) Considere (P) um polgono regular de n lados. Suponha que os
5 vrtices de (P) determinem 2n tringulos, cujos lados no so lados de
35 !
(B) (E) e 163 (P). O valor de n :
5!
35 ! (A) 6. (D) 20.
(C)
5 (B) 8. (E) No existe este polgono.
(C) 10.

AFA-EFOMM 213
Matemtica III Assunto 2

13 (Morgado) No incio de uma festa h 6 rapazes desacompanhados e (A) 48. (D) 96.
10 moas desacompanhadas. Quantos so os estados possveis no fim (B) 60. (E) 120.
da festa? (C) 72.

18 (UFPE Adaptado) No mapa abaixo esto esboadas as ruas de um


14 (Morgado) So dados n pontos em crculo. Quantos n-gonos (no
bairro. As ruas verticais so paralelas entre si e a distncia entre duas ruas
necessariamente convexos) existem com vrtices nesses pontos?
consecutivas a mesma; o mesmo acontece com as ruas horizontais.
Calcule o nmero de formas de sair de A e chegar at B percorrendo a
15 (Jos Plnio PRC) Quantos nmeros distintos podem ser formados menor distncia possvel.
pelo produto de dois nmeros ou mais do multiconjunto {3, 4, 4, 5, 5,
6, 7, 7, 7}. (Obs.: Um multiconjunto um conjunto em que o nmero de B
cpias de um elemento relevante.)

16 (EN 91) A partir de um conjunto de 19 atletas, formam-se 57 times


de 4 atletas cada. Todos os atletas participam de um mesmo numero de
times e cada par de atletas fica junto no mesmo time um mesmo numero
x de vezes . O valor de x ?

(A) 1. (D) 4.
(B) 2. (E) 5. A
(C) 3.

17 (OBM) Dizemos que uma palavra Q quase anagrama de outra palavra 19 (Morgado) Escrevem-se nmeros de cincos dgitos (inclusive os
P quando Q pode ser obtida retirando-se uma letra de P e trocando a ordem comeados por zero) em cartes. Como 0, 1 e 8 no se alteram de cabea para
das letras restantes, resultando em uma palavra com uma letra a menos do baixo e como 6 de cabea para baixo se transforma em 9, um s carto pode
que P. Um quase-anagrama pode ter sentido em algum idioma ou no. Por representar dois nmeros (por exemplo, 06198 e 86190). Qual o nmero
exemplo, RARO, RACR e ARCO so quase anagramas de CARRO. Quantos mnimo de cartes para representar todos os nmeros de cinco dgitos?
so os quase anagramas da palavra BACANA que comeam com A?

RASCUNHO

214 Vol. 1
Trigonometria A ssunto
1
Matemtica IV

Introduo 1.2 Linhas trigonomtricas notveis

A trigonometria surgiu com o objetivo de estabelecer relaes entre


ngulo Seno Cosseno Tangente
ngulos (normalmente fceis de medir) e comprimentos (s vezes
difceis de mensurar, como no caso da largura de um rio extenso ou do 1 3 3
comprimento de um morro/prdio muito alto) em figuras geomtricas. 30
2 2 3
Hoje, a trigonometria tambm tem aplicaes como ferramenta puramente
algbrica, til para descrever fenmenos fsicos e para simplificaes
matemticas. 2 2
45 1
Os seus objetivos nesta seo incluem memorizar as definies 2 2
trigonomtricas e as relaes algbricas entre elas, memorizar
transformaes trigonomtricas e identificar oportunidades de aplicao 3 1
60 3
(usualmente a parte mais difcil e mais importante), resolver equaes e 2 2
inequaes trigonomtricas e compreender o comportamento das funes
trigonomtricas e suas inversas.
Demonstrao: basta aplicar as definies s figuras abaixo:
1. Definies e relaes bsicas
Tringulos retngulos com um ngulo comum sempre so
semelhantes (caso AA) e, portanto, tm a mesma razo entre lados 45
l 2
30
correspondentes. Essas razes recebem nomes especiais, definidos
l
abaixo:
l l 3
2
hipotenusa cateto 45 60
cateto oposto
sen = oposto l
hipotenusa a l
cateto adjacente cateto 2
cos =
hipotenusa adjacente
taan =
cateto oposto
=
sen 1.3 Relaes entre ngulos complementares
cateto adjacente cos
hipotenusa
cateto sen(90 a) = cosa
oposto cos(90 a) = sena
a
cateto tan(90 a) = cota
adjacente
Demonstrao: Basta ver que o cateto oposto ao ngulo a adjacente
1 1 1 cos ao ngulo 90 a .
csc = , sec = , cot = =
sen cos tan sen

Obs.: Em um tringulo qualquer, vale a lei dos senos, que relaciona um 1.4 Linhas trigonomtricas para ngulos quaisquer
ngulo a, o lado a oposto a esse ngulo e o raio R do crculo circunscrito (ciclo trigonomtrico)
de um tringulo pela frmula a = 2R.sena
Podemos estender as definies de linhas trigonomtricas para ngulos
1.1 Relaes fundamentais maiores que 90 (ou menores que zero) com auxlio do ciclo trigonomtrico
(circunferncia de raio 1, como nas figuras).
sen2a + cos2a = 1
Para ngulos entre 0 e 90 (1o quadrante), temos por definio (fazendo
tan2a + 1 = sec2a 1o hipotenusa = 1) que o cosseno a projeo do raio no eixo horizontal
cot2a + 1 = csc2a e o seno a projeo no eixo vertical. Estendendo esta ideia, definimos
o cosseno (seno) de um ngulo x qualquer como o tamanho da projeo
Demonstrao: Substituindo as definies, vemos que todas essas no eixo horizontal (vertical) do raio que forma um ngulo x com o eixo
relaes so equivalentes ao teorema de Pitgoras. horizontal. As demais linhas trigonomtricas continuam definidas em
funo do seno e do cosseno como nos ngulos agudos.

AFA-EFOMM 215
Matemtica IV Assunto 1

sen sen Demonstrao: O primeiro caso segue da definio, dado que x e


1o Q 2o Q x+2p tm a mesma representao no ciclo trigonomtrico. O segundo
caso segue de tan(x + ) = sen( x + ) = senx = tan x .
sen cos( x + ) cosx
a a
cos cos cos cos
EXERCCIOS RESOLVIDOS
01 Calcule, para todo x, 3(sen4 x + cos4 x) 2(sen6 x + cos6 x).

Soluo:
No basta atribuir valores para x, j que queremos determinar a expresso
sen > 0 sen > 0 para todo x. Elevando ao quadrado a relao fundamental, temos que
cos > 0 cos < 0
(sen2x + cos2x)2 = 1, que nos d sen4 x + cos4x = 1 2sen2x cos2x
(repare que usamos (a + b)2 = a2 + 2ab + b2).
sen sen Agora, elevando ao cubo a relao fundamental, temos que
(sen2x + cos2x)3 = 1, que nos d
sen6x + cos6x = 1 3sen2x cos2x (sen2x + cos2x) = 1 3sen2x cos2x
(repare que usamos (a + b)3 = a3 + b3 + 3ab(a + b)). Da, a expresso
a a dada igual a: 3 (1 2sen2x cos2x) 2 (1 3sen2x cos2x) = 1.
cos cos
02 Simplifique a expresso sen4 x + 4 cos2 x cos4 x + 4sen2 x .

Soluo:
3o Q 4o Q Quando s temos seno e cosseno, inevitvel usarmos a relao
sen < 0 fundamental.
sen < 0
cos > 0 O 1o radical igual a:
cos < 0
(1 cos x )
2
2
+ 4 cos2 x = 1 2 cos2 x + cos4 x + 4 cos2 x =
1.5 Reduo ao 1 quadrante o
(1 + cos x )
2
= 1 + 2 cos2 x + cos4 x = 2
= 1 + cos2 x
Casos principais:
ngulos suplementares (2o para 1o quadrante): sen(180 x) = senx, De forma anloga, temos que o 2o radical igual a 1 + sen2 x.
cos(180 x) = cosx Portanto, a expresso dada igual a: (1 + cos2x) (1 + sen2x) =
ngulos explementares (3o para 1o quadrante): sen(180 + x)= senx, cos2x sen2x = cos2x.
cos(180 + x) = cosx
4 5sen + 7 cos
ngulos replementares (4o para 1o quadrante): sen(360 x)= senx, 03 Sendo tan = , calcule F = .
15 6 cos 3sen
cos(360 x) = cosx
Soluo:
Demonstrao: Basta desenhar o ponto no ciclo trigonomtrico, Em F, basta dividir o numerador e o denominador por cos:
observar a orientao dos eixos cosseno/seno e compar-lo (visualmente sen cos 4
5 +7 5 + 7
ou por meio de congruncia de tringulos) com o ponto correspondente cos cos 5 tan + 7 15 125
F= = = =
no 1o quadrante. cos sen 6 3 tan 4 78
6 3 6 3
cos cos 15
Obs.: Na prtica, a melhor forma de resolver este tipo de problema
usando as frmulas de adio e subtrao de arcos que veremos em 2.1.
04 Sabendo que 9 cos2x 5 senx cosx + 4 sen2x = 3 , determine
tanx.
1.6. Paridade
sen( x)= senx Soluo: Usando que 3 = 3 (sen2x + cos2x), observamos que
essa uma equao homognea de grau 2 em senx, cosx (reveja
cos( x)= cosx
o conceito na apostila de lgebra bsica). Dividindo por cos2x,
tan( x)= tanx obtemos 9 5 tanx + 4tan2x = 3 sec2x (= 3 + 3 tan2x), ou seja,
tan2x 5 tanx + 6 = 0. Resolvendo a equao do 2o grau, obtemos
Demonstrao: So as simetrias (congr uncias) no ciclo
tanx = 2 ou tan x = 3.
trigonomtrico.

Obs.: Usaremos, a partir de agora, indiscriminadamente as unidades 05 Simplifique a expresso sen(270o + a) + sen(450o a).
grau e radiano para representar ngulos (180 graus equivalem a p radianos).
Soluo: Escrevendo 270 = 3 90o e 450 = 5 90o, podemos
1.7 Periodicidade desenhar os pontos no ciclo trigonomtrico e concluir que:
sen(270o + a) = cosa e sen (450o a) = sen(90o a) = cosa.
Seno e cosseno tm perodo 2p: sen( x + 2 p) = senx e cos( x + 2p) Logo, a expresso vale zero.
= cosx para todo x.
Tangente tem perodo p: tan(x + p) = tan x para todo x.

216 Vol. 1
Trigonometria

2. Transformaes trigonomtricas C

Para manipular expresses trigonomtricas (simplificar expresses,


resolver equaes/inequaes), importante compreender e memorizar
transformaes importantes como adio/subtrao de arcos, arco duplo, 1
arco metade, fatorao (transformao soma em produto), transformao
produto em soma e outras que veremos a seguir.
+

2.1 Adio e subrao de arcos
0 P
sen(a b) = sena cosb senb cosa
cos(a b)=cosa cosb sena senb
PC
tan tan sen(a + b) = = AB + QC = sena cosb + senb cosa
tan( ) = 1
1 tan tan
OP
cos(a + b) = = OA BQ = cosa cosb sena senb
1
Demonstrao: Inicialmente, sobreponha um tringulo retngulo de
ngulo a a um de ngulo b e hipotenusa 1 como na sequncia de figuras Dividindo uma pela outra e, em seguida, dividindo numerador e
abaixo. Em seguida, pense no seno como a projeo da hipotenusa no denominador por cosa cosb:
cateto separado e no cosseno como a projeo no cateto colado para
obter as seguintes relaes: sen cos sen cos
+
C sen( + ) cos cos cos cos tan + tan
tan( + ) = = =
cos( + ) cos cos sen sen 1 tan tan

cos cos cos cos
90
1 B
Para as frmulas de subtrao, basta escrever a b = a + ( b)
90 e usar paridade.

Obs.: Como usamos somente a definio de seno e cosseno como
0 P A projees, esta demonstrao funciona mesmo que a e b no sejam
ngulos agudos.
C

sen 2.2 Arco duplo

1 B cos 2 a = cos2 a sen2 a = 2 cos2 a 1 = 1 2sen2 a


sen( 2 a) = 2sena cos a
cos

2 tan( a)
tan( 2 a) =
1 tan2 a
0 A
Demonstrao: Basta tomar b = a na frmula de adio de arcos.
C
sen cos
sen 2.3 Arco metade
sen sen
Q B a 1 + cos a
cos =
2 2

cos cos sen a 1 cos a
sen =
2 2
P
0 A a 1 cos a
cos cos tan =
2 1 + cos a
No tringulo OBC, temos: OB = cosb e BC = senb.
No tringulo OAB, temos: OA = cosa hipotenusa = cosa cosb e Demonstrao: Trocando 2a por a na frmula do cosseno do arco
AB = sena cosb. duplo, obtemos as duas primeiras frmulas. Dividindo uma pela outra,
No tringulo QBC, temos: BQ = hipotenusa sena = senb sena e obtemos a terceira.
QC = senb cosa.
Logo, olhando para o tringulo OPC, temos:

AFA-EFOMM 217
Matemtica IV Assunto 1

a a 1 + cos a 2.5 Transformao produto em soma


cos a = 2 cos2 1 cos2 =
2 2 2
Em algumas ocasies (por exemplo, quando um produto est sendo
a a 1 cos a somado a uma parcela) interessante fazer o contrrio da fatorao, i.e.,
cos a = 1 2sen2 sen2 =
2 2 2 transformar um produto em uma soma.
Obs.: Em algumas situaes, interessante utilizar essas 1
frmulas ao contrrio, fatorando expresses da forma 1 cosa sena cos b = ( sen ( a + b ) + sen ( a b ) )
2
2 a 1
por exemplo, 1 cos a = 2sen 2 ou 1 sena (neste caso, primeiro cos a cos b = ( cos ( a + b ) + cos ( a b ) )
2
faz-se sena = cos(90 a)).
1
sena senb = ( cos ( a + b ) cos ( a b ) )
2
2.4 Fatorao (ou transformao soma em produto)
Demonstrao: basta observar as contas intermedirias da
Fatorar (transformar soma em produto) uma das principais
demonstrao do teorema 10.
ferramentas algbricas para simplificao de expresses e resoluo de
equaes, de forma que estes resultados merecem ateno especial. 2.6 Truque do tringulo retngulo

xy xy Para lidar com combinaes lineares de senos e cossenos de um


senx sen y = 2sen cos mesmo arco (ex.: senx+ cosy, 3senx + 2cosx, 5senx + 12cosx, etc.)
2 2
reescrevemos a expresso como:
x+y xy
cos x cos y = + 2 cos cos
2 2 a senx + b cos x = R sen( x + ), em que
x+y xy b
cos x cos y = 2sen sen R = a2 + b2 , tan =
2 2 a

tana tanb = sen(a b) seca secb Demonstrao: Considerando um tringulo retngulo de catetos a e
b como na figura abaixo:
cota cotb = sen(a b) csca cscb

Demonstrao: Fazendo x = a + b, y = a b em 2.1., temos R = a2 + b 2


x+y xy b
a= ,b= e:
2 2

senx + seny = sen( a + b) + sen( a b) = 2sena cos b =
x+y xy a
= 2sen cos
2 2 Temos a = R cosa, b = R sena
cos x + cos y = cos( a + b) + cos( a b) = 2 cos a cos b = Logo,
x+y xy a senx + b cosx = R (senx cosa + sena cosx) =
= 2 cos cos
2 2 = R sen(x + a)
cos x cos y = cos( a + b) cos( a b) = 2sena senb =
x+y xy Obs.: Embora a intuio geomtrica utilizada s funcione para
= 2sen sen ngulos agudos, possvel demonstrar o resultado no caso geral.
2 2
a a
Como 1 , basta escolher a tal que cos = e
Trocando x por x, obtemos as demais frmulas.
2
a +b 2
a + b2
2

Substituindo a definio de tangente e cotangente: b


sen = e substituir.
a + b2
2

sena senb sena cos b + senb cos a


tan a + tan b = + = = Demonstrao: A primeira expresso a mais difcil de demonstrar:
cos a cos b cos a cos b partindo do seno do arco duplo, multiplicando em cima e embaixo por cosseno
= sen( a + b) sec a sec b para forar o aparecimento da tangente, e usando a relao fundamental:
cos a cos b senb cos a + sena cos b
cot a + cot b = + = = x x x
sena senb sena senb sen 2 tan 2 tan
x x 2 2 x 2 2
= sen( a + b) csc a csc b senx = 2sen cos = 2 cos = =
2 2 x 2 sec2 x 1 + tan2 x
cos
Trocando b por b, obtemos as demais frmulas. 2 2 2

Obs.: Para fatorar expresses com linhas trigonomtricas diferentes A segunda expresso simplesmente a tangente do arco duplo, e a
(por exemplo, sena + cosb), basta transformar cosseno em seno (por terceira expresso pode ser obtida dividindo-se a primeira pela segunda.
exemplo, fazendo cosb = sen(90 b)) e usar 2.4.

218 Vol. 1
Trigonometria

EXERCCIOS RESOLVIDOS
01 Calcule sen 15o e sen 75o: 03 (Lei das tangentes) Em um tringulo no retngulo, prove que so
iguais a soma e o produto das tangentes dos ngulos internos.
Soluo:
Utilizando as frmulas de subtrao e adio de arcos, temos: Soluo:
Como A, B e C so ngulos de um mesmo tringulo, temos
sen 15o = sen(45o 30o) = sen 45o cos 30o sen 30o cos 45o= A + B + C = 180 (Em todo problema com esse dado A, B e C ngulos
2 3 1 2 6 2 de um tringulo, essa obviamente a primeira ideia). Da A + B =
= . = .
2 2 2 2 4 180 C e, aplicando a funo tangente dos dois lados, temos que
Analogamente, sen 75o = sen(45o + 30o) = sen 45o cos 30o + sen 30o tan A + tan B
tan(A + B) = tan(180 C), logo = tan C. Eliminando
1 tan A tan B
cos 45o = 6 + 2 o denominador, chegamos a tan A + tan B + tan C = tan A tan B tan C.
4
Obs.: Das relaes entre ngulos complementares (1.2.), obtemos cos15=
75 e cos75 = sen15. 04 Sendo A, B e C ngulos de um mesmo tringulo, prove que:

A B C
3 sen A + sen B + sen C = 4cos cos cos .
02 Sabendo que sen x + cos x = , determine os possveis valores de Soluo: 2 2 2
sen(2 x) e tan(2 x). 4
Assim como no exemplo anterior, temos A + B = 180 C, o que d
Soluo: sen C = sen(A + B). Ento: sen A + sen B + sen C = sen A + sen
Elevando a expresso dada ao quadrado para que aparea o termo B + sen(A + B) =
2 sen x cos x da frmula do sen(2 x), obtemos A +B A B A +B A +B
= 2sen cos + 2sen cos =
9 7 2 2 2 2
sen2 x + 2 sen x cos x + cos2 x = 1 + sen2 x = .
16 6 A + B A B A + B
Para calcular a tangente, basta calcular o valor de cos 2x pela relao = 2sen
2 cos 2 + cos 2 (* ) .

fundamental:
A +B C C
sen22x + cos22x = 1 fcil ver que sen
2 = sen 90 2 = cos 2 .

49 207
cos2 2x = 1 = A B A +B
256 256 Alm disso, cos + cos 2
207 2
|cos 2 x| = A B A B A B A B
16 = cos cos + sen sen + cos cos sen sen , o u s e j a ,
2 2 2 2 2 2 2 2
7
A B A +B A B
|tan 2 x | = 16 = 7 207 cos + cos 2 = 2cos 2 cos 2 . Substituindo em (*), temos
207 207 2
o resultado.
16

3. Equaes e inequaes
A primeira etapa para resolver equaes ou inequaes trigonomtricas Senos iguais implicam ngulos congruentes (x y = 2 kp) ou
simplificar as expresses por meio de transformaes, com o objetivo suplementares x + y = (2 k + 1)p.
de chegar a uma igualdade ou desigualdade simples entre linhas
trigonomtricas iguais.
3.2. Caso geral de equaes trigonomtricas
3.1 Igualdade entre mesmas linhas trigonomtricas
No caso geral, deve-se sempre utilizar substituies, transformaes
cos x = cos y x = y + 2 kp, k Z e redues de quadrante para se obter uma situao de igualdade entre
tan x = tan y x = y + kp , k Z mesmas linhas trigonomtricas.
sen x = sen y x = y + 2 kp ou x = y+(2 k +1)p, k Z Ex.:
Para resolver sen x = cos 2x, podemos inicialmente escrever
Demonstraes: Observando o ciclo trigonomtrico e o resultado
sobre reduo ao 1 quadrante:
sen x = cos x ; para encontrar a soluo de sen2 x 3sen x + 2 =0,
2
Cossenos iguais implicam ngulos congruentes (x y = 2 kp) ou fazemos a substituio t = sen x; e para resolver uma equao do
replementares (x + y = 2 kp). tipo sen x + sen 3x = cos x, comeamos por transformar o lado
Tangentes iguais implicam ngulos congruentes (x y = 2 kp) ou esquerdo em um produto.
explementares (x y = p + 2 kp). Combinando as expresses, obtemos
x = y, x = y + p , x = y + 2p, etc.

AFA-EFOMM 219
Matemtica IV Assunto 1

3.3 Inequaes trigonomtricas 4. Funes trigonomtricas


Para casos simples (ex: inequaes em que os argumentos das Como definimos seno e cosseno de maneira nica para todo
funes esto limitados ao intervalo [0,2 p]), basta desenhar o ciclo nmero real x, podemos definir funes f : , f ( x ) = cos x e
trigonomtrico e identificar os intervalos que funcionam. g : , g( x ) = sen x . E, restringindo domnios de forma apropriada,
Para o caso geral, a ideia deixar um lado igual a zero, fatorar o outro lado podemos definir funes para todas as linhas trigonomtricas:
e utilizar quatro sinais para lidar com cada fator conforme resultados abaixo:

tan : x ; x + k , k  , cot : { x ; x k , k } 
sen z > 0 z ( 2 , + 2 ) 2


(i.e., z ... ( 2, ) (0, ) ( 2 , 3 ) ...) sec : x ; x + k , k  , csc : { x ; x k , k } 
2

4.1 Perodo
cos z > 0 z + 2 , + 2
 2 2
5 3 3 5 Tipo de funo Perodo
(i.e., z ... , , , ...)
2 2 2 2 2 2 sen( ax + b), cos( ax + b), 2
sec( ax + b), csc( ax + b) a
EXERCCIOS RESOLVIDOS

01 Utilize a equao sen 3x = sen 2x para obter o valor de cos36. tan( ax + b), cot( ax + b)
a
Soluo: Demonstrao: Basta substituir e usar as expresses de reduo ao
Veja que x = 36 soluo dessa equao, pois sen108 = sen72. 1o quadrante.
Inicialmente, vamos provar que sen3x = 3senx 4sen3 x:
sen3 x = sen(2x + x) = sen2 x cos x + sen x cos 2x =
= 2sen x cos x cos x + sen x(cos2 x sen2 x) =
4.2 Grfico
= 2sen x(1 sen2 x) + sen x(1 2sen2 x) = 3sen x 4sen3 x. Para esboar o grfico de uma funo trigonomtrica, aconselhvel
determinar, se possvel, o perodo, as razes, os pontos de mximo/mnimo
Agora, no problema, temos a equao 3sen x 4sen3 x = 2sen e o comportamento prximo aos pontos fora do domnio da funo. Os
x cos x. grficos das funes principais so:
Para sen x 0, temos 3 4sen2 x = 2 cos x 3 4(1 cos2 x)
Seno:
= 2cos x 4 cos2 x 2cos x 1 = 0. Como cos36 > 0, segue
1+ 5
que cos36 = .
4
Obs.: Em alguns problemas, conveniente saber de antemo o
valor de cos36.

02 Resolva a equao senx + 3 cos x = 1.


Cosseno:
Soluo:
Esse problema pode ser resolvido de outra forma, mas a melhor
soluo utilizando o truque do tringulo retngulo. Dividindo ambos
1 3 1
os lados da equao por 2, temos que senx + cos x = , ou
2 2 2
seja,
1
cos senx + sen cos x = . Usando a frmula de seno da soma, Tangente:
3 3 2
temos

1 5
sen x + = , que d x + = + 2 k ou x + = + 2k ,
3 2 3 6 3 6

para inteiro. Ento S = + 2 k , + 2 k k Z .
6 2

220 Vol. 1
Trigonometria

4.3 Inversa
(C) .
4
Restringindo o domnio e o contra-domnio das funes trigonomtricas,
pode-se garantir que elas sejam bijetoras. Por exemplo, as funes (D) .
7

sen : , [11 , ] cos : [0, ] [11
, ] e tan : , so
2 2 2 2 (E) .
2
todas bijetoras e contnuas. Definem-se as funes trigonomtricas
inversas como: 02 (EN-2000) Em um tringulo retngulo, a hipotenusa o triplo de um
dos catetos. Considerando o ngulo oposto ao menor lado, podemos
, ] , , definida por y = arc sen x x = sen y
arc sen : [11
2 2 afirmar que tan + cos igual a:
, ] [0, ] , definida por y = arc cos x x = cos y
arc cos : [11
5 11 2
(A) (D)
6 4
arc tan : , , definida por y = arc tan x x = tan y
2 2 11 2
(B) (E) 12 + 2
12 4
Obs.: de forma similar, define-se arc cot, arc sec e arc csc com
(C) 2

contra-domnios (0, ), [0, ] , e 2 , 2 {0} respectivamente.
2 7 7
03 (EFOMM-96) Sabendo que A = 6 tg + 4 sen cos , ento o
6 3 6
EXERCCIOS RESOLVIDOS valor de A igual a:
1 1
07 Calcule x = arctan + arctan .
2 3
3 4 12 6
12
Soluo: (A) 2 (D)
4
1
= arctan 2 (B)
3
3
(E)
Defina (na grande maioria dos problemas que 2
= arctan 1 4
3 (C) 3
envolvem funes inversas trigonomtricas, isso uma boa 2

2
ideia!). Essas definies nos do as seguintes informaes: 04 (AFA-2000) Simplificando a expresso (cossec x ) 2 , para
1 cossec x 0, obtemos: (cossec x )2
tan = 2
e , , . (A) cos x
tan = 1 2 2
(B) cos2 x
3 (C) sen2 x
tan + tan (D) cos 2x
Ento, x = + tan x = tan ( + ) =
1 tan tan
05 (AFA-2000) O acesso ao mezanino de uma construo deve ser
1 1 feito por uma rampa plana, com 2 m de comprimento. O ngulo a
+
tan x = 2 3 = 1. que essa rampa faz com o piso inferior (conforme figura) para que
1 1 nela sejam construdos 8 degraus, cada um com 21,6 cm de altura, ,
1
2 3 aproximadamente, igual a:


Como , , , temos que x = + (, ).
2 2

Como tan x = 1, temos que x = . 2m
4

Definies, relaes bsicas e adio de arcos

EXERCCIOS NVEL 1 (A) 15


(B) 30
01 (EFOMM-1999) A soma das razes da equao 4 . cos2q = 1 : (C) 45
(0 < q < p) (D) 60

(A) p. 06 A soma de dois arcos 400. Calcule seus arcos sabendo que seus
3 cossenos so nmeros simtricos.
(B) .
2

AFA-EFOMM 221
Matemtica IV Assunto 1

07 Simplificar as expresses: exerccios nvel 2


a. cos (90 + a) cos(180 a) + sen (180 + a) (90 + a); 01 (EFOMM-2000) As razes da equao 2x2 + 3x 1 = 0 so tg B e
tg C, sendo B e C ngulos de um tringulo. O ngulo A desse tringulo vale:
b. sen(360 + a) + cos a cos(90 a) + sen(90 a) sen(360 a);
sen( a) tan(90 + a) cos a (A) 30. (D) 90.
c. + .
sen(180 + a) cot a sen(90 + a) (B) 45. (E) 120.
cos( 90 + a) sec( a) tan(180 a) (C) 60.
d. .
sec( 360 + a) sen(180 + a) cot(90 a)
02 (AFA-1998) O valor da expresso cos 35 (sen 25 + cos 55) +
sen( 270 a) tan(180 b) cot( 450 a) sen( c 90)
sen 35 (cos 25 sen 55) + tg 31 + tg 14 :
o o
e. +
cot( b 270 ) cos( 540 + a) cos(180 + c) tan(1260 + a) 1 tg 31 tg 14 o
o

15 2+ 3
2 3 (C)
08 Calcule para x = o valor de: (A)
4 2 2
(A) 3sec2 x + sen2 x 2tan x+ cos2 x
(B) 2sen2 x 2tan x + sec2 x 2 3
(B) 3 + 2 (D)
2 3
09 Se sen x + sen2 x = 1, calcule E = cos2 x + cos4 x.
2
03 ( ENEM-2001) Se x 0, e cos2 x sen2 x = , o valor de
10 (EFOMM-02) O resultado da simplificao da expresso 4 5
tg x cotg x : cos2 x + 4 sen2 x + 5 sen x cos x :
sec2 x
cossec 2 x 1

(A) sen x. 19 + 5 21
(A) 13 + 21 (C)
(B) cos x. 10
(C) 1.
(D) 1. (B) 17 + 3 21 (D) 21 + 2 21
(E) 0. 10 3

04 (ITA-77) Considere um tringulo ABC cujos ngulos internos A, B, C


11 Verifique as identidades:
B+C
verificam a relao sen A = tan . Ento, podemos afirmar que:
a. sen6 x + cos6 x 2sen4 x cos4 x + sen2 x = 0 2
b. sec4 x sec2 x = tan2 x + tan4 x
1 2 cos2 x (A) Com os dados do problema, no podemos determinar A nem B nem C.
c. = tan x cot x
sen x cos x (B) Um desses ngulos reto.
sec x cos x 5
d. = tan3 x (C) A = , B + C =
csc x sen x 6 6
5 5
(D) A = , B = , C =
12 Mostre que se tan2 a = 1 + 2tan2 b, ento, cos2 b = 2cos2 a. 6 12 12
(E) n.d.a.
13 Se sen x . cos x = m, determine em funo de m:

a. y = sen x + cos x 05 (ITA-79) Se a e b so ngulos complementares, 0 < a < , 0 < b <
sen a + sen b 3a 2 2
b. y = sen4 x + cos4 x e = 3 , ento sen + cos ( 3 b ) igual a:
sen a sen b 5
sec x + csc x 1
14 Determine o valor de sabendo que senx = .
1 + tan x 5 (A) 3 .
3 3.
15 Determine sen x e tan x sabendo que cos x = e x 3 Q. (B)
5 3
16 (EFOMM-1998) Resolvendo sen 15 sen 75, encontra-se: (C) 2 .

(A) 3 . (D)
2. (D) 2.
2 2
(E) 1.
2 3.
(B) . (E)
2 2
06 (ITA-81) Seja f : 0, definida por f ( x ) = sec2 x + csc2 x .
2
a
(C) 2 . Se 0, tal que tan = , ento f( ) igual a:
2 2 b

222 Vol. 1
Trigonometria

a+ b 2 2 1
(A) (D) a + b 05 (ITA-99) Se x 0, tal que 4 tan4 x = + 4 , ento o valor
2 ab 2 cos4 x
1 2 de sen(2x) + sen(4x) :
(B) a + b2 (E) nenhuma das anteriores
2
2 2 15 1
(C) a b (A) (D)
4 2
ab
15
(B) (E) 1
07 (ITA) Seja a real com 0 < a < . 8
2
3 3 3 15
A expresso sen + a + sen a sen a idntica a: (C)
8
4 4 2
3
06 (EFOMM-1999) Sabendo que < < e que sen = , o valor
2 cot 2 a
(A) (D) 1 + 3 cot a 25 5
1 + cot 2 a 2

de cos + sen ((p
20
2)) igual a:
2
2 cot a 1 + 2 cot a
(B) (E)
1 + cot 2 a 1 + cot a (A) 9 . (D) 4 + 5 .
25 25

2 39
(C) (B) . (E) 3 2 .
2 25
1 + cot a 9
(C) 2 2 .
Transformaes trigonomtricas
07 (AFA-1999) O valor da expresso cos 15 + sen 105 :
EXERCCIOS NVEL 1

(A) 6+ 2
1
01 (EFOMM-96) Sabendo que sen x = + cos x, ento sen 2x vale: 4
5
(B) 6 2
4
25 2
(A) (D) (C) 6+ 2
24 5
2
9 24 6 2
(B) (E) (D)
25 25 2
9 13 11
(C) 08 (EFOMM-01) O valor numrico de y = sen cos :
25 12 12

02 (EFOMM-94) Sendo 0 < x <
2
e sen x = 3 sen 2 x , ento tg 2x vale:
(A)
1
8
( )(
1+ 3 1+ 3 )
(A) 0
(B) 1
(D) 6
(E) 35
1
(
(B) 1 + 3 1 + 3
8
)( )
(C) p 1
(
(C) 1 + 3 3 3
8
)( )

03 (EFOMM-01) O valor de ( tg 10o + cot g 10o ) sen 20o : (D) tg2
11

(A) 0,5 (D) 2,5 (E)
(B) 1 (E) 4 12
(C) 2
09 (ITA -2001) Sendo e os ngulos agudos de um tringulo retngulo,
04 (EFOMM-1995) Se sen2a = x e sen2b = y, ento sen(a + b) cos(a e sabendo que sen2 (2) 2 cos(2) = 0, ento sena igual a:
b) igual a: 4
2 8
(A) . (D) .
(A) x + y (D) x + y 2 4
(B) 2(x + y) (E) x+y 4
2
(B) . (E) zero.
2 2
(C) x y 4
(C) 8 .
2

AFA-EFOMM 223
Matemtica IV Assunto 1

1 ab a + b
10 (ITA-87) O valor de x > 0 que satisfaz a equao x = tan : (C)
12 1 + ab a b
1 + ab a b
(A) x = 4 3 . (D)
1 ab a + b
(B) x = 5 4 3 .
(C) x = 7 3 .
(D) x = 7 4 3 . 04 (ITA-2003) Para todo x real, a expresso cos2 ( 2 x ) sen2 ( 2 x ) sen x
(E) x = 9 4 3 . igual a:

5 (A) 24 sen( 2 x ) + sen(5 x ) + sen( 7 x )


11 Sendo y = sen cos , o valor numrico de y :
12 12 (B) 24 2 sen( x ) + sen(7 x ) sen( 9 x )
(C) 24 sen( 2 x ) sen(3 x ) + sen( 7 x )
1 3
(A) + (D) 24 sen x + 2 sen(5 x ) sen(9 x )
2 4
3 (E) 24 sen x + 2 sen(3 x ) + sen( 5 x )
(B)
2
1 05 Em um tringulo ABC, vale a relao 9BC2 + 9CA2 19AB2 = 0. Qual o
(C)
2 cot C
valor de cot A + cot B ?
(D) 3 +2

(E) 2( 3 + 1) 06 Sejam o lado de um polgono regular de n lados, r e R,


respectivamente, os raios dos crculos inscrito e circunscrito a este

12 (EN -1999) Coloque (F) falso ou (V) verdadeiro nas proposies abaixo polgono. Prove que r + R = cot .
2 2n
e assinale a opo correta.
07 Mostre que, se os ngulos de um tringulo ABC verificam a igualdade
I. (1 + cot x ) (1 cos x ) = 1x , x k , k 
2 2
sen4A + sen4B + sen4C = 0, ento o tringulo retngulo.

II. (1 + sec 4 x ) = 2 sec2 x + tan4 x x , x 2
+ k , k  tan2 x + tan2 y = 6

13 11 1 08 Resolva o sistema tan x tan y
III. sen
12
cos = 
12 4 tan y + tan x = 6

09
(A) F-V-V

(B) F-F-V a. Prove que: tan x tan x tan + x = tan 3 x .
(C) V-V-F 3 3
(D) V-V-V b. Usando (a), mostre que tan 20 tan 30 tan 40 = tan 10.
(E) V-F-V
10 Verifique as igualdades:
EXERCCIOS NVEL 2
a. tan 50 + cot 50 = 2 sec10.
3 3
3 + cos 4 x b. cos4 + cos4 = .
01 Demonstrar a identidade tan2 x + cot 2 x = 2 . 8 8 4
1 cos 4 x
c. tan 50 tan 40 = 2 tan 10.
02 (EFOMM-00) A funo cos a + cos 3a + cos 5a + cos 9a equivale a:
Equaes e inequaes trigonomtricas
(A) 4cos 4a cos 3a cos 2a
(B) 3cos 3a cos 2a cos a EXERCCIOS NVEL 1
(C) 2cos 2a 2cos a
(D) 3sen a cos 2a sen 2a 01 (EFOMM-95) Se x [0,2p], o nmero de solues da equao
(E) 2sen 3a cos a 2sen3x senx + 1 = cos2x igual a:

03 (EN-1997) Sabendo-se que tan x = a, tan y = b, pode-se reescrever (A) 1.


sen( 2 x ) + sen( 2 y ) (B) 2.
z= como:
sen( 2 x ) sen( 2 y ) (C) 3.
(D) 4.
1 ab a b (E) 6.
(A)
1 + ab a + b
1 + ab a + b
(B)
1 ab a b

224 Vol. 1
Trigonometria

02 (AFA-2000) Se (senx, sen2x, cosx) uma progresso geomtrica 5 5


estritamente crescente, com 0 < x < 2p, ento o valor de x : (C) { x | x }.
12 6
5
(A) (C) (D) { x | x }.
12 8 12 12
3 sec x cos x
(B) (D) 09 (EN-1997)Se x [0, 2], oconjuntosoluode <1
10 6
: 9 csc x sen x

03 (AFA-1998) O conjunto soluo, em , da equao (cos x)(sen 2x)


7 4
= (sen x)(1 + cos 2x), : (A) x : x , ,
6 3 6 3

(A)
5 4
(B) (B) x : x , ,
4 3 4 3
(C) {x l x = 2k p p /2, k Z}.
(D) {x l x = 2k p p /3, k Z}. 7 5
(C) x : x , ,
6 4 6 4
04 Resolver as equaes abaixo:
5 4
(D) x : x , ,
a. sen 2x = sen 5x. f. cos 2x + cos 8x = 0. 4 3 4 3
b. sen 3x = cos 4x. g. tan 5x = tan 2x.
c. sen 4x + sen x = 0. h. tan 5x + tan x = 0.
d. cos 5x = cos 7x. i. tan x tan 3x = 1.
10 (ITA- 2000) Para x no intervalo 0, , o conjunto de todas as solues
e. cos 3x + sen 5x = 0. 2

05 (EFOMM-01) O conjunto soluo da equao sen x + cos x = 1 : da inequao sen( 2 x ) sen 3 x + > 0 o intervalo definido por:
2
(A) S = { x / x = + 2 h, h Z}

(B) S = { x / x = h ou x = + 2 h, h Z} (A) < x < (D) <x<
10 2 4 2

(C) S =
x / x = 2 h ou x = + 2 h, h Z
2 (B) < x < (E) <x<
12 4 4 3
(D) S = { x / x = 2 h, h Z}

(C) < x <

3 6 3
(E) S = x / x = + 2 h, h Z
2
EXERCCIOS NVEL 2
3
06 (AFA -2003) Dado que sen x + cos x = , tem-se que cos x
3 4 01 (ITA-87) O nmero de razes reais da equao sen2x + sen4x + sen6x
vale:
+ sen8x + sen10x = 5 :

1 + 3 6 (A) um nmero maior do que 12.


(A) (C)
2 3 (B) zero.
2 6 (C) 2.
(B) (D) (D) 10.
3 6 (E) 1.
07 (AFA-02) O conjunto dos valores reais de x que tornam verdadeira a
desigualdade cos2 (x p) p : 1
02 (EN-2003) O nmero de solues reais da equao sen = x 2
igual a n; assim, pode-se concluir que: x
(A) { x / x ou x }
(B) { x / x } (A) n = 0
(B) n = 1
(C)
(C) n = 2
(D)
(D) n =3
(E) n > 3
1 2
08 (AFA-1998) O conjunto soluo da inequao senx cosx< ,
4 2
03 (ITA-88) Sobre a equao tan x + cot x = 2 sen( 6 x ), podemos afirmar
para 0 x p, : que:

(A) { x |

x < }. (A) apresenta uma raiz no intervalo 0 < x <
12 6 4
5
(B) { x | <x< }. (B) apresenta duas razes no intervalo 0 < x <
12 3 2

AFA-EFOMM 225
Matemtica IV Assunto 1

Funes trigonomtricas
(C) apresenta uma raiz no intervalo < x<
2
3 EXERCCIOS NVEL 1
(D) apresenta uma raiz no intervalo < x <
2
(E) no apresenta razes reais. 01 Calcule arcsen(sen150).

1 02 Calcule:
04 (ITA-88) Seja a equao sen3 x cos x sen x cos3 x = , onde m
um nmero real no nulo. Podemos afirmar que: m 1
a. y = arcsen .
2
(A) A equao admite soluo qualquer que seja m no-nulo. 3
(B) Se| m|< 4, esta equao no apresenta soluo real. b. y = arctan 3 + arccos .
2
(C) Se m > 1, esta equao no apresenta soluo real. 1
(D) Se| m|> 2, esta equao sempre apresenta soluo real. c. y = cot arcsen .
3
(E) Se m < 4, esta equao no apresenta soluo real. d. y = arcsen x + arccos x.
05 (AFA-2000) Os valores de m para os quais a equao 3
03 (EN) Seja x = arccos , x [0, ]. Ento, sen(2x) igual a:
2 (sen x cos x) = m2 2 admite solues, so: 5

(A) 1 m 1. (C) 0 m 2 . 24 1
(A) (D)
(B) 2 m 2. (D) 2 m 2 . 25 5
4 2
(B) (E)
06 (EN-1999) O produto das solues da equao 5 5
2 sen3 x + 5 cos2 x + 4 sen x + 2 tan2 x = 4 + 2 sec2 x no intervalo 16
5 (C)
25
12 , 6 :

2 2
04 (AFA) O valor de cotg arcsen
3
(A) (D)

(A) 2.
(B) (E) 2
(B) 2 2 .
(C)
(C) 2.
4
07 (ITA-88) A respeito da soluo da equao
(D) 3 2 .
sen x + 3 cos x = 2, 0 x < 2 , podemos afirmar que: 4

(A) existe apenas uma soluo no primeiro quadrante. 1 1


(B) existe apenas uma soluo no segundo quadrante. 05 (AFA-2000) O valor de sen arc cos + arc sen :
2 3
(C) existe apenas uma soluo no terceiro quadrante.
(D) existe apenas uma soluo no quarto quadrante.
(E) existem duas solues no intervalo. (A) 2 2 1 .
2
08 (AFA-2000) Os valores de , 0 < 2p, que satisfazem a (B) 2 6 1 .
1 6
desigualdade x2 + <sen, para todo x real, pertencem ao intervalo:
2 (C) 2 3 1 .
3

(A) 0 < < (D) 2 6 1.
2
6
(B) 0 < <
6 06 (AFA-1999) O valor real que satisfaz a equao arcsenx + arcsen2x
5 = p/2, para x pertencente ao intervalo (0,1), :
(C) <<
6
5 1 1
(D) < < (A) . (C) .
6 6 5 2

(B) 5 . (D) 2.
5 2

226 Vol. 1
Trigonometria

1 x 2 EXERCCIOS NVEL 2
07 (EN-2002) Seja f(x) = definida nos reais e seja g(x) = tan x
2 x
1+ x
01 (ITA) A soluo da equao arctanx + arctan = definida no
x +1 4
definido no intervalo aberto , . Se x ] p, p[, ento o valor da conjunto dos reais diferentes de 1 :
2 2
funo composta no nmero x/2 igual a: (A) 1.
(B) .
(A) cos (2x) (C) e 1.
(B) tan x (D) 2.
(C) sen x (E) 2 e 1.
(D) cos x
02 (ITA) Sendo z = cos (arctan(a2 + b2) + arccot (a2 + b2)), podemos
08 (EN-2002) Sejam A, B e C os pontos de interseo da curva com os afirmar que:
eixos coordenados conforme a figura abaixo, em que k e w so constantes
reais. (A) z = 0
(B) z = 1
y 3
(C) z =
B 2
y = k cos(wx) (D) cos (a2 + b2), se a2 + b2 1
(E) impossvel determinar o valor de z
A C x
03 (ITA) Seja k uma constante real e considere a equao em x: arcsen
Supondo que o tringulo de vrtices A, B e C tem 3p unidades de rea e 1+ x2
= k, x 0. Podemos afirmar que:
que k + w 14 = 0, o valor de (k w) : 2x

(A) 14. (A) Para cada k real, a equao admite uma nica soluo.
(B) 10. (B) Para cada k real, a equao admite duas solues.
(C) 10. (C) Existe k real tal que a equao admite uma infinidade de solues.
(D) 12. (D) No existe k real tal que a equao admita soluo.
(E) Existe k real tal que a equao admite uma nica soluo.
09 (ITA-80) Sobre a funo f(x) = sen2 x, podemos afirmar que:
04 (ITA) Seja f(t) = 4 + 3cos(pt) + 4sen(pt) a funo definida nos reais.
(A) uma funo peridica de perodo 4p. Sobre esta funo das alternativas abaixo correta?
(B) uma funo peridica de perodo 2p.
(C) uma funo peridica de perodo p. (A) f(t) funo par.
(D) uma funo peridica onde o perodo pertence ao intervalo (p, 2p). (B) f(t) funo mpar.
(E) no uma funo peridica. (C) o maior valor que f(t) assume 9.
(D) o maior valor que f(t) assume 3.
10 Determine o perodo das seguintes funes trigonomtricas: (E) o maior valor que f(t) assume 1/2.
x
(A) y = sen . (E) y = tan2 x. 05 (AFA) O grfico que melhor representa a funo y = |sen x + cos
3
x|, com 0 x < 2p, :
2x x
y = cos3 .
(B) y = 3 tan . (F)
3 4 2 y
(A) (C) y
(C) y = 4 3 sec(x). 2 2
(D) y = sen2 x.

11 (ITA-88) O conjunto imagem da funo f: [0,1] [0, p], f(x) = 1 1


3x 1
arccos :
2 x x
p 2p p 2p
2 2
(A) 0, , (D)
0, 3
4 3 (B) y (D) y
2 2
(B) [0, p] (E) 0, 2

3 1 1
(C) ,
4 4
x x
p 2p p 2p

AFA-EFOMM 227
Matemtica IV Assunto 1

06 (ITA-81) Seja g uma funo no nula dos reais nos reais que x
satisfaz, para todo x e y, g(x + y) = g(x) + g(y). Se f real for definida por (A) y + sen = 3.
2
f(x) = sen 2g( x ) , a 0, ento podemos garantir que: x
a (B) y + sen = 3 + 2/2.
2
(A) f peridica com perodo pa. (C) y + |cos2x| = 4.
(B) Para a = n, n natural, temos f(n) = 2sen(g(1)).
(C) Se g(1) 0, ento g(1) = f(0). x
(D) y + sen = 3 2/2.
(D) Se g(T) = pa, ento T perodo de f. 2
(E) g(T) = 2p, ento T perodo de f.
(E) y + |sen2x| = 3.
07 (AFA) Analise as alternativas seguintes e classifique-as como
verdadeiras (V) ou falsas (F). 09 (ITA) Considere os contradomnios das funes arco seno e arco
( ) O perodo e o conjunto-imagem da funo f: definida por
cosseno como sendo , e [0,p], respectivamente. Com respeito
1 1 1 2 2
f(x) = senx cosx so, respectivamente, 2p e , . funo f: [1,1] f(x) = acrsen x + arccos x, temos que:
4 4 4
( ) A funo y = 2 arccos4x tem por domnio o conjunto de todos os
(A) no crescente e mpar.
1 (B) no par nem mpar.
valores de x pertencentes a 0, 4, .
(D) injetora.
(E) constante.
( ) Para todo x , , o valor de (tg2x + 1) (sen2x 1) 1.
2 2
A opo que corresponde classificao anterior : cos( 2 x )
10 (AFA) Considere a funo real definida por y = e as
1 + sen( 2 x )
(A) F V F. seguintes afirmaes:
(B) V V F.
(C) F F V. I. A funo decrescente em todo seu domnio
(D) V F V.
II. O grfico da funo apresenta assntotas em arccos + k , k
2
08 (EN) A funo que melhor se adapta ao grfico abaixo :

III. A funo negativa em 0,
y 4
3
IV. A funo admite inversa em 0,
2

2 So verdadeiras somente as afirmaes contidas nos itens:

(A) I e II.
x (B) II e III.
3p p p p p 3p (C) III e IV.
4 2 4 4 2 4 (D) I e IV.

RASCUNHO

228 Vol. 1
Trigonometria

RASCUNHO

AFA-EFOMM 229
Introduo geometria plana euclidiana A ssunto
1
Matemtica V

1. Conceitos primitivos e axiomas


Na geometria euclidiana, trabalha-se com noes de elemento e
conjunto, embora denotemos por alguns nomes diferentes. Os conceitos
primitivos so os objetos com os quais iremos trabalhar, mas no
definiremos formalmente. So eles:
o ponto, um objeto adimensional, e que deve ser lidado como um
elemento.
a reta, um objeto de dimenso 1, que um conjunto de pontos.
o plano, um objeto de dimenso 2, tambm um conjunto de pontos.

A partir daqui, comearemos o estudo da geometria plana. Todos os


objetos estaro contidos em um mesmo plano, ou seja, sero coplanares.

2. Outros objetos iniciais e definies

Segmento de reta AB o conjunto de pontos que esto entre os


pontos A e B. Chamamos A e B de extremidades do segmento. Todo ponto
que est entre A e B est na reta AB, logo o segmento de reta AB est

contido na reta AB . A cada segmento de reta, associamos uma medida,
que um nmero real positivo. A unio de dois segmentos adjacentes
tem por medida a soma das medidas de ambos.

Formalmente, dados dois pontos A e B, chamamos de semirreta AB o
conjunto dos pontos X que esto entre A e B ou so tais que B est entre A
e X. Para entender, pense em A dividindo a reta em dois conjuntos infinitos
Apesar de no podermos defini-los, podemos estabelecer notaes e
em sentidos diferentes, um dos quais contm o ponto B. Esse conjunto
relaes entre eles, atravs dos axiomas. Os axiomas so como regras 
do jogo, as verdades que no podemos provar, e que servem pra iniciar a semirreta AB . O vrtice (ou origem) dela o ponto A. Quando duas
o estudo desse sistema. semirretas possuem o mesmo vrtice e a sua unio a reta suporte delas,
dizemos que so semirretas opostas.
Existem cinco grupos principais de axioma na geometria euclidiana,
segundo a axiomatizao de Hilbert. Seguem alguns dos principais axiomas O ponto M entre A e B tal que os segmentos AM e MB tm a mesma
da geometria: medida chamado de ponto mdio.

Existem infinitos pontos;


dois pontos distintos determinam uma reta que os contm;
numa reta existem infinitos pontos, fora dela tambm;
trs pontos distintos que no estejam em uma mesma reta determinam
um plano que os contm;
dados uma reta e um ponto fora dela, existe e nica uma segunda Dizemos que um conjunto do plano convexo se, e somente se,
reta que contm o ponto dado e no intersecta a reta dada, embora para todo par de pontos A e B do conjunto, o segmento AB est contido
esteja no mesmo plano. [Ax. De Euclides] no conjunto tambm. Caso existam dois pontos do conjunto tais que o
segmento com extremidade neles no est contido no conjunto, dizemos
que o conjunto no-convexo.
A essa reta, chamamos de paralela. Foram suprimidos alguns axiomas,
para facilitar o entendimento.

Conjunto convexo Conjunto


no-convexo

230 Vol. 1
Introduo geometria plana euclidiana

EXERCCIO RESOLVIDO Segue a nomenclatura dos ngulos quanto s suas medidas:


01. Sejam A, B e P pontos colineares tais que B est entre P e A. Sejam
M e N pontos mdios dos segmentos AP e PB respectivamente. Calcule ngulo Agudo: ngulo maior que 0 e menor que 90.
a medida de MN, sabendo as medidas de AP = a, PB = b. ngulo Reto: ngulo de 90.
ngulo Oblquo: ngulo maior que 90 e menor que 180.
Soluo:
ngulo Raso: ngulo de 180.
Como AP = a, e M mdio de AP, ento MP = a/2. Como
PB = b, e N mdio de PB, ento NP = b/2. Agora, como N est ngulo Cncavo ou Reentrante: ngulo maior que 180 e menor que 360.
entre M e P, tem-se que MN = MP PN = (a b)/2. ngulos complementares: ngulos que somam 90.
ngulos suplementares: ngulos que somam 180.
3. ngulos ngulos replementares: ngulos que somam 360.
ngulos explementares: ngulos cuja diferena de 180.
Quando duas semirretas OA e OB possuem mesmo vrtice, elas
determinam uma regio do plano que chamamos de ngulo [denotamos
AB], e as semirretas chamamos de lados do ngulo. Associamos a 4. Paralelismo e teorema angular de Tales
cada ngulo uma medida, que um nmero real positivo. Quando dois
ngulos tm por interseo apenas um lado em comum, dizemos que Com o contedo de tringulos, e aps a formalizao de alguns
so adjacentes, e a medida da unio a soma das medidas. O ngulo teoremas, podemos concluir o seguinte teorema: na figura, se r//s, ento
formado por duas semirretas opostas associado medida em graus os ngulos a e b so congruentes. Na verdade, vale a recproca tambm,
de 180. Existem outras unidades de medida, como o radiano e o grado. a qual servir como um bom critrio de paralelismo entre retas.
Para converter, s fazer uma regra de trs com a seguinte equivalncia:
180 = rad. = 200gr
Dado um ngulo AB, chamamos a semirreta interna a ele OX de
bissetriz, se, e somente se, os ngulos XA e XB so congruentes, isto r
, tm a mesma medida.

B
^ ^
AO X = XO B
^
OX bissetriz de AO B
x
O s

A Na figura, r//s =

Na figura, temos que so ngulos congruentes:


Dizemos que dois ngulos so opostos pelo vrtice quando os lados
de um so semirretas opostas aos lados do outro. Prova-se que se dois a = c = e = g, b = d = f = h.
ngulos so opostos pelo vrtice, ento so congruentes.

Os ngulos e so opostos pelo vrtice =

AFA-EFOMM 231
Matemtica V Assunto 1

Alguns pares de ngulos, como na figura anterior, recebem um nome 03 Efetue:


pela posio relativa s retas paralelas e transversal. So eles:
a. 234519 + 373243
Alternos internos: (c, e), (d, f) b. 871832 543742
Alternos externos: (a, g),(b, h) c. 52347 4
Colaterais internos: (c, f),(d, e) d. 562533 3
Colaterais externos: (a, h), (b, g)
04 Sendo dado um ngulo de medida a, escreva simplificadamente uma
Correspondentes: (a, e),(b, f),(c, g),(d, h).
frmula que calcule:

Como consequncia, tem-se o Teorema Angular de Tales: em a. o suplemento de a;


um tringulo, a soma dos ngulos internos constante e igual a 180. b. o complemento da metade de a;
Analogamente, podemos concluir a relao do ngulo externo: cada ngulo c. o replemento de um tero do suplemento de a;
externo de um tringulo mede a soma dos outros dois ngulos internos d. o suplemento do dobro do complemento da metade de a.
no-adjacentes a ele.
05 Um ngulo igual ao dobro do complemento do seu qudruplo. Quanto
mede esse ngulo?

06 Dois ngulos suplementares so tais que um o triplo do complemento


do outro. Quanto vale a razo entre esses ngulos?

07 Dois ngulos AB e BC so adjacentes, e o ngulo AC mede 120.


Calcule a medida do ngulo formado pelas bissetrizes de AB e BC.

08 Em um relgio de ponteiro convencional, qual o ngulo formado


pelos ponteiros quando marcam:

a. 5:20h
b. 10:44h

09 As semirretas OA, OB, OC e OD formam os ngulos adjacentes AB, BC


e CD. Sabendo que OA e OD so semirretas opostas, e que o ngulo BC
mede 130, quanto mede o ngulo formado pelas bissetrizes de AB e CD?

10 Nas figuras, prove as relaes dos bicos.

EXERCCIOS NVEL 1

01 So dados os pontos A, B, C, D e E, nessa ordem, sobre uma reta.


Sabe-se que AB + CD = 3 BC e DE = AB. Sendo M mdio de BE, tem-se
que MD = 2 e AE = 16. Calcule MC.

(A) 2
(B) 3
(C) 4
(D) 5
(E) 6

02 So dados os pontos A, B, C e D, nessa ordem, sobre uma reta, de


forma que AD = 20 e BC = 12, sendo AB menor que a metade de CD.
Calcule a distncia entre os pontos mdios de AB e CD.

232 Vol. 1
Introduo geometria plana euclidiana

11 (EPCAR-2004) Considere as retas r e s (r//s) e os ngulos , e da 05 Pelo vrtice de um ngulo AB reto, traa-se uma reta r qualquer,
figura abaixo: externa a ele. Calcule o ngulo formado pelas bissetrizes dos ngulos
agudos que as semirretas OA e OB formam com a reta r.
r

06 Os ngulos AB e BC so adjacentes, sendo a medida do primeiro
igual a 70. Calcule o ngulo formado pelas bissetrizes dos ngulos AC
e BC.

07 Aps ver um relgio de ponteiro s 13:00, o ponteiro das horas
s percorreu um ngulo de 42. Qual o horrio indicado pelo relgio aps
esse movimento?
Pode-se afirmar que:
08 Aps as 15:00, qual o primeiro horrio em que os ponteiros das
(A) + + = 270 horas e dos minutos formam um ngulo de 130?
(B) + + = 180
(C) + = 09 As semirretas OA, OB, OC e OD formam ngulos adjacentes AB, BC,
(D) + = + 90 CD e DA, nessa ordem, tais que os trs primeiros so proporcionais a
1, 3 e 6 respectivamente. Sabe-se que OD semirreta oposta bissetriz
12 Na figura, prove que vale a relao x = a + b + c. do ngulo BC. Calcule o ngulo AD.

10 Os ngulos AB e BC so adjacentes, sendo AC = 100. Sendo


OX, OY e OZ semirretas bissetrizes, respectivamente, de AB, BC e XY,
e sendo BZ=10, ento o maior dentre AB e BC mede:

(A) 60.
(B) 70.
(C) 80.
(D) 90.
(E) faltam dados.
EXERCCIOS NVEL 2
11 Dados os ngulos adjacentes AB, BC, CD e DA, traam-se as
bissetrizes OX, OY e OZ dos ngulos AB, CD e XY, respectivamente. Sabe-se
01 So dados os pontos A, B, C e D, nessa ordem, sobre uma reta.
que XC + XD 4 BZ = 80 e que BZ mede 50. Calcule o ngulo CD.
Sabe-se que AB e CD so congruentes. Prove que o ponto mdio de AD
tambm ponto mdio de BC.
(A) 10.
(B) 20.
02 Sobre uma reta marcam-se os pontos M, A e B. Sendo O o ponto mdio (C) 40.
de AB, calcule k para que valha a seguinte relao: MA2 + MB2 = k (MO2 + (D) 60.
AO2) (E) 80.

03 O suplemento da tera parte de um ngulo excede o complemento do 12 (OBM) Trs quadrados so colados pelos seus vrtices entre si e a
seu triplo em 130. Quanto mede o replemento do quntuplo desse ngulo? dois bastes verticais, como mostra a figura.

04 As medidas de quatro ngulos replementares esto em progresso


aritmtica. Analise as afirmativas a seguir:

I. Dois deles so complementares.


II. Existe um que o dobro do outro.
III. Existem dois deles que so suplementares.

Quais so verdadeiras?

(A) apenas I. A medida do ngulo x :


(B) apenas II.
(C) apenas III. (A) 39.
(D) II e III. (B) 41.
(E) I e II. (C) 43.
(D) 44.
(E) 46.

AFA-EFOMM 233
Matemtica V Assunto 1

13 Na figura que segue, quanto vale a soma a + b + c + d + e?

15 Quantas vezes os ponteiros das horas e dos minutos so perpendiculares


em um dia de funcionamento?

(A) 180. (D) 540. (A) 44.


(B) 270. (E) faltam dados. (B) 46.
(C) 360. (C) 48.
(D) 23.
14 Nas figuras, calcule a medida de x: (E) 24.

RASCUNHO

234 Vol. 1
Polgonos A ssunto
2
Matemtica V

Neste bloco, voc ver uma pequena introduo ao conceito de Pode ser ainda calcular o nmero de diagonais de um polgono de
polgonos. Aqui analisaremos principalmente a parte qualitativa de gnero n, pela seguinte expresso:
polgonos. Mais adiante, com ferramentas mais avanadas, voc poder
deduzir as principais relaes mtricas e angulares envolvidas nas questes n( n 3)
D=
mais comuns do assunto. 2
Dem: De fato que de cada vrtice do polgono partem n 3 retas
1. Definio e nomenclatura uma vez que, um vrtice no se liga a si mesmo, nem aos seus dois
Chamamos de polgono a regio delimitada pela unio de segmentos vizinhos(lados).
no-colineares consecutivamente, desde que tal linha seja fechada. Como o polgono tem gnero n, teramos n(n 3) reatas, porm cada
Aos segmentos chamaremos lados, aos extremos dos segmentos reta contada duas vezes(uma em cada vrtice).
chamaremos vrtices, e ao nmero de lados, que igual ao nmero de
vrtices, chamaremos de gnero do polgono. Qualquer segmento com 3. Polgonos regulares
extremidades em dois vrtices chamado de diagonal, desde que no
Dizemos que um polgono convexo equiltero quando todos os seus
seja um lado do polgono.
lados so congruentes.
Dizemos que um polgono simples
quando no existem dois lados no-
consecutivos que se intersectem
mutuamente. Caso existam dois lados
no-consecutivos com interseo,
dizemos que o polgono complexo.
Caso seja simples, definimos como
ngulo interno qualquer ngulo formado
por dois lados consecutivos, definido na
regio interna ao polgono. Observe que
o nmero de ngulos internos igual ao gnero do polgono. Dizemos que um polgono convexo equingulo quando todos os
seus ngulos internos so congruentes.
Se o polgono simples, ento podemos classific-lo como convexo
ou no-convexo. Caso seja convexo, chamamos de ngulo externo cada
menor ngulo formado por um lado e um prolongamento de outro lado
adjacente. Considere sempre que para cada vrtice existe um ngulo
externo, j que na verdade so dois ngulos opostos pelo vrtice, logo
so congruentes. Dessa maneira, o nmero de ngulos externos tambm
igual ao gnero do polgono.

Observe pelas figuras que um polgono equiltero no necessariamente


Pentgono ABCDE Pentgino IJFGH no Pentgono MLKPN
equingulo, e vice-versa. Dessa maneira, existem os polgonos regulares:
complexo convexo convexo
so os polgonos equilteros e equingulos simultaneamente. Todo
2. Frmulas importantes polgono regular admite um centro, que um ponto que equidista dos
vrtices, bem como dos lados. Logo, sempre existe um crculo circunscrito
Por triangulao, que consiste em quebrar um polgono em tringulos a um polgono regular, bem como um inscrito nele.
atravs de suas diagonais, podemos concluir que a soma dos ngulos
internos de um polgono simples dada pela frmula:
S = 180o (n 2),
sendo n o gnero do polgono.

Obs.: fcil provar que todo polgono simples tringulorizvel por


induo.
Para polgonos convexos, a soma de
um ngulo interno com um externo sempre
ser um ngulo raso, logo podemos concluir
que, para polgonos convexos, a soma
dos ngulos externos de cada vrtice
constante e igual a 360 graus.

AFA-EFOMM 235
Matemtica V Assunto 2

Num polgono regular, todos os ngulos internos so iguais a 02 ABCDE um pentgono regular, e ABIJK outro pentgono regular.
Calcule o ngulo AC.
180o ( n 2) 360o
i = , e todos os ngulos externos so iguais a e =
n n 03 Em um quadriltero ABCD convexo, os ngulos internos em A e B
medem 120 e 80 respectivamente. Calcule o ngulo agudo formado
. Ao traar as diagonais, cada ngulo interno fica dividido em partes pelas bissetrizes internas nos vrtices C e D:
congruentes, iguais metade do ngulo externo.
(A) 60.
(B) 65.
(C) 70.
(D) 75.
(E) 80.

04 Um polgono regular tem 20 diagonais. Quanto mede seu ngulo


interno?

05 Aumentando o nmero de lados de um polgono em 3 unidades, seu


nmero de diagonais aumenta em 21. Determine o nmero de diagonais
do polgono.

06 Calcule o nmero de diagonais que no passam pelo centro de um


polgono regular de 2n lados.
No polgono regular de gnero n temos ainda os seguintes fatos conhecidos:
07 A soma dos gneros de dois polgonos 16. Se os seus nmeros de
diagonais diferem de 26, a diferena entre seus gneros :
n
n par: o nmero de diagonais que passam pelo centro e que
2 (A) 1.
n ( n 3)
n n ( n 4) (B) 2.
no passam = .
2 2 2 (C) 3.
n mpar: nenhuma diagonal passa pelo centro. (D) 4.
(E) 5.
EXERCCIOS RESOLVIDOS
08 A soma dos ngulos internos de dois polgonos regulares 1980, e
01 ABCD um quadrado, e ABP e BCQ so tringulos equilteros, a diferena entre seus gneros 3. Determine os polgonos.
o primeiro interno ao quadrado, e o segundo externo a ele. Prove que
D, P e Q so colineares. 09 Dois polgonos regulares so tais que o gnero de um excede o do outro
em 3 unidades, e o nmero de diagonais de um o qudruplo do nmero de
Soluo:
diagonais do outro. Determine os ngulos internos desses dois polgonos.
Considerando que ABCD e ABP possuem o mesmo lado, ento o
tringulo APD issceles, e =30 implica PDA=75. Logo PDC=15. 10 Um polgono regular convexo tem o seu nmero de diagonais expresso
Analogamente, considerando ABCD e BCQ, tem-se que o tringulo QCD por n2 10n + 8, onde n o seu nmero de lados. O seu ngulo interno
issceles, com C=150. Logo QDC=15. Como PDC e QDC so x tal que:
iguais a 15, ento os pontos D, P e Q so colineares.
(A) x < 120.
02 Um polgono regular de gnero desconhecido ABCDEF tal que o (B) 120 < x < 130.
ngulo ACE mede 150. Calcule o nmero de diagonais do polgono. (C) 130 < x < 140.
(D) 140 < x <150.
Soluo: (E) x > 150.
ABC um tringulo issceles. Sendo x=BCA, tem-se que o ngulo
BCD mede x + 150 + x. Alm disso, o ngulo externo do polgono
EXERCCIOS NVEL 2
dado por 2x. Como o ngulo interno suplementar do externo, tem-se
que (2x + 150) + 2x = 180, logo 2x = 15. Sendo n o gnero, tem-
-se que n = 360 / 15 = 24. Logo D=24 21 / 2 = 252 diagonais. 01 ABCDE um pentgono regular, e ABI um tringulo equiltero, sendo
I interno ao pentgono. Calcule o ngulo CD.

EXERCCIOS NVEL 1 02 Em um polgono convexo de 15 vrtices, so escolhidos quatro de


seus vrtices, sem que haja dois consecutivos. Quantas so as diagonais
01 ABCD um quadrado e ABP um tringulo equiltero. Calcule o ngulo que partem desses 4 vrtices?
CDP nos seguintes casos:
03 Em um polgono convexo, dois ngulos internos medem 130, e todos
a. o tringulo interno ao quadrado.
b. o tringulo externo ao quadrado. os outros medem 128. Determine o gnero desse polgono.

236 Vol. 1
Polgonos

04 Em um polgono regular, as mediatrizes de dois lados consecutivos 11 (ITA-2003) Considere trs polgonos regulares tais que os nmeros que
formam um ngulo de 20. Calcule o ngulo formado entre as duas expressam a quantidade de lados de cada um constituam uma progresso
diagonais menores que partem do mesmo vrtice. aritmtica. Sabe-se que o produto desses trs nmeros igual a 585 e que
a soma de todos os ngulos internos dos trs polgonos igual a 3780o. O
nmero total das diagonais nesses trs polgonos igual a:
05 As diagonais de um polgono regular convexo so medidas, e
apresentam os valores: {m1, m2, m3, ... , m14}. Calcule o ngulo externo
(A) 63.
desse polgono, sabendo que, em graus, ele apresenta um valor inteiro. (B) 69.
(C) 90.
06 Em um polgono regular ABCDE..., as bissetrizes externas traadas de (D) 97.
A e C so perpendiculares. Qual o gnero do polgono? (E) 106.

07 Dois polgonos regulares so tais que a razo entre seus ngulos 12 (ITA-1998) Considere as afirmaes sobre polgonos convexos:
internos 5/4, e a razo entre seus ngulos externos 1/2. Calcule o
I. Existe apenas um polgono cujo nmero de diagonais coincide com
nmero de diagonais do polgono com maior gnero.
o nmero de lados.
II. No existe polgono cujo nmero de diagonais seja o qudruplo do
08 Um ladrilho com formato de polgono regular tal que, rotacionado em nmero de lados.
torno do centro de 40 no sentido horrio ou de 60 no sentido anti-horrio, III. Se a razo entre o nmero de diagonais e o de lados de um polgono
fica encaixado perfeitamente no espao vago deixado antes de rotacionar. um nmero natural, ento o nmero de lados do polgono mpar.
Determine o nmero mnimo de lados que tal ladrilho pode possuir.
Ento:
09 Dado um polgono convexo regular ABCDEF... de gnero desconhecido,
considere as bissetrizes de seus ngulos internos A e D. Sabendo que o ngulo (A) Todas as afirmaes so verdadeiras.
3 (B) Apenas I e III so verdadeiras.
formado por essas bissetrizes igual a da soma de todos os ngulos (C) Apenas I verdadeira.
40
internos do polgono, pede-se para calcular quantas diagonais ele possui. (D) Apenas III verdadeira.
(E) Apenas II e III so verdadeiras.
10 O nmero de gneros de polgonos regulares tais que quaisquer duas
de suas diagonais, que passam pelo seu centro, formam entre si ngulo
expresso em graus por um nmero inteiro, :

(A) 17. (D) 23.


(B) 18. (E) 24.
(C) 21.

RASCUNHO

AFA-EFOMM 237
Tringulos A ssunto
3
Matemtica V

1. Definio e propriedades iniciais


Tringulo o polgono de gnero 3. No possui diagonais, e sempre
convexo. Possui portanto trs lados, trs vrtices, trs ngulos internos
[que somam 180] e trs ngulos externos.

Podemos classificar os tringulos quanto s medidas dos lados:

I. Escaleno: todos os seus lados so diferentes;


II. Issceles: possui pelo menos dois lados iguais;
III. Equiltero: possui todos os lados iguais. Uma desigualdade muito til a seguinte: num tringulo, o maior
lado sempre oposto ao maior ngulo, e o menor lado sempre oposto
Podemos classific-los tambm quanto aos ngulos internos: ao menor ngulo.

I. Acutngulo: todos os seus ngulos internos so agudos; 3. Congruncia de tringulos


II. Retngulo: possui um ngulo interno reto; Dizemos que dois tringulos so congruentes se, e somente se, os
III. Obtusngulo: possui um ngulo interno obtuso. lados e os ngulos internos de um so homologamente congruentes aos
lados e ngulos internos do outro. Formalmente, dizemos:
Se um tringulo retngulo, chamamos o lado oposto ao ngulo reto
de hipotenusa, e os outros dois lados de catetos.
ABC = DEF A = D
, B
 = E , C
 = F , AB = DE, AC = DF, BC = EF.

Para demonstrar
que dois tringulos so
congruentes, basta testar
se vale um dos cinco
casos de congruncia que
seguem:

I. LAL Lado/ngulo/lado: Tringulos com par de lados iguais formando


2. Desigualdade triangular teorema da envolvente ngulos congruentes so congruentes entre si.
A desigualdade triangular estabelece o seguinte: dados dois pontos,
x
A e B, e um ponto X qualquer varivel, sempre vale que AB AX + XB,
com igualdade se, e somente se, X est entre A e B. Essa desigualdade a
pode ser estendida: dada uma poligonal fechada, um lado menor que a y
soma de todos os outros lados.
II. ALA ngulo/lado/ngulo: Tringulos com par de ngulos iguais com
os lados comuns a eles congruentes so congruentes entre si.

x a
b

III. LAAo Lado/ngulo/ngulo oposto: Tringulos com lado, ngulo


A partir disso, podemos concluir a condio de existncia de um tringulo: adjacente ao lado e ngulo oposto a ele congruente um aos do outro
dados os lados de um possvel tringulo, ele existe se, e somente se: so congruentes.
a < b + c x a

b < c + a a b < c < a + b b

c < a + b
IV. LLL Tringulos com todos os lados congruentes um aos do outro
Alm do mais, deduz-se o Teorema da Envolvente: se dois caminhos so congruentes.
convexos de A para B so tais que a regio definida por um [o envolvente] y
contm a regio definida pelo outro [o envolvido], ento o comprimento
daquele ser maior que o desse. z
x

238 Vol. 1
Tringulos

V. 90HC Caso especial para tringulos retngulos: Tringulos


retngulos que possuam hipotenusas iguais e um cateto de um igual
a um do outro so congruentes.

a
b

Obs.: LLA no caso de congruncia!


Os tringulos ABC e ABD
possuem dois pares de lados d. Bissetriz externa [e o exincentro]: se o tringulo ABC escaleno,
em comum e os ngulos ento existe D sobre a reta BC tal que AD bissetriz do ngulo externo
opostos a um deles iguais. em A. Chamamos AD de bissetriz externa. So trs bissetrizes externas,
Eles no so congruentes: um uma para cada vrtice. Duas retas bissetrizes externas e uma interna
est dentro do outro do vrtice remanescente so concorrentes num ponto chamado
exincentro relativo quele vrtice. So trs exincentros, e eles so
4. Cevianas notveis pontos notveis centros de crculos tangentes s retas suportes dos lados do tringulo.
Dado um tringulo ABC, dizemos que o segmento AD uma ceviana
se o ponto D est sobre a reta suporte do lado BC. Caso D esteja sobre o Mediatriz
lado BC, dizemos que AD ceviana interna. Caso contrrio, AD ceviana A mediatriz de um segmento AB
externa. a reta perpendicular a AB no
seu ponto mdio. Num tringulo,
Algumas cevianas possuem propriedades impor tantes, e tm as mediatrizes dos lados nem
nomenclatura especial. sempre so cevianas, j que no
a. Mediana [e o baricentro]: passam necessariamente pelo
se M ponto mdio de vr tice oposto. Porm, no caso
BC, chamamos AM de do tringulo, as mediatrizes dos
mediana relativa ao lado lados so concor rentes num
BC. Cada tringulo possui ponto chamado circuncentro, que
trs medianas, que so equidista dos vrtices, e, portanto,
concorrentes num ponto centro de uma circunferncia
chamado baricentro. O circunscrita ao tringulo.
baricentro G divide uma
mediana AM na razo
AG : GM = 2. Traadas 5. Tringulo issceles
as medianas, o tringulo Podemos provar que um tringulo tem uma das propriedades abaixo
original fica dividido em seis se, e somente se, ele issceles. Seguem:
tringulos de reas iguais.
Os ngulos das bases so congruentes;
a altura do vrtice principal tambm mediana;
b. Bissetriz interna [e o incentro]: se D sobre o lado BC tal que AD
a altura do vrtice principal tambm bissetriz;
bissecta o ngulo interno em A, chamamos AD de bissetriz interna
a bissetriz do vrtice principal tambm mediana.
relativa ao vrtice A, ou relativa ao lado BC. Cada tringulo possui
trs bissetrizes internas, que so concorrrentes num ponto chamado
incentro. O incentro equidista dos lados do tringulo, logo centro de
uma circunferncia inscrita no tringulo.

c. Altura [e o ortocentro]: se D sobre a reta BC tal que AD


perpendicular a BC, dizemos que AD altura relativa a BC. A altura pode
ser uma ceviana externa ou at mesmo um lado, como nas figuras. As
trs alturas traadas a partir de cada vrtice so concorrentes num
ponto chamado ortocentro.

AFA-EFOMM 239
Matemtica V Assunto 3

6. Lugares geomtricos iniciais b. Mediatriz: dado um segmento AB, o LG dos pontos P que equidistam
de A e B a mediatriz de AB.
Dada uma propriedade , dizemos que certo conjunto o lugar
geomtrico (LG) de se, e somente se, todos os pontos que satisfazem
a esto no conjunto, e vice-versa. Inicialmente, podemos deduzir trs
importantes LGs.
P est na mediatriz de AB,
a. Par de retas bissetrizes: dadas duas retas concorrentes r e s, o LG logo PA = PB
dos pontos P que equidistam de r e s o par de retas bissetrizes dos
ngulos formados pelas retas.

c. Par de paralelas: dada uma reta r e uma distncia d, o LG dos pontos


P que distam d da reta r o par de paralelas a uma distncia d.

P e Q pertencem a r e r paralelas distando x de r.


dist(P, r) = PP = x = QQ = dist(Q, r)

EXERCCIOS RESOLVIDOS
01 Seja P interno ao tringulo ABC. Prove que AP + PB < AC + CB. Soluo:
[Teorema da Envolvente] Observe que os tringulos ABY e XBC so congruentes pelo caso
LAL. (AB = XB, BY = BC e os ngulos B medem 120). Logo, as
Soluo: medianas relativas aos lados AY e XC so congruentes [ou seja, BM
Considere prolongar o segmento AP at X sobre BC. Por = BN]. Alm disso, o ngulo entre elas de 60 [por argumento de
desigualdade triangular temos: rotao], logo o trigulo BMN equiltero.
03 ABC um tringulo em que =120. Sejam D, E e F ps das
No tringulo ACX: AP + PX < AC + CX. bissetrizes internas de A, B e C, respectivamente. Prove que:
No tringulo PXB: PB < PX + XB. a. E exincentro do tringulo ABD, bem como F, de ACD.
b. tringulo EFD retngulo [em D].
Somando e usando a lei do corte, tem-se a concluso de que um
caso particular do teorema da envolvente. Usando a ideia de prolongar Soluo:
o segmento, podem-se provar as verses poligonais generalizadas. Considerando o tringulo ABD, tem-se que BE bissetriz interna e
que AE bissetriz externa [j que, prolongando AB de AX, tem-se ED =
02 A, B e C so pontos colineares, com B entre A e C. Constroem-se, EX = 60]. Logo, E exincentro do tringulo ABD. Consequentemente,
num mesmo semiplano gerado por AB, os tringulos equilteros ABX DE bissetriz externa. Analogamente, F exincentro do tringulo ACD,
e BCY. Sendo M e N mdios de AY e CX, prove que o tringulo BMN e DF bissetriz externa desse tringulo. Como os ngulos ADB e ADC
equiltero. so suplementares, essas bissetrizes DE e DF so perpendiculares
entre si. Logo, o tringulo EFD retngulo em D.

EXERCCIOS NVEL 1

01 Em um tringulo ABC, tomam-se sobre os lados AB e BC os pontos Quantas afirmativas so verdadeiras?


D e E, respectivamente, tais que BD = BE, e a medida dos ngulos BCD
e BAE so iguais. Analise as afirmativas: (A) Nenhuma.
(B) Apenas uma.
I. os tringulos BEA e BDC so congruentes; (C) Apenas duas.
II. os ngulos BDC e BEA so congruentes; (D) Apenas trs.
III. os segmentos BE e AD so congruentes; (E) Todas.
IV. os segmentos CD e AE so congruentes.

240 Vol. 1
Tringulos

02 Dadas as seguintes proposies, analise se so elas verdadeiras (V) 09 ABC um tringulo no qual o ngulo interno em A o dobro do em B.
ou falsas (F): Sejam X e Y pontos sobre BC e AC tais que AB = AX = XY = YC. Calcule
os ngulos do tringulo ABC.
I. Dois tringulos retngulos so congruentes se possuem dois lados
congruentes.
10 Dado um tringulo ABC, marca-se um ponto D sobre AC de forma que
II. Se em um quadriltero ABCD, BC = CD, e BC = DC, ento os
AB = AD. Sabendo que o ngulo interno em B excede o em C em 30,
tringulos ABC e ADC so congruentes.
III. Se no quadriltero ABCD, AC = BD e os ngulos ABD e ACD so retos, calcule a medida do ngulo CBD.
conclumos que AB = CD.
EXERCCIOS NVEL 1
Ento, tem-se:
01 Dado um tringulo ABC, sobre os lados AC e BC so marcados os
(A) V-F-V. pontos M e N, respectivamente, tais que AB = MC, MB = MN, e os ngulos
(B) F-V-V.
ABM e CMN so congruentes. Sabendo que BC = 50, quanto vale o
(C) V-V-V.
ngulo ACB?
(D) V-V-F.
(E) F-F-V.
(A) 50.
03 Em um tringulo ABC issceles, dois lados medem 14 cm e 4 cm. (B) 40.
Calcule o permetro do tringulo. (C) 60.
(D) 25.
(A) 18 cm.
(E) 75.
(B) 22 cm.
(C) 32 cm.
(D) Podem ser 22 cm ou 32 cm. 02 Na figura, os tringulos ABC e CDE so congruentes, de forma que
(E) Faltam dados. AC = CD. Se EC = 5, EF = 2, ento quanto mede AF?

04 Entre que valores est compreendido um dos lados de um quadriltero


que possui lados medindo 3 cm, 5 cm e 11 cm?

05 Em um tringulo ABC, calcule, em funo do ngulo interno em A, os


ngulos formados:

(A) pelas bissetrizes internas em B e C.


(B) pelas alturas traadas de B e C.
(C) pelas bissetrizes externas de B e C.
(D) pelas bissetrizes interna em B e externa em C. (A) 2.
(B) 3.
06 ABC um tringulo issceles tal que AB = AC = 13, BC = 10. Sobre (C) 4.
o lado BC, toma-se um ponto P, e sobre os lados AB e AC tomam-se X (D) 5.
e Y, respectivamente, tais que PX // AC e PY // AB. Calcule a soma PX + (E) 7.
PY.

07 Na figura, tem-se AB = BC e AC = CD = DE = EF. Calcule . 03 Na figura, AB = BD e os tringulos ABC e BDE so congruentes.


Calcule a razo entre os ngulos BCA e DAC.

08 ABC um tringulo issceles com AB = AC. Sobre os lados BC e


AC, tomam-se os pontos P e Q, de forma que AP = AQ. Calcule o ngulo (A) 1:1.
QPC, sabendo que BP = 30. (B) 3:2.
(C) 1:2.
(A) 10. (D) 1:3.
(B) 15. (E) 2:1.
(C) 20.
(D) 25.
(E) 30.

AFA-EFOMM 241
Matemtica V Assunto 3

04 Considere os tringulos ABC e ABC, sobre os quais so feitas as 09 ABC um tringulo equiltero. Prolonga-se BC de um segmento BP,
seguintes afirmaes: de forma que o ngulo APB mea 20. Sobre o segmento AP, toma-se um
ponto Q tal que o tringulo PQC issceles. Calcule a medida do ngulo
I. Se AB = AB, BC = BC e = , ento os tringulos so QBA.
congruentes.
II. Se AB=AB, BC = BC e B = B, ento os tringulos so congruentes. 10 Sobre os lados AB e AC de um tringulo ABC, tomam-se os pontos M
III. Se AB = AB, BC = BC, = , e BC > AB, ento os tringulos e N, tais que MN // BC e MN passa pelo incentro de ABC. Calcule a medida
so congruentes. de MN, sabendo que AB = 10, BC = 11 e AC = 12.

As afirmaes verdadeiras so:


11 ABC um tringulo equiltero, X um ponto interno, e M, N e P esto
sobre os lados AB, AC e BC, de forma que XM // AC, XN // BC e XP // AB.
(A) apenas II.
Sabendo que XM = 3, XN = 4 e XP = 5, calcule o lado do tringulo ABC.
(B) apenas I e II.
(C) apenas II e III.
(D) apenas I e III. 12 Em um tringulo ABC, AB = AC, e o ngulo = 40. Tomam-se os
(E) todas. pontos D e E sobre AB e AC, respectivamente, tais que os ngulos DCA e
EBC medem 15 e 35, nessa ordem. Calcule BD.
05 ABCD um quadriltero convexo, de permetro 2p. Podemos garantir
ento que AC + BD est entre: 13 O tringulo ABC equiltero de lado 3 cm. Toma-se sobre o lado BC o
ponto P. Seja Q o p da perpendicular de P a AB, R o p da perpendicular
(A) 0 e p. de Q a AC e S o p da perpendicular de R a BC. Calcule a distncia PB
(B) p e 2p. para que os pontos P e S coincidam.
(C) 2p e 3p.
(D) 3p e 4p. 14 Em um grande salo de baile h vrias pessoas espalhadas, e as
distncias entre elas so todas distintas. Uma pessoa A vai falar com outra
06 Em um quadriltero ABCD, no qual os ngulos ABC e ADC so retos, pessoa B se, dentre todas, B for a mais prxima de A, e depois retorna ao
tem-se que o ngulo ACD o dobro do ngulo ACB, e tambm AB = 2. lugar de origem. Uma pessoa por vez sai do seu lugar, e depois retorna.
Calcule a medida de AD, sabendo que um valor inteiro. Qual o nmero mximo de saudaes de pessoas diferentes que algum
pode receber?
(A) 4.
(B) 2.
(A) 2.
(C) 3.
(B) 3.
(D) 6.
(C) 4.
(E) 5.
(D) 5.
07 Considere ABC um tringulo equiltero. Prolongando BC de um (E) 6.
segmento CP qualquer, toma-se, sobre a altura de B no tringulo ABP,
um ponto Q tal que QB = 30. Calcule a medida do ngulo QPC. 15 No tringulo ABC, retngulo em A, a bissetriz interna de B intersecta a
altura AH em E e o lado AC em M. A bissetriz interna de C intersecta AH em
(A) 15. F e o lado AB em N. Se AM = 2 cm, NA = 6 cm, o valor de EF, em cm, :
(B) 20.
(C) 30. (A) 3.
(D) 35. (B) 4.
(E) 45. (C) 4,5.
(D) 5.
08 Em um tringulo issceles ABC de base AC e ngulo interno em B = (E) 6.
40, traa-se a ceviana interna CM e marca-se o incentro I do tringulo
MCB. Os ngulos AMC e IAC so congruentes. Calcule BI.

(A) 20.
(B) 40.
(C) 10.
(D) 50.
(E) 30.

242 Vol. 1
Tringulos

RASCUNHO

AFA-EFOMM 243
Quadrilteros A ssunto
4
Matemtica V

1. Definies e propriedades iniciais De fato, tambm possvel provar que, se uma dessas propriedades
satisfeita, ento o quadriltero necessariamente um paralelogramo.
Quadriltero o polgono de gnero 4. Sempre possui duas diagonais. Alm disso, vale que: se o quadriltero convexo ABCD tal que AB e CD
Pode ser polgono complexo, ou simples, e a, convexo ou no convexo so segmentos paralelos e congruentes, ento ele um paralelogramo.
como nas figuras abaixo. Caso seja simples, a soma dos seus ngulos D C
internos 360.
E K
D
I M
A

G
J A
H L B
B C F
Na figura, ABCD um paralelogramo.
ABCD um EFGH um IJKL um
quadriltero simples quadriltero simples quadriltero auto 4. Retngulo
e convexo e no convexo intersectante
Dizemos que um quadriltero retngulo se for um quadriltero
Diferente dos tringulos, os quadrilteros possuem uma estrutura no
equingulo. Dessa maneira, todos os seus ngulos sero retos. Como
to bem definida a partir de poucas informaes [dessa maneira, difcil
todos os ngulos so congruentes, em particular os opostos so iguais;
inclusive estabelecer critrios de congruncia de quadriltero, de forma
logo, todo retngulo paralelogramo, herdando suas propriedades.
que no discutiremos a respeito disso]. Por outro lado, alguns quadrilteros
possuem definies interessantes que levam a propriedades bastante teis, Uma propriedade que o retngulo tem a mais, diferente dos
e ento recebem nomes especiais. paralelogramos, que suas diagonais so sempre congruentes.
D C
2. Trapzio
Dizemos que se o #ABCD convexo tal que AB//CD, ento o O
quadriltero um trapzio de bases AB e CD. Caso os outros lados
opostos no sejam paralelos, a eles chamaremos lados oblquos.
A B
Existem 3 tipos especiais de trapzio:
Trapzio issceles: os lados oblquos so congruentes; ABCD um retngulo de centro O, o ponto O equidista dos
Trapzio escaleno: os lados oblquos no so congruentes; vrtices
Trapzio retngulo: um lado oblquo perpendicular s bases [costuma Obs.: Todo tringulo retngulo pode ser obtido atravs do trao de uma
ser chamado de altura]. diagonal em um retngulo. Logo, pode-se deduzir que a mediana relativa
B C E H I L hipotenusa sempre mede a metade dela. [Na figura, o tringulo ABD
retngulo, AO mediana relativa hipotenusa e vale a metade da diagonal,
ou seja, vale a metade de BD.]

5. Losango
A D F G J K
Dizemos que um quadriltero losango se for um quadriltero
ABCD um trapzio issceles, EFGH um trapzio equiltero. Dessa maneira, todos os seus lados so congruentes. Em
retngulo, IJKL um trapzio escaleno. particular, tem-se que os lados opostos so congruentes; logo, todo
losango um paralelogramo, herdando assim suas propriedades.
Uma estratgia comum em resoluo de problemas com trapzio
traar alguma paralela, normalmente a algum lado oblquo, quebrando um Uma propriedade que o losango tem a mais, diferente dos
trapzio em um tringulo e um paralelogramo. paralelogramos [e, portanto, tambm dos retngulos], que suas diagonais
so perpendiculares. Tambm so bissetrizes dos ngulos internos.
3. Paralelogramo D C
Dizemos que se o #ABCD tal que AB//CD e AD//BC, ento o
quadriltero um paralelogramo. Atravs de congruncia de tringulos,
I
podemos provar as seguintes propriedades:

Os pares de ngulos internos opostos so congruentes;


Os pares de lados opostos so congruentes; A B
As diagonais do paralelogramo se bissectam, isto , se cortam no
ponto mdio. ABCD um losango, o ponto I equidista dos lados

244 Vol. 1
Quadrilteros

6. Quadrado A B

Dizemos que um quadriltero quadrado se for um quadriltero M N


regular, isto , se for equingulo e equiltero ao mesmo tempo. Dessa
maneira, todo quadrado retngulo e losango ao mesmo tempo, portanto, D C
possui as propriedades deles.
MN a base mdia no trapzio
D C
ABCD

MN//AB//CD E MN = AB + CD
O 2
Observe que a base mdia passa pelos pontos mdios das diagonais
45 do trapzio. O segmento formado pelos pontos mdios das diagonais
do trapzio chamado de Mediana de Euler, e mede o mdulo da
B semidiferena entre as bases do trapzio.
A
ABCD um quadrado de centro O B
A B
A
7. Base mdia de tringulo Q Q P
P
Dado um tringulo ABC, sejam M e N os pontos mdios dos lados AB
e AC. Dizemos que o segmento MN uma base mdia relativa ao lado BC. D C D C
Nesse caso, valem as seguintes afirmativas: MN//BC e 2 MN = BC, ou
seja, a base mdia paralela e igual metade do lado ao qual ela relativa. PQ mediana de Euler em Sendo ABCD um trapzio, PQ med. de
ABCD Euler, vale que PQ//AB//CD e PQ =
A
CD AB
2

M N EXERCCIOS RESOLVIDOS
01. No trapzio ABCD, as bases AB e CD medem 17 e 32,
respectivamente. Sabendo que o ngulo interno em B o dobro do
B C encontrado em D, calcule a medida de BC.

MN a base mdia do ABC Soluo:


Considere P sobre a base CD de forma que BP//AD. Dessa
BC
MN//BC e MN = maneira, #ADPB paralelogramo, e, por transporte de ngulos
2 via paralelismo, os ngulos ADP, BPC e PBA so congruentes.
Como B o dobro de D, o ngulo PBC igual tambm. Com isso,
Se, em um tringulo ABC, M ponto mdio de AB, e N est sobre AC o tringulo BPC issceles, com BC = CP = CD PD = CD AB
de forma que MN seja paralelo a BC, ento necessariamente N ponto = 32 17 = 15.
mdio de AC.

A P 02. ABCD um quadriltero convexo qualquer. Prove que o


quadriltero formado pelos pontos mdios dos lados de ABCD
um paralelogramo.
K
M N Soluo:
J Sejam M, N, P e Q mdios de AB, BC, CD e DA, respectivamente.
No tringulo ABC, MN base mdia relativa a AC; logo, MN//AC e
B C Q I R MN = AC/2. Analogamente, no tringulo ACD, PQ // AC e
PQ = AC/2. Logo, MN//PQ e MN = PQ, o que caracteriza #MNPQ
M ponto mdio de AB JK a metade de QR como paralelogramo.
MN//BC implica N mdio de AC JK no a base mdia

EXERCCIOS NVEL 1
8. Base mdia de trapzio e mediana de Euler
Usando argumentos de base mdia de tringulo, podemos provar 01 Em um trapzio retngulo, as bases medem 8 cm e 18 cm. Se um
o seguinte resultado: se AB e CD so bases de um trapzio de lados dos ngulos internos do trapzio mede 45, ento a altura do trapzio :
oblquos AD e BC, e M e N so pontos mdios desses lados oblquos,
ento chamamos MN de base mdia do trapzio, e vale que MN//AB// (A) 12 cm. (D) 10 cm.
CD e MN mede a semissoma das bases, ou seja, MN = (AB + CD)/ 2. (B) 18 cm. (E) 9 cm.
(C) 13 cm.

AFA-EFOMM 245
Matemtica V Assunto 4

02 ABCD um paralelogramo tal que AB = 6 e AD = 8. A bissetriz do ngulo 04 ABCD um paralelogramo de lados medindo 7 cm e 10 cm. Calcule
interno em A corta BC no ponto E. Calcule a medida do segmento CE. o comprimento das diagonais do quadriltero formado pelas bissetrizes
dos ngulos internos de ABCD.
03 Dado o retngulo ABCD cuja diagonal mede 10 cm, inscreve-se nele
um paralelogramo que possui os lados paralelos s diagonais do retngulo. 05 ABC um tringulo acutngulo, H p da altura traada de A a BC, e
Qual o permetro do paralelogramo? M, N e P so os pontos mdios de AB, AC e BC, respectivamente. Calcule
o permetro do quadriltero MNPH, sabendo que AB = 8, BC = 12 e BH =
04 As bases de um trapzio escaleno medem 3 cm e 9 cm. Os segmentos 3.
determinados pelas diagonais do trapzio sobre a base mdia so
proporcionais aos nmeros: 06 No interior do tringulo ABC, toma-se um ponto I sobre a bissetriz de
A de forma que o ngulo AB reto. Sendo M ponto mdio de BC, calcule
(A) 1,1,1 (D) 1,4,1 IM, sabendo que AB = 15, AC = 19 e BC = 20.
(B) 1,2,1 (E) 2,3,4
(C) 1,3,1 07 Em um tringulo ABC, AM mediana. No tringulo ABM, traa-se a
mediana BP, que corta AC no ponto Q. Calcule a razo AQ : QC.
EXERCCIOS NVEL 2
(A) 1 : 1 (D) 2 : 1
01 No trapzio ABCD retngulo, a base menor AB mede a metade do lado (B) 1 : 2 (E) 2 : 3
oblquo BC. Sendo M mdio de BC, tem-se que o ngulo DMB mede 120. (C) 1 : 3
Calcule o ngulo interno em C no trapzio ABCD.
08 So dadas duas paralelas. De um ponto A de uma delas, traa-se a
02 As bases de um trapzio medem 10 cm e 8 cm e os lados no paralelos perpendicular comum AC e uma reta oblqua AB. Uma reta traada de B
medem 5 cm e x cm. Quais so os possveis valores de x? intersecta AC em E e a outra paralela em D, de forma que ED = 2 AB.
Sabendo que o ngulo ABC de 60, qual a medida do ngulo DBC?
03 As diagonais AC e BD de um quadriltero convexo ABCD se cortam num
ponto P. Os permetros dos tringulos ABC e ABD so iguais, bem como os 09 ABCD um quadrado de lado 2. Sobre os lados BC e CD marcam-se os
dos tringulos ACD e BCD. Mostre que ABCD um trapzio issceles. pontos M e N de forma que MN = 45. Calcule o permetro do tringulo MCN.

RASCUNHO

246 Vol. 1
Crculos A ssunto
5
Matemtica V

1. Definio e nomenclatura Um teorema importante: se P um ponto externo a um crculo, e PA e


PB so segmentos tangentes traados de P ao crculo, ento PA e PB so
Dizemos que a circunferncia de centro O e raio r o conjunto {P|OP = r}. congruentes. Alm disso, o segmento OP, que une o centro do crculo ao
O crculo de centro O e raio r o conjunto {P|OP r}. O crculo a unio da ponto P, bissetriz do ngulo APB.
circunferncia com a regio interna a ela, que, prova-se, uma regio convexa.
O comprimento de uma circunferncia de raio r dado por 2r, sendo
um nmero irracional, aproximado por 3.1415926535 (existem expresses
precisas para o clculo de ).

3. Posio relativa entre crculos


De acordo com a posio entre dois crculos, podemos classific-los
como:
2. Posies relativas a uma reta
Concntricos: so crculos que possuem o mesmo centro.
Como o crculo uma regio convexa, uma reta pode intersectar a Internos: um est totalmente contido no interior do outro.
circunferncia em zero, um ou dois pontos, no mximo. Dizemos, nessa Tangentes internos: um tangencia o outro internamente. (Suas
ordem, que ela externa, tangente ou secante ao crculo, de acordo com circunferncias s se intersectam em um ponto, no qual existe uma
o nmero de intersees com a circunferncia (0,1 ou 2). reta tangente comum.)
Secantes: as circunferncias se intersectam em dois pontos.
r: reta externa Tangentes externos: um tangencia o outro externamente. (No nico ponto
s: reta secante de interseo das circunferncias, existe uma reta tangente comum.)
l: reta tangente Externos: os crculos no se intersectam.

No caso em que crculos so tangentes, interna ou externamente, os


centros e o ponto de tangncia so sempre colineares. Dessa maneira,
fcil calcular a distncia entre os centros.
No caso em que crculos so externos, o menor segmento e o maior
segmento ligando os dois conjuntos esto contidos na reta que contm
os centros.

4. ngulos no crculo
Uma vantagem dos crculos a estrutura de transporte e clculo
facilitado de ngulos que podemos deduzir a partir de algumas definies.
Para o que segue, seja um crculo de centro O.

Demonstra-se que a reta tangente ngulo central


a um crculo perpendicular ao raio
traado no ponto de tangncia. Ou seja, Se A e B so pontos sobre a circunferncia,
se r tangente a uma circunferncia chamamos o ngulo AB de ngulo central. C
de centro O no ponto T, ento r Identificamos cada arco AB com seu respectivo
perpendicular a OT. ngulo central AB, de forma que diremos que a B
A
medida do arco igual medida do ngulo.

AFA-EFOMM 247
Matemtica V Assunto 5

Consequncias imediatas: A F
B
A circunferncia mede 360. F
P
Um dimetro define dois arcos de 180, que chamamos de E K
semicircunferncia. K
G
Se uma corda tem medida igual ao raio da circunferncia, ento o arco J
A B L
correspondente a ela mede 60. G
O comprimento de um arco proporcional ao ngulo central que o
determina.
^
Vejamos o caso em que A PB excentrico interno.

B A Traando a reta AB e usando ngulos inscritos



AB
AB ' .
temos:
= AB ' B = e B ' AA '
P 2 2
^
Como A PB externo do PAB : APB = ABB +
AB + AB
A BAA =
2
ngulo inscrito B
Obs.: Os outros casos podem ser demonstrados com a mesma ideia.
Se A, P e B so pontos da circunferncia, dizemos que o ngulo
APB ngulo inscrito na circunferncia (seu vr tice est sobre a Em problemas de polgonos regulares, ngulos entre diagonais podem
circunferncia). Por consequncia da definio anterior, junto com o ser facilmente calculados atravs dos ngulos excntricos. Lembre-se de
teorema do bumerangue, temos que o arco AB mede o dobro do ngulo que cada lado determina arcos iguais, e, fazendo as contas, de medida
APB, como mostra a figura. igual ao ngulo externo.
Dessa maneira, fixado um certo arco AB, todos os ngulos inscritos
que olham para esse arco tm a mesma medida [ideia do arco-capaz]. ngulo de segmento
Em particular, todos os ngulos inscritos em uma semicircunferncia
medem 90. Se uma reta XA tangente a um crculo no B
ponto A, e AB uma corda, dizemos que o ngulo 0
P P2 P3
C XB ngulo de segmento, e mede a metade do
P1 P4 arco AB contido na sua regio interna. A
x
P5 E
D De fato, basta considerar que a tangente o caso limite da secante, de
modo que vale o mesmo resultado do ngulo inscrito.
B
A 5. Arco capaz
Vejamos o caso em que O interno ao ngulo: Dado um segmento AB fixo, e um ngulo constante, o lugar
geomtrico dos pontos P tais que o ngulo APB mede o arco capaz
P Como OA = OP temos OP = O PA.
^
de sobre AB. Ele a nio de dois arcos de circunferncia congruentes,
^ ^ ^
Da figura, O PB = A PB O PA e como OP = OB com extremidades em A e B, como na figura.
^ ^ ^ ^
temos: O BP = O PB = A PB O PA. P
A ^
Do bumerangue OB = OP + A PB + O BP =
^
O ^
2 A PB. 30 O __
B Arco capaz de 30 sobre AB
__
Os pontos P e P olham AB
Obs.: O caso em que O externo pode ser feito de modo similar. A B
segundo um ngulo de 30
ngulos excntricos
Sendo O centro do arco capaz,
Se duas cordas AC e BD se intersectam no ponto P, interno ao crculo, temos AB = 2 = 60
dizemos que o ngulo APB excntrico interno, e vale a semissoma
30
dos arcos AB e CD para os quais tal ngulo olha. Caso as retas AC e BD
se intersectem fora do crculo no ponto P, dizemos que o ngulo APB
excntrico externo, e mede a semidiferena entre os arcos AB e CD, P
em mdulo.
Observe que um crculo de dimetro AB, excetuando os pontos A e B,
arco capaz de 90 sobre AB.

248 Vol. 1
Crculos

6. Quadriltero inscritvel EXERCCIOS RESOLVIDOS


Atravs dos resultados anteriores, podemos adquirir uma tcnica para 01 Fixam-se os pontos B e C, e faz-se variar um ponto A de forma
incluir pontos em uma circunferncia. Alm do mais, essa ferramenta ser que BC = 60. Determine o lugar geomtrico do incentro do tringulo
til para transportar ngulos. ABC.
Primeiro, observe que trs pontos colineares (ou ainda, um tringulo)
sempre determinam um crculo que passa por eles (crculo circunscrito Soluo:
a eles). Nem sempre, portanto, quatro pontos esto em um crculo por Tem-se que, como = 60, ento B + C = 120, logo (B+C)/2
exemplo. Veremos duas condies interessantes para que isso ocorra. = 60. Sendo I o incentro de ABC, no tringulo BIC, tem-se que BC
Dizemos que o #ABCD inscritvel quando existe uma circunferncia = 180 (B+C)/2 = 120. Como B e C so fixos, e o ngulo BC
que passa pelos vrtices. Para isso, existem dois critrios angulares constante e igual a 120, tem-se que I varia em um arco capaz de
iniciais: 120 sobre o segmento BC. (dado , tem-se BC=90 + A/2, como
visto no bloco de tringulos)
I. Um quadriltero inscritvel se, e somente se, a soma dos ngulos
internos opostos vale 180. 02 O tringulo ABC est inscrito em um crculo, e uma corda KL
II. Um quadriltero inscritvel se, e somente se, atende ideia do arco corta os lados AB e AC nos pontos P e Q. Sabendo que o #PQCB
capaz, ou seja, #ABCD inscritvel se, e somente se, os ngulos inscritvel, prove que o tringulo AKL issceles.
BAC e BDC so iguais.
Soluo:
D D J que o #PQCB inscritvel, ento os ngulos APQ e ACB so
A A
congruentes. Por soma de arcos, tem-se que, da, os arcos AK e AL
so congruentes, portanto as cordas AK e AL so congruentes. Segue

da o resultado.

B B 03 O crculo exinscrito relativo a BC no tringulo ABC tangencia a


reta AC em P. Prove que o segmento AP mede o semipermetro do
C C tringulo.

+ = 180 = Soluo:
Os ngulos opostos so ngulos olhando o mesmo Sejam X e Q os pontos de tangncia do exincrculo com os lados
complementares. arco iguais. BC e AB, respectivamente. Por segmentos tangentes iguais, tem-se:
CP = CX = x, BQ = BX = y, AP = AQ = c + x = y + b. Logo, no
tringulo ABC, 2p = AB + BC + AC = c + (x+y) + b = 2AP. Da,
7. Quadriltero circunscritvel AP = p, o semipermetro do tringulo.
Da mesma forma, podemos estabelecer critrios para que um
quadriltero tenha seus lados tangentes a uma circunferncia. 04 O crculo inscrito no tringulo ABC tangencia o lado AB no ponto
Todo tringulo possui um crculo inscrito, j que suas bissetrizes F. Prove que AF = p a, sendo p o semipermetro e BC = a.
so concorrentes. Em um quadriltero, nem sempre isso ocorre. Caso
ocorra, o ponto de encontro equidistante dos lados, logo centro de Soluo:
uma circunferncia que tangencia os lados. Sejam D, E e F os pontos de tangncia do incrculo com os lados
Existe o Teorema de Pitot: o #ABCD circunscritvel, ou seja, seus BC, AC e AB, respectivamente. Ento: AE = AF = x, BD = BF = y,
lados tangenciam uma circunferncia, se, e somente se, vale a seguinte CE = CD = z. Logo, no tringulo ABC, 2p = 2x + 2y + 2z, logo x
relao: AB + CD = AD + BC (as somas dos lados opostos so iguais). + y + z = p. Como BC = a = y + z, tem-se que AF = x = p a.

05 Prove que o simtrico do ortocentro de um tringulo em relao


a qualquer lado pertence ao crculo circunscrito ao tringulo.

Soluo:
Seja ABC o tringulo (considere por ora acutngulo), de ortocentro
H, e seja H* o simtrico de H com relao ao lado BC. Tem-se ento que
so congruentes os ngulos HBC e HAC (os dois so complementares
do ngulo C interno). Por simetria, HBC e H*BC so congruentes, logo
H*BC e HAC so congruentes. Como A, H e H* so colineares, tem-se
H*BC = H*AC. Logo o quadriltero H*BAC inscritvel, ou seja, H*
pertence ao circuncrculo de ABC. O caso em que ABC obtusngulo
pode ser tratado por analogia.

AFA-EFOMM 249
Matemtica V Assunto 5

EXERCCIOS NVEL 1 EXERCCIOS NVEL 2

01 dado um crculo A de raio 10 cm e dois crculos B e C tangentes a 01 Considere um tringulo ABC de ngulos internos iguais a 50, 60 e
A internamente, e externamente entre si. Calcule o permetro do tringulo 70. Sendo DEF o tringulo cujos vrtices so os pontos de tangncia do
formado pelos centros dos crculos A, B e C. incrculo de ABC com seus lados, calcule os ngulos do tringulo DEF.
02 ABC um tringulo cujo permetro 10 cm, e BC mede 4 cm. Sejam
02 AB dimetro de um crculo de centro O e raio 9 cm. Prolonga-se AB D e E sobre os lados AB e AC, respectivamente, tais que DE tangente ao
de um segmento BP, e traa-se uma secante PMN, com M e N sobre o incrculo de ABC. Calcule o permetro do tringulo ADE.
crculo, de forma que PM = OA. Calcule o comprimento de NA, sabendo
que o ngulo BPM mede 20.
03 ABC um tringulo escaleno retngulo em A, e AD a bissetriz interna,
com D sobre BC. Toma-se o segmento DE perpendicular a BC, com E sobre
03 Um tringulo acutngulo ABC est inscrito em um crculo, de forma
o lado AC. Calcule o ngulo BD.
que AB congruente ao lado do tringulo equiltero inscrito nesse crculo,
e BC congruente ao lado do quadrado inscrito nesse crculo. Calcule o
04 dado um crculo de centro C, e um ponto O externo a ele. Os segmentos
maior ngulo interno do tringulo ABC.
traados de O tangentes ao crculo so OX e OY. Traa-se tambm uma secante
04 O quadriltero ABCD convexo inscritvel, e sua regio interna contm OBA, cortando a circunferncia em B e A, com B entre O e A. Sabendo que o
o centro do crculo no qual est inscrito. Sabe-se que AB e CD tm medidas arco AX o dobro do arco XB e o ngulo XOY reto, calcule o ngulo BX.
iguais s do quadrado e do enegono inscritos nesse crculo. Calcule o
ngulo formado pelas diagonais de ABCD. 05 dada uma circunferncia e um ponto P, externo ao crculo, a partir
do qual se traam os segmentos tangentes PC e PD. Marca-se sobre a
05 ABCDEFGHIJKL um polgono regular. Calcule o ngulo formado: circunferncia um ponto A, e sobre AD um ponto B tal que AB = BC. Dessa
maneira, BC = 80. Calcule o ngulo PBC.
(A) pelas diagonais AC e BD.
(B) pelas diagonais BE e DH. (A) 50.
(C) pelos prolongamentos dos lados CD e HI. (B) 40.
(D) pelos prolongamentos das diagonais BD e HK. (C) 80.
(D) 60.
06 Um tringulo ABC est inscrito num crculo de raio 6 cm, e seu (E) 65.
permetro mede 16 cm. Sabendo que = 30, ento a soma AB + AC
igual a: 06 Duas circunferncias de raios R e r se cortam nos pontos A e B. Em
cada circunferncia, a partir do ponto B, traam-se cordas BQ e BN, que
07 Um hexgono regular ABCDEF e um pentgono regular AXYZW cortam a outra circunferncia nos pontos P e M [de forma que Q e M
esto inscritos em uma mesma circunferncia. Sabendo que o arco CY esto na circunferncia de raio R, e P e N esto na circunferncia de raio
menor que 90, e que CY lado de um polgono regular inscrito nessa r]. Sabe--se que AB bissetriz do ngulo QBN. Assinale a proposio
circunferncia, determine o nmero de diagonais desse polgono. verdadeira:
08 ABC um tringulo issceles de base BC, e ABPQ um quadrado (A) se R > r, ento PQ > MN.
construdo externamente a ele. Calcule a medida do ngulo QCB. (B) PQ = MN se, e somente se, R > r.
09 A e B so dois pontos de uma circunferncia que a dividem em arcos (C) sempre PQ = MN.
de medidas proporcionais a 3 e 7. As tangentes traadas por A e B formam (D) AQ = NA e AM = AP.
um ngulo igual a: (E) N.R.A.
(A) 60. 07 A, B e C so trs pontos de um crculo. A bissetriz do ngulo ABC
(B) 66. intersecta o crculo em D. Do ponto D traa-se uma corda paralela a AB que
(C) 72. intersecta o crculo em E. Se DE = 3 cm, quanto mede o segmento BC?
(D) 78.
(E) N.R.A.

10 Em um quadriltero ABCD convexo, os vr tices B, C e D so


equidistantes do vrtice A. Se = 140, o ngulo C mede:

(A) 40.
(B) 110.
(C) 50.
(D) 140.
(E) 70.

250 Vol. 1

Você também pode gostar